Download as pdf or txt
Download as pdf or txt
You are on page 1of 204

Pharmacy Prep

Biomedical Sciences

Q&A
Misbah Biabani, Ph.D
Director
Toronto Institute of Pharmaceutical Sciences (TIPS) Inc.
Toronto, ON M2N K7

Pharmacy Prep
Professional Exams Preparation Center
4789 Yonge St. Suites 415-417 Toronto ON M2N 5M5, Canada
WWW.PHARMACYPREP.COM
416-223-PREP (7737) / 647-221-0457
Toronto Institute of Pharmaceutical Sciences Inc.
© 2000- 2020 TIPS Inc. All Rights Reserved.

Copyright © 2000-2020 TIPS Inc. Unauthorized reproduction of this manual is strictly prohibited and
it is illegal to reproduce without permission. This manual is being used during review sessions
conducted by PharmacyPrep
Disclaimer

Your use and review of this information constitutes acceptance of the following terms and
conditions:
The information contained in the notes intended as an educational aid only. It is not intended
as medical advice for individual conditions or treatment. It is not a substitute for a medical
exam, nor does it replace the need for services provided by medical professionals. Talk to your
doctor or pharmacist before taking any prescription or over the counter drugs (including any
herbal medicines or supplements) or following any treatment or regimen. Only your doctor or
pharmacist can provide you with advice on what is safe and effective for you. Pharmacy prep
make no representation or warranty as to the accuracy, reliability, timeliness, usefulness or
completeness of any of the information contained in the products. Additionally, Pharmacy prep
do not assume any responsibility or risk for your use of the pharmacy preparation manuals or
review classes.
In our teaching strategies, we utilize lecture-discussion, small group discussion,
demonstrations, audiovisuals, case studies, written projects, role play, gaming techniques, study
guides, selected reading assignments, computer assisted instruction (cai), and interactive video
discs (ivd).
Our preparation classes and books does not intended as substitute for the advise of
NABPLEX®. Every effort has been made to ensure that the information provided herein is not
directly or indirectly obtained from pebc® previous exams or copyright material. These
references are not intended to serve as content of exam nor should it be assumed that they are
the source of previous examination questions.
©2000-2020 TIPS. All rights reserved.

Foreword by
Misbah Biabani, Ph.D
Coordinator, Pharmacy Prep
Toronto Institute of Pharmaceutical Sciences (TIPS) Inc
4789 Yonge St. Suites 415-417
Toronto ON M2N 5M5, Canada

Copyright © 2000-2020 TIPS Inc. Unauthorized reproduction of this manual is strictly prohibited and
it is illegal to reproduce without permission. This manual is being used during review sessions
conducted by PharmacyPrep
Part 1: Biomedical Sciences 15%
1. Human Anatomy
2. Gastrointestinal System
3. Nervous System
4. Cardiovascular System
5. Endocrine System
6. Renal System
7. Liver Function and Pathophysiology
8. Respiratory System
9. Urinary System
10. The Eye and Ear
11. Blood and Anemia
12. Biochemistry
13. Nutrition
14. Microbiology
15. Cell and Molecular Biology
16. Pharmacogenetics
17. Immunology and Immunizations
18. Biotechnology
19. Toxicology

Copyright © 2000-2020 TIPS Inc. Unauthorized reproduction of this manual is strictly prohibited and
it is illegal to reproduce without permission. This manual is being used during review sessions
conducted by PharmacyPrep
Copyright © 2000-2020 TIPS Inc. Unauthorized reproduction of this manual is strictly prohibited and
it is illegal to reproduce without permission. This manual is being used during review sessions
conducted by PharmacyPrep
Human Anatomy
PHARMACY PREP
HUMAN ANATOMY
1. What is the opposite of anterior view? opposite opposite
A. Ventral view Abduction Adduction
B. Dorsal view Extension Flexion
C. Lateral view Lateral Medial
Dorsal (posterior) Ventral (anterior)
D. Medial view Proximal Distal
E. Distal view Prone Supine
Ans: B

2. Abduction is an anatomical movement of:


A. A muscle, which moves a body part away from the midline of the body.
B. A muscle, which moves a body part toward the midline of the body.
C. A muscle, which moves a body part downward the body.
D. A muscle, which moves a body part upward the body.
Ans: A
Tips: Abduction is moving away from middle line.
Strategic thinking: in answering this type of question:
1. Read the question.
2. Pick out the keyword. Determine the root word from the keyword. In this
case, it’s ABDUCT, which in English means, take away.
3. Choose the option that contains the word. In anatomy, there’s always almost
a term referring to the opposite meaning. To avoid confusion, memorize ONE
of these terms only as the other (as in this case, ADDUCTION) will always
mean the opposite.
(More terminologies in Anatomy chapter are found in your book in which you can apply
the same strategy for an easier and effective learning)

3. Adduction is an anatomical movement of:


A. A muscle, which moves a body part away from the midline of the body.
B. A muscle, which moves a body part toward the midline of the body.
C. A muscle, which moves a body part downward the body.
D. A muscle, which moves a body part upward the body.
Ans: B

4. Movement of limb or body part closer to or towards the mid line of the body is:
A. Abduction
B. Adduction
C. Extension
D. Flexion
Ans: B

Copyright © 2000-2020 TIPS Inc. Unauthorized reproduction of this manual is prohibited. This manual is being
used during review sessions conducted by PharmacyPrep.
1-1
Human Anatomy
5. Planes and sections of the body: Separates the body into equal right and left portions is
referred to as:
A. Sagittal plane
B. Mid-sagittal plane
C. Coronal plane
D. Parasagittal plane
Ans: B
Tips: Sagittal plane separates the body into right and left portions
Midsagittal plane (Median plane) separates the body/body part into equal right/left
portions
Parasagittal plane separates the body into unequal right and left portions.
Coronal plane separates the body/body-part into anterior and posterior portions.
Transverse plane divides the body/body-part into superior and inferior portions.
Oblique plane passes through the body/body-part at an angle

6. Epithelial tissue is present in all of the following EXCEPT:


A. Sweat gland
B. Milk gland
C. Endocrine gland
D. Blood vessel
E. Bile
Ans: D
Tips: Blood vessel, arteries, veins, and capillaries have endothelial cells. Epithelial
tissue: Covering/lining or glandular, are 2 basic types endocrine "ductless" produce
hormones. Exocrine have ducts, sweat, oil, saliva, bile enzymes, mucin (mucus).

7. Intramuscular injections are often given in the following areas, EXCEPT:


A. Deltoid muscle of the arm
B. Vastus lateralis muscle of the thigh
C. Ventrogluteal muscle of the hip
D. Biceps muscle of the inner arm
E. Dorsogluteal muscle of buttock
Ans: D
Tips: The most typical site for vaccine is deltoid muscle of the arm.

8. All of the following have skeletal muscles, EXCEPT:


A. Arm
B. Biceps Tips: Heart is made of cardiac tissues. Arm, biceps, triceps and
C. Triceps foot is consisting of skeletal muscle.
D. Heart
E. Quadriceps
Ans: D

Copyright © 2000-2020 TIPS Inc. Unauthorized reproduction of this manual is prohibited. This manual is being
used during review sessions conducted by PharmacyPrep.
1-2
Human Anatomy
9. Anterior excruciate ligament is found in?
A. Shoulder
B. Knee
C. Hip
D. Ankle
E. Arm
Ans: B
Tips: Anterior excruciate ligament connects to femur, tibia and patella.

10. What cavity has lumbar vertebrae?


A. Ventral cavity
B. Dorsal cavity
C. Thoracic cavity
D. Abdominal cavity
E. Pelvic cavity
Ans: B
Tips: dorsal cavity consist of cranial cavity and vertebral cavity.

11. Pelvic cavity consists of all, EXCEPT:


A. Rectum
B. Reproductive organs
C. Urinary bladder
D. Kidney
E. Uterus
Ans. D
Tips: Kidneys are present in abdominal cavity.
Strategic thinking: in answering this type of question:
1. Read the question. Be very mindful of the terms like EXCEPT, LEAST, MOST.
These are clues that lead you to the best option.
2. Picture out the location of the body part being asked. From the choices,
imagine where these organs are located.
3. The question asked for a body part that is NOT within the pelvic cavity. The
kidneys are situated higher anatomically, found particularly in the abdomen.

12. The pelvis bones or hip joints have?


A. Pubis, ilium, and ischium
B. Scapula, ilium and ischium
C. Humerus, ilium, and ischium
D. Ligament, humerus, scapula
Ans: A
Tips: Hip consists of pubis, ilium and ischium bones. Humerus bone connects from
shoulder to elbow.

Copyright © 2000-2020 TIPS Inc. Unauthorized reproduction of this manual is prohibited. This manual is being
used during review sessions conducted by PharmacyPrep.
1-3
Human Anatomy
13. Which of the following does NOT include the cranial bones?
A. Facial bones
B. Parietal
C. Temporal
D. Occipital “PEST OF”
E. Ethmoid Parietal (2), Ethmoid, Sphenoid,
Ans: A Temporal (2), Occipital, AND Frontal.

14. All of the following are in facial bones, EXCEPT:


A. Lacrimal
B. Mandible
C. Vomer
D. Hyoid
Ans: D
Tips: Hyoid bone is horse shoe shape bone situated in anterior middle line of neck.
Facial bones: Nasal (2), Lacrimal (2), Inferior nasal concha (2), Maxilla (2), Mandible (1),
Vomer (1), zygomatic (2) and Palatine (2).

15. "Articulations" refer to:


A. Bone formation
B. A joint between two bones
C. Bone death
D. Structures on bones where muscles attach
Ans: B
Tips: A joint between two bones is referred to as articulations.

16. Which of the following opposites of anatomical views are incorrect?


A. Dorsal and frontal (ventral)
B. Lateral and medial
C. Prone and Supine
D. Sagittal and para sagittal
Ans: D

17. Which of the following is not part of the axial skeleton?


A. Femur
B. Sternum
C. Mandible
D. Sacrum
E. Vertebral column
Ans: A
Tips: The axial skeleton consist of 80 bones from head, the sternum, ribs, vertebral
column. The femur is thigh bone and not from axial skeleton.

Copyright © 2000-2020 TIPS Inc. Unauthorized reproduction of this manual is prohibited. This manual is being
used during review sessions conducted by PharmacyPrep.
1-4
Human Anatomy
18. Incomplete closure of the vertebral column results in:
A. Kyphosis
B. Scoliosis
C. Sinusitis
D. Spina bifida
E. Sciatic
Ans: D
Tips: It is birth defect with incomplete closing the backbone and membrane around
spinal cord. Cause by teratogenicity of Valproic acid, phenytoin.

19. Which of the following joint is "multiaxial ball and socket " type?
A. Hip joints
B. Knee joints
C. Elbow joint
D. Shoulder and hip joint
Ans: D

20. What are the following muscle fibers that help in muscle contraction?
A. Actin and myosin
B. Myosin and inhibin
C. Inhibin and actin
D. Actin, myosin and inhibin
Ans: A
Tips: Actin and myosin fibers cause contractions. Inhibin is a hormone released from
maturing follicles and corpus luteum.

21. A muscle which can both flex the hip and extend to the knee is?
A. Sartorius
B. Rectos femoris
C. Semimembranosus
D. Biceps femoris
Ans: B
Tips: Rectus femoris is muscle in the quadriceps, the rectus femoris is attached to the hip
and helps extend or raise the knee. This muscle also used to flex the thigh. The rectus
femoris is the only muscle that can flex the hip.

22. Which of the following physical activity is NOT a weight bearing exercise?
A. Walking
B. Jogging
C. Running
D. Swimming
Ans: D

23. All are examples of weight bearing joints, EXCEPT:

Copyright © 2000-2020 TIPS Inc. Unauthorized reproduction of this manual is prohibited. This manual is being
used during review sessions conducted by PharmacyPrep.
1-5
Human Anatomy
A. Hip joints
B. Knee joints
C. Skull joints
D. Shoulder joints
Ans: C

24. Which of the following bones is not part of the cranium?


A. Sphenoid
B. Palatine
C. Ethmoid
D. Occipital
Ans: B
Tips: The palatine bones are two irregular bones of the facial skeleton in many animal
species. Together with the maxillae they comprise the hard palate. They are situated at
the back part of the nasal cavity between the maxilla and the pterygoid process of
the sphenoid bone.

25. Which of the following exercises provide the least help in the prevention of osteoporosis
or improvement of bone mineral density?
A. Jogging
B. Walking
C. Rope jumping
D. Swimming
E. Horse riding
Ans: D
Tips: Swimming is not a weight bearing exercise and thus does not improve bone mineral
density. However swimming good exercise.

26. Melanin is a dark color light sensitive pigment that is found in skin which protects us
from?
A. Microorganism
B. UV radiation
C. Excretion of minerals
D. Thermal regulation
Ans: B

27. Which of the following has incorrect combination of bones to a specific organ?
A. Pubic, ilium and ischium
B. Radius, ulnar and humerus
C. Femur and tibia and fibula
D. Ball-and-socket joint, hinge joint
Ans: D
Tips: Ball -and-socket joints are found in the shoulder and hips. Hinge joints are in knees.

Copyright © 2000-2020 TIPS Inc. Unauthorized reproduction of this manual is prohibited. This manual is being
used during review sessions conducted by PharmacyPrep.
1-6
Human Anatomy
28. Synovial joints include all, EXCEPT:
A. Hinge joint (elbow joint)
B. Saddle joint (thumb joint)
C. Gliding joint (wrist joint)
D. Neck bones
E. Knee joint
Ans: D
Tips: Ligaments are type of fibrous bones surrounded with synovial fluids in joints such
as hip, elbow, thumb, wrist, and knee joints.

29. Popliteal vein pain is an indicator of?


A. Deep vein thrombosis
B. Intermittent claudication
C. Raynaud’s phenomenon
D. Coronary artery pain
Ans: A

30. What is perleche?


A. Dry lips
B. Cuts in lips
C. Cuts at corner of mouth
D. White fluid secretion at the corners of the mouth
Ans: C

31. Which of the following is a muscle fiber?


A. Femur
B. Patella
C. Starch
D. Actin
Ans: D

32. Ligaments?
A. Connect bone to bone
B. Connect bone to muscle BONE TO BONE = LIGAMENT
C. Connect to muscle to muscle BONE TO MUSCLE = TENDON
D. Connect arteries to bones
Ans: A

33. A sprain is an injury to?


A. Bones
B. Tissues
C. Ligament
D. Cartilage
E. Muscles

Copyright © 2000-2020 TIPS Inc. Unauthorized reproduction of this manual is prohibited. This manual is being
used during review sessions conducted by PharmacyPrep.
1-7
Human Anatomy
Ans: C
Tips: A sprain is an injury to a ligament caused by over stretching or twisting. The ankle
is a common area for sprains. It is mainly the lateral ligaments of the joints that are
involved.

34. A strain is an injury to?


A. Bones
B. Ligament
C. Cartilage
D. Muscles
E. Tendons
Ans. E
Tips: Strain is an injury to a muscle (tear in tendon), which is also referred as torn or
pulled muscle. It is a result of overstretching.

35. What organ is found on the left side of the body?*


A. Liver
Right upper Left upper
B. Spleen
Gallbladder, pancreas Spleen
C. Appendix
D. Gall bladder
Right lower Left lower
E. Pancreas
Appendix Sigmoid colon
Ans: B

IBS, IBD, Prostatitis, colorectal Left lower Refer to physician


cancer, pelvic inflammatory disease. quadrant
Urinary obstructions
IBS, IBD, gastroenteritis, ectopic Periumbilical Refer to physician
pregnancy
IBS, IBD, appendicitis Right lower Refer to
quadrant physician/emergence
Dyspepsia, GERD, pancreatitis Left upper
Liver, (hepatitis), gallbladder Right upper
(cholecystitis), pancreatitis.

36. What medications cause femur bone pain?


A. Proton pump inhibitors
B. Antihistamine
C. Bisphosphonates
D. Anticholinergic
Ans: C

37. What is normally found in all connective tissues?


A. Albuminoids
B. Lipids

Copyright © 2000-2020 TIPS Inc. Unauthorized reproduction of this manual is prohibited. This manual is being
used during review sessions conducted by PharmacyPrep.
1-8
Human Anatomy
C. Polysaccharide
D. All of the above
Ans: D

38. What is true about sciatic nerve?


A. Innervate all foot
B. Innervate the hips
C. Innervate buttock and large muscle of thigh
D. Innervate the quadriceps
E. Innervates heel
Ans: C

39. Which of the following bones is considered a sesamoid bone?


A. Sternum
B. Ethmoid
C. Femur
D. Patella
Ans: D
Tips: Sesamoid bones are embedded into tendons or soft bones of knee joints. The
patella and fabella are sesamoid bones.

40. Which of the following is not associated with pelvic area?


A. Cystitis
B. Urethritis
C. Pyelonephritis
D. Abdomen
E. Inguinal hernia
Ans. D
Tips: pelvic cavity: reproductive organs, bladder, urethra, prostate
Abdominal cavity: Gastrointestinal system, stomach, small intestine, large intestine,
kidneys, liver, gall bladder, pancreas, spleen
Thoracic cavity: lungs, chest.

Copyright © 2000-2020 TIPS Inc. Unauthorized reproduction of this manual is prohibited. This manual is being
used during review sessions conducted by PharmacyPrep.
1-9
Human Anatomy

Copyright © 2000-2020 TIPS Inc. Unauthorized reproduction of this manual is prohibited. This manual is being
used during review sessions conducted by PharmacyPrep.
1-10
Gastrointestinal System

PHARMACY PREP
GASTROINTESTINAL SYSTEM

1. What are the parts of small intestines in sequence from stomach to rectum?
A. Duodenum, Jejunum and Ileum
B. Jejunum, Duodenum and Ileum DJI = Duodenum, jejunum and ileum
C. Ileum, Duodenum and Jejunum
D. Duodenum, Ileum and Jejunum
Ans: A

2. The lower part of gut contains the following microorganisms.


A. 10 to 50% anaerobic
B. 90 to 99% anaerobic
C. 10 to 50% aerobic
D. 90 to 99% aerobic
Ans: B
Tips: The colon has the highest anaerobic bacteria, and it can range from 90 to 99%.

3. When piece of GI organ pushes up through the diaphragm opening in to chest, it is called?
A. Hernia hiatus
B. Hernia inguinal
C. Umbilical hernia
D. Femoral hernia
Ans: A

4. Which part of the GI tract has the least pH?


A. Esophagus
B. Stomach
C. Small intestine
D. Colon
Ans. B
Tips: The stomach has the least pH, which makes it the most acidic ranging from pH 1.5 to 3.5.

5. What is the most basic (alkaline) section of the gastrointestinal tract?


A. Esophagus
B. Stomach
C. Small intestine
D. Colon
Ans: D
Tips: Colon is the most basic part of the GI tract.

6. Intrinsic factor is released from?


A. G cell
B. Stomach

Copyright © 2000-2020 TIPS Inc. Unauthorized reproduction of this manual is prohibited. This manual is being used during
review sessions conducted by PharmacyPrep.
2-1
Gastrointestinal System

C. Parietal cell
D. Mucus cell
E. Pancreas
Ans: C
Tips: Parietal cells (also known as oxyntic or delomorphous cells), are the epithelial cells that
secrete hydrochloric acid (HCl) and intrinsic factor. They are located in the gastric glands found
in the lining of the fundus and in the body of the stomach. They contain an extensive secretory
network (called canaliculi) from which the HCl is secreted by active transport into the stomach.

7. Deficiency of parietal cell can cause?


A. Deficiency of vitamin D
B. Deficiency of vitamin K
C. Achlorhydric
D. Chlorhydric
Ans: C
Tips: Parietal cell produces HCL and intrinsic factors. Achlorhydria or hypochlorhydria refers to
states where the production of hydrochloric acid in gastric secretions of the stomach and other
digestive organs is absent or low, respectively.
Strategic thinking: in answering this type of question:
1. Read the question.
2. Make an association of the function of the cells, in this case parietal, with what is
expected to happen if the body has less of these. As discussed from the TIPS, parietal
cells play a major role in the stomach’s acidity.
3. Choose the option that describes a condition of low acidity, as this is the
consequence of decreased parietal cells.
(Refer from your Misbah’s Evaluating review book the cells that play an important function in
the normal physiology of the body system they belong. Knowing these would help you
determine the consequences in cases where there is either deficiency or excess of these. In
addition, note the surgical alteration/removal of the organs that secrete these cells as this will
largely aid you analyze problems expected to arise when a patient is under medication.
Example: Gastrectromy: partial or complete removal of the stomach. Patient is taking an
antifungal, (i.e. Ketoconazole) which requires acidic environment for optimal absorption. )

8. A patient with intrinsic factor deficiency developed megaloblastic anemia (pernicious anemia).
What is the correct recommendation?
A. Oral vitamin B 12 supplements
B. Antacids with vitamin B 12
C. Vitamin B 12 injections
D. Vitamin B 12 with acidic drugs
Ans: C

9. Maltase breaks down maltose to?*


A. Glucose +fructose
A. Glucose +fructose  sucrase
B. Glucose + glucose
B. Glucose + glucose  maltase
C. Glucose + Starch  amylase
Copyright © 2000-2020 TIPS Inc. Unauthorized reproduction
D. Glucose of this manual
+ galactose is prohibited. This manual is being used during
lactase
review sessions conducted by PharmacyPrep.
E. Starch + sucrose  alpha glucosidase 2-2
Gastrointestinal System

C. Glucose + Starch
D. Glucose + lactose
E. Starch + sucrose
Ans. B
Tips: Maltase is the enzyme that breaks down maltose to two glucose units.
ENZYME CARBOHYDRATE PRODUCT
Amylase Starch and glycogen Maltose
Maltase Maltose Glucose + glucose
Sucrase Sucrose Glucose + Fructose
Lactase Lactose Glucose + galactose
Strategic thinking: in answering this type of question:
1. Read the question.
2. Look for keyword. In this case, BREAKS. So, we are looking for products that are
simpler in form than the one mentioned in the question. In this case, we know that
Maltose is a DISACCHARIDE. We are looking for (a) monosaccaride/s which is/are
the basic unit of carbohydrates. Options C, D and E are wrong because the
combination of the carbohydrates mentioned include disaccharide (lactose) and
complex carbohydrates (starch) in them.
3. Correct answer is B. this type of questions requires both memorization and analysis.
Memorize the basic units of a product.

10. Celiac disease is caused by?


A. Deficiency of gluten
B. H. pylori infection
C. It is type of inflammatory bowel disease
D. None of the above
Ans: D
Tips: Celiac disease is due to sensitivity to gliadin fraction of ingested gluten, resulting in
immunologically mediated inflammatory damage to the lining of the small intestine in
genetically predisposed individuals. The inflammation may lead to malabsorption by reducing
the amount of surface area available for absorption of nutrients, fluids and electrolytes.

11. Steathorrhea can result to?


A. Fat absorption impaired
B. Indigestion of carbohydrate
C. Secretion of vitamin A, D, E and K
D. Excessive absorption of lipids
Ans: C
Tips: Steatorrhea can result into excessive secretion of fat soluble vitamins A, D, E and K. Drugs
like Orlistat (Xenical) can cause steatorrhea.

12. Ulcerative colitis occurs at?


A. Colon
B. Stomach

Copyright © 2000-2020 TIPS Inc. Unauthorized reproduction of this manual is prohibited. This manual is being used during
review sessions conducted by PharmacyPrep.
2-3
Gastrointestinal System

C. All GI tract
D. Small intestine
E. Esophagus
Ans: A

13. Excessive acid secretion causes?


A. Crohn’s disease
B. Peptic ulcers
C. Ulcerative colitis (inflammatory)
D. H. pylori ulcers
E. Irritable bowel disease
Ans: B

14. Irritable bowel syndrome (IBS) symptoms include all, EXCEPT:


A. Diarrhea
B. Constipation
C. GI bleeding
D. Bloating
E. GI upset
Ans: C
Tips: Irritable bowel syndrome (IBS) is a chronic disorder with symptoms of bloating, cramps,
constipation/ diarrhea, N&V symptoms. On the other hand, Inflammatory Bowel Disease (IBD)
and (Crohn's disease and UC) may cause bleeding and diarrhea.

15. Cystic fibrosis is a?


A. Disorder of pancreatic secretion.
B. Disorder lung function.
C. Disorder of bones.
D. Disorder of electrolyte imbalance.
Ans: B
Tips: Cystic fibrosis (CF) is the most common, fatal genetic disease affecting young Canadians.
CF is a multi-organ disease affecting primarily the lungs and the digestive system. In the lungs,
CF causes severe breathing problems. A build-up of thick mucus makes it difficult to clear
bacteria and leads to cycles of infection and inflammation, which damage the delicate lung
tissues. They must follow a demanding daily routine of physical therapy to keep the lungs free of
congestion and infection. In the digestive tract, CF makes it extremely difficult to digest and
absorb adequate nutrients from food.

16. Why are most drugs absorbed from the small intestine?
A. Because drug first enters the small intestine.
B. Because small intestine has large surface area.
C. Because small intestine is not acidic.
D. Because small intestine is small.

Copyright © 2000-2020 TIPS Inc. Unauthorized reproduction of this manual is prohibited. This manual is being used during
review sessions conducted by PharmacyPrep.
2-4
Gastrointestinal System

Ans: B
Tips:
The large surface area of the small intestine is due to the unique structural features of its inner
wall or mucosa that is covered in wrinkles or folds called plicae circulares, from which
microscopic finger-like pieces of tissue called villi project, and each villus also has finger-like
projections called microvilli. The small intestine also has rugae that allow distention and
contraction. These features are designed to increase the amount of surface area available for
the absorption of nutrients, and to limit the loss of said nutrients to intestinal fauna.

17. Crohn’s disease commonly occurs at the?


A. Colon
B. Stomach
C. Terminal small intestine and colon
D. Small intestine
Ans: C
Tips: Crohn’s commonly occurs at terminal small intestine and colon.

18. Which of the following protein digestive enzyme is released from pancreas into the small
intestine?
A. Pepsin
B. Trypsinogen
C. Maltase
D. Sucrase
Ans: B
Tips: Pancreas produces 3 enzymes amylase, trypsin and lipase and released as pancreatic juice
into the small intestine. The other enzymes produced in cells converting villi in small intestine’s
ileum are maltase, sucrase, lactase, peptidase, and lipase.

19. What food/diet induces HCl secretion?


A. Carbohydrate
B. Fat
C. Proteins
D. Vitamins
Ans: C
Tips: Proteins stimulate gastrin production and gastrin secretes HCl from parietal cells.

20. About Calcium, all are correct, EXCEPT:


A. Calcium is absorbed in all parts of small intestine.
B. Two mechanisms of absorption are active transport and passive absorption.
C. Lactose and vitamin D enhance calcium absorption.
D. The efficacy of absorption increases as calcium intake increases.
Ans: D
Tips: The efficacy of absorption decreases as calcium intake increases. High doses of Calcium
supplements like 1500 mg have to be divided 500 mg tid.

Copyright © 2000-2020 TIPS Inc. Unauthorized reproduction of this manual is prohibited. This manual is being used during
review sessions conducted by PharmacyPrep.
2-5
Gastrointestinal System

21. What is not true about the gastrointestinal system in elderly populations?
A. Increased pH
B. Deficiency of intrinsic factors
C. Increase GI motility
D. Deficiency of Vitamin B 12
E. Vitamin B12 injection is recommended
Ans: C

22. A customer of your pharmacy presents with a prescription of Loperamide and Lactulose. He is
experiencing abdominal pain and bloating. He has hard-bullet like stools followed by diarrhea.
He may be experiencing?
A. Ulcerative colitis
B. Crohn’s disease
C. Inflammatory bowel disease
D. Irritable Bowel Syndrome
Ans: D
Tips: Ref: CTC e-therapeutics
Symptoms of IBS include abnormal stool frequency (a) <3 bowel movements per week or (b) >3
bowel movements per day; abnormal stool form (c) lumpy/hard stool or (d) loose/watery stool;
(e) defecation straining; (f) urgency or also feeling of incomplete bowel movement, passing
mucus and bloating.

23. What is NOT a symptom of Irritable Bowel Syndrome (IBS)?


A. Constipation
B. Bleeding from gut
C. Nausea and vomiting
D. abdominal cramps, and pain
Ans: B

24. Which of the following GI conditions is NOT associated with GI bleeding?


A. Gastritis
B. Gastroenteritis
C. Pancreatitis
D. GI cancer
E. Inflammatory bowel disease
Ans: C
Tips: Pancreatitis has no GI bleeding and pancreatitis symptoms are upper abdominal pain that
radiates into the back. It may be aggravated by eating, especially foods high in fat. It is
characterized by tender abdomen, nausea and vomiting and increased heart rate.

25. Chronic alcoholism may cause?


A. Gastritis
B. Crohn's disease

Copyright © 2000-2020 TIPS Inc. Unauthorized reproduction of this manual is prohibited. This manual is being used during
review sessions conducted by PharmacyPrep.
2-6
Gastrointestinal System

C. Peptic ulcer
D. GI cancer
Ans: A
Tips: NSAIDs, smoking and chronic alcoholism cause gastritis. Gastritis causes inflammation of
the GI lining. The most common cause of gastritis in Canada is due to chronic alcoholism.

26. What type of hernia is common in seniors or elderly?


A. Hiatal hernia
B. Sliding hernia
C. Para esophageal hernia
D. Inguinal or groin hernia
Ans: D
Tips: Inguinal or groin hernia is the most common in middle to elderly age people.

27. Fistula or granulomas or skip patches occur in?


A. Irritable bowel disease
B. Ulcerative colitis
C. Crohn's disease
D. Gastritis
Ans: C

28. Polypeptides and caffeine stimulate secretion of?


A. Parietal cells
B. Vitamin B 12
C. Gastrin
D. Chyme
Ans: C

29. When parietal cells are stimulated, they secrete?


A. Intrinsic factor and gastrin
B. Intrinsic factor and HCl
C. Intrinsic factor and HCO 3
D. Intrinsic factor and mucus
Ans: B

30. A patient of your pharmacy recently had gastrectomy and ileum was removed. Which of the
following will be deficient?
A. Iron leading to Iron deficiency anemia.
B. Calcium due to deficiency of vitamin D.
C. Vitamin B 12 due to deficiency of intrinsic factor.
D. Vitamin K due to deficiency if GI bacteria.
Ans: C

31. Innervation of Vagal nerve causes?

Copyright © 2000-2020 TIPS Inc. Unauthorized reproduction of this manual is prohibited. This manual is being used during
review sessions conducted by PharmacyPrep.
2-7
Gastrointestinal System

A. Constipation
B. Diarrhea
C. Urinary retention
D. Urinary incontinence
Ans: B
Tips: vagal nerve act as parasympathetic (cholinergic) nerve cause diarrhea. Anticholinergic
drugs cause constipation.

32. Achlorhydric patient decreases absorption of?


A. Cyanocobalamin
B. Calcium carbonate
C. Iron supplements
D. Nutrient
E. Food
Ans: B
Tips: achlorhydric is a state of less acidic; calcium carbonate has higher absorption in
chlorhydric. Calcium citrate does not require acidic environment for absorption.

33. What prostaglandin is mainly present in gastrointestinal tract (GIT)?


A. PGE1
Arachidonic acid
B. PGE2

C. PGF2 alpha Prostaglandin (PG) PGE1 and PGE2
D. A and B protects GI mucosa.
Ans: A
Tips: PGE1 analogs misoprostol is for gastric mucosal protection.

34. What is incorrect about digestive enzymes?


A. Protease breaks down proteins.
B. Amylase breaks down carbohydrates.
C. Lipase breaks down fats.
D. Nuclease breaks down proteins.
Ans: D
Tips: Nucleic acid digestion is catalyzed by Nuclease and Nucleosidase.
Proteins digestion is catalyzed by Pepsin, Trypsin, and Peptidases.
Carbohydrate digestion is catalyzed by salivary amylase, pancreatic amylase, altase
Fat digestion is catalyzed by Lipase.

35. All of the following digestive enzymes are produced from the pancreas, EXCEPT:
A. Pancreatic amylase
B. Trypsin
C. Lipase
D. Gastrin
E. Nuclease

Copyright © 2000-2020 TIPS Inc. Unauthorized reproduction of this manual is prohibited. This manual is being used during
review sessions conducted by PharmacyPrep.
2-8
Gastrointestinal System

Ans: D
Tips: Gastrin is produced from stomach’s G-cell. Pancreatic enzymes include pancreatic amylase,
Trypsin, Lipase, Nuclease, and Nucleosidase.

36. Correct statements regarding the characteristics of hernias may include:


A. It is a protrusion of a loop of an organ or tissue normally through the GI area
B. Hernia hiatus is a herniation of stomach through the esophageal hiatus of diaphragm
C. Strangulated hernia is an incarcerated hernia that is so tightly constricted as to compromise
the blood supply of the contents of the hernia sac.
D. All are correct
Ans: D
Tips: Hernia is a protrusion of a loop of an organ or tissue normally through the GI area. Hernia
hiatus is a herniation of the stomach through the esophageal hiatus of the diaphragm.
Strangulated hernia is an incarcerated hernia that is so tightly constricted as to compromise the
blood supply of the contents of the hernia sac.

37. Which of the following influences the release of pancreatic juice and bile?
A. Cholecystokinin and secretin
B. Secretin and gastrin
C. Cholecystokinin and gastrin
D. Renin and cholecystokinin
Ans: A
Tips: Cholecystokinin stimulates pancreas and bile from the gallbladder.
Secretin is released into the blood stream and acts on the acinar cells of the pancreas to secrete
water and bicarbonate into the pancreatic ducts that drain into the duodenum.

38. Gastric emptying slowed by all the following, EXCEPT:


A. Fatty food
B. Vigorous exercise
C. Hot meal
D. Hunger
E. Emotional stress
Ans. C
Tips:
Factors promote stomach emptying Factors Inhibit gastric emptying
Gastric volume (increase food volume in Duodenal distention,
stomach promotes gastric emptying. High intensity, intermittent exercise can
slow gastric emptying
Clear liquids empty rapidly than solid food Fats and proteins breakdown in small
intestine
Protein empty fastest than carbohydrate Temperature cold liquids (4c) empty slowly
and then fats

39. Correct statement regarding pernicious anemia may include:

Copyright © 2000-2020 TIPS Inc. Unauthorized reproduction of this manual is prohibited. This manual is being used during
review sessions conducted by PharmacyPrep.
2-9
Gastrointestinal System

A. Due to dietary deficiency of vitamin B 12.


B. Prevented by oral administration of vitamin B 12.
C. Treated by parenteral administration of folic acid.
D. Treated by parenteral administration of vitamin B 12.
E. Caused by deficiency of iron
Ans: D
Tips: Pernicious anemia is characterized by deficiency of vitamin B12-cyanocobalamine and
therefore should be treated with intramuscular injection of vitamin B12 because this vitamin is
poorly absorbed orally.
Strategic thinking:
1. Read the question.
2. Choose elimination method in answering this type of question to increase your
chance of arriving at the correct answer. In light of this item, 3 options are relevant,
options A, B and D. But you can easily eliminate A and B on the basis that Pernicious
anemia is not a deficiency of Vitamin 12 but of the intrinsic factor that facilitates the
absorption of Vitamin B12. If questions asks for the result of pernicious anemia, the
correct answer is A. But in this case it is D as explained in the TIPS.
3. D is the option that makes the most sense.

40. Fecal occult blood test (FOBT) is recommended for screening of?
A. Peptic ulcer
B. Crohn’s disease
C. Ulcerative colitis
D. Colon cancer
E. Benign prostatic hyperplasia
Ans: D

41. C. difficle is an anaerobic bacteria present in colon. It is?


A. Obligate anaerobic
B. Obligate aerobic
C. Facultative anaerobic
D. Facultative aerobic
Ans: A
Tips: C. difficle is a gram positive, spore forming bacterium that is an obligate anaerobe and
potentially fatal GI pathogen. C. difficle is very sensitive to even low levels of oxygen.

42. A patient was diagnosed with endocarditis due to S. viridans. The most powerful source of
infection is?
A. Mouth
B. Lung
C. GI tract
D. Urinary tract
E. Liver
Ans: A

Copyright © 2000-2020 TIPS Inc. Unauthorized reproduction of this manual is prohibited. This manual is being used during
review sessions conducted by PharmacyPrep.
2-10
Nervous System

PHARMACY PREP
NERVOUS SYSTEM

1. Which part of the brain controls important cognitive skills in humans, such as emotional
expression, problem solving, memory, language
(mesolimbic pathways)?
A. Frontal lobe
B. Occipital lobe
C. Temporal lobe
D. Parietal lobe
Ans: A

2. Blood brain barrier is present at?


A. Endothelial cells lining capillaries in cerebrospinal tissues.
B. Epithelial cells lining cerebrospinal tissues or capillaries.
C. Epithelial cell lining cerebral artery capillaries.
D. In blood circulations capillaries.
Ans: A

3. What is the difference between nociceptive pain and nerve pain?


A. The nerve pain is slower or chronic.
B. Nociceptive pain does not involve myelin sheets.
C. Nerve pain affects the vertebral spinal column.
D. Nerve pain are neuralgia or neuropathic pain
E. All of the above
Ans: E

4. Only acts on sympathetic nervous system?


A. Heart
B. Eye
C. Gall bladder
D. Tongue
E. Gastrointestinal tract
Ans: A
Single innervation Target organs Dual innervation target organs
(sympathetic nervous system only) (sympathetic & parasympathetic)
Sweat glands Eye
Peripheral blood vessels Salivary gland
Hair follicles Lung
Brown adipose tissue Heart (including blood vessels)
Adrenal medulla Pancreas
Kidney Liver
Bladder
Reproductive organs
Gastrointestinal tract
Copyright © 2000-2020 TIPS Inc. Unauthorized reproduction of this manual is prohibited. This manual is being used during
review sessions conducted by Pharmacy Prep.
3-1
Nervous System

5. A man is unable to hold a typing paper between his index and middle finger. Which of the
following nerves was likely injured?*
A. Radial nerve
B. Median nerve
C. Ulnar nerve
D. Axillary nerve
E. Sciatic nerve
Ans: B
Tips: Median nerve compressed at the wrist in numbness or pain between the index and middle
finger.
Strategic thinking: in answering this type of question:
1. Read the question. Identify and organize information.
2. Look for keyword/s or key phrases. In this case, unable to hold a typing paper
between his index and middle finger. All of the nerves from the choices are present
in the arm. The question specifies the INDEX and MIDDLE FINGERS. The only nerve
that passes through the carpal tunnel is median nerve. All the other nerves, although
they supply the length of the arm, they are not specific.
3. The correct answer is B.

6. Dyskinesia is associated with?


A. Cerebrum
B. Cerebellum
C. Spinal cord
D. Thalamus
Ans: B

7. Which part of the brain controls respiration?


A. Corpus collosum
B. Cerebellum
C. Medulla oblongata
D. Thalamus
Ans: C
Tips: Medulla oblongata is the part of the brainstem that helps regulate autonomic function
(involuntary organ function) breathing, heart and blood vessel function, and digestion, sneezing,
and swallowing.

8. A patient has difficulty in chewing and is also experiencing problems during mastication. What
cranial nerve is responsible for mastication?
A. Vagus
B. Olfactory
C. Glossopharyngeal
D. Optic
E. Trigeminal
Ans: E
Copyright © 2000-2020 TIPS Inc. Unauthorized reproduction of this manual is prohibited. This manual is being used during
review sessions conducted by Pharmacy Prep.
3-2
Nervous System

Tips: Trigeminal nerve is cranial nerve or fifth cranial nerve is responsible for sensation in the
face and motor functions such as biting and chewing and it is largest of cranial nerve.

9. MP is 30-year-old man who presents at your pharmacy with high fever and muscle stiffness.
Pharmacist refers patient to the emergency. Patient is likely using which of the following
medications?
A. Bromocriptine Dopamine antagonist: Haloperidol is used
B. Tetracycline schizophrenia (psychosis). These drugs
C. Haloperidol increase body temperature and cause
D. Fluoxetine Neuroleptic malignant syndrome.
E. Phenelzine
Ans: C
Strategic thinking: in answering this type of question:
1. Read the question. Identify and organize the information.
2. Question mentions about the side effects from using a particular drug. Some
information requires purely memorization. You device technique to memorize side
effects from a class of drug (MNEMONICS). Use the Misbah’s EE Flashcards that may
help you do this. In light of this question, NMS is a side effect of antipsychotics.
“FEVER” is your mnemonic which stands for Fever, Encephalopathy, Vital sign
instability, Elevated WBC/CPK, Rigidity (stiffness)
3. Correct answer is C. it is the only antipsychotic from the given choices.

10. Which of the following is NOT a Parkinson’s disease symptom?


A. Tremors
B. Rigidity
C. Dyskinesia
D. Tardive dyskinesia
Ans: D
Tips: Tardive dyskinesia is associated with extra pyramidal symptoms caused by antipsychotic
drugs like haloperidol. However, Parkinson’s disease does not cause tardive dyskinesia.

11. Vagus nerve innervates?


A. Vision
B. Auditory function
C. Laryngeal
D. Defecation
Ans: D

12. Which of the following is NOT a sensory (somatic) nerve?


A. Ulnar
B. Radial
C. Esophageal nerve
D. Sciatic
Ans: C
Copyright © 2000-2020 TIPS Inc. Unauthorized reproduction of this manual is prohibited. This manual is being used during
review sessions conducted by Pharmacy Prep.
3-3
Nervous System

13. Which of the following statement is correct about sciatic nerve?


A. It passes through the neck down to the buttocks.
B. It passes through the posterior thigh and all of the leg and foot.
C. It passes through the elbow to wrist.
D. It passes through knee down to foot.
Ans: B
Tips: Sciatic nerve is the branch of sacral plexus. It divides into tibial and common fibular nerves,
which supplies the muscles of the posterior thigh and all of the leg and foot.

14. Damage of the axillary nerve is thought to cause?


A. Paralysis of the fifth finger
B. Impaired sensation in deltoid muscle of shoulder
C. Numbness of extremities
D. Inability to bend elbow
E. Loss of sensations in thumb
Ans: B
Tips: Axillary nerve supplies deltoid and teres minor muscles, shoulder joint, and skin on back of
the arm. Injury to axillary nerve can cause loss of sensation in lower part of deltoid muscles.

15. Damage to radial nerve can cause?


A. Inability to move neck
B. Inability to move shoulders
C. Inability to move wrist and fingers
D. Inability to move toes
E. Impaired sensation in deltoid muscle of shoulder
Ans: C
Tips: Radial nerve is a branch of brachial plexus. It supplies the triceps brachii, brachioradialis,
and muscle of the extensors compartment of forearm. Radial nerve palsy (wrist drop) is caused
by prolong compression of radial nerve, which runs along the underside of bone in the upper
arm. It is also called Saturday night palsy. It usually improves once the pressure is relieved.

16. A CNS disease where the myelin sheath of motor neurons is degenerating or being destroyed,
which interferes with neuronal impulses?
A. Parkinson's Disease
B. Multiple sclerosis
C. Alzheimer's dementia
D. Fibromyalgia
Ans: B

17. Lesions of substantia nigra are caused by destruction of dopaminergic neurons that occur in
patient with?
A. Alzheimer's disease
B. Migraine
Copyright © 2000-2020 TIPS Inc. Unauthorized reproduction of this manual is prohibited. This manual is being used during
review sessions conducted by Pharmacy Prep.
3-4
Nervous System

C. Meniere's disease
D. Parkinson's Disease
Ans: D

18. Involuntary functions are controlled by?


A. Cerebellum
B. Cerebrum
C. Cortex
D. Hypothalamus
Ans: B

19. Dopaminergic pathways are associated with?


A. Occipital lobe
B. Middle lobe
C. Frontal lobe
D. Temporal lobe
Ans: C
Tips: Dopaminergic pathways are associated with frontal lobes

20. Compression of ulnar nerve at the elbow will cause?


A. Numbness of small fingers
B. Wrist drops
C. Inability move elbow
D. Inability to move shoulder
Ans: A
Tips: Cubital tunnel syndrome (ulnar nerve compression in elbow). Ulnar nerve innervates small
fingers.

21. All of the following drugs have CNS side effects, EXCEPT:
A. Rifampin
B. Cefuroxime sodium
C. Diphenhydramine
D. Ipratropium
Ans: D

22. Which of the following medical condition is associated with behavioral problem?
A. Restless leg syndrome
B. Dementia
C. Schizophrenia
D. Alzheimer’s disease
Ans: C
Tips: Behavioral problems include major depression, mania, and schizophrenia.

23. The area of the brain associated with Alzheimer’s disease?


Copyright © 2000-2020 TIPS Inc. Unauthorized reproduction of this manual is prohibited. This manual is being used during
review sessions conducted by Pharmacy Prep.
3-5
Nervous System

A. Frontal lobe
B. Temporal lobe
C. Occipital lobe
D. Brain stem
Ans: A

24. What is nystagmus?


A. Eye adjustment problem
B. Rapid eye movement
C. Eye imbalance
D. Eye visual fixation problem
E. Vestibular problem
Ans: B

25. Which of the following drug has nystagmus as side effect due to CNS toxicity?
A. Carbamazepine
B. Valproic acid
C. Lamotrigine
D. Phenytoin
Ans: D
Tips: Phenytoin toxicity side effect is nystagmus.

26. Which of the following antibiotics can be used to treat meningitis?


A. Tetracycline
B. Rifampin
C. Vancomycin
D. Metronidazole
Ans: B

27. Radial nerve passes through what region?


A. Arm
B. Shoulder
C. Wrist
D. Forearm
E. Chest
Ans: D

28. Injury to axillary nerve results in loss of sensation over?


A. Arm
B. Shoulder
C. Wrist
D. Elbow
Ans: B

Copyright © 2000-2020 TIPS Inc. Unauthorized reproduction of this manual is prohibited. This manual is being used during
review sessions conducted by Pharmacy Prep.
3-6
Nervous System

29. Sensory receptors differ from pain receptors such that they:
A. Are less affected by morphine analgesic dose.
B. Unmyelinated
C. Response is more delayed.
D. Ends in ventral spine
Ans: A
Tips: Sensory Receptors are specialized neurons or nerve endings that respond to changes in the
environment by converting energy from a specific stimulus into an action potential (a process
known as transduction).
Pain receptors are any one of the many free nerve endings throughout the body that warn of
potentially harmful changes in the environment, such as excessive pressure or temperature. The
free nerve endings constituting most of the pain receptors are located chiefly in the epidermis
and in the epithelial covering of certain mucous membranes. They also appear in the stratified
squamous epithelium of the cornea, in the root sheaths and the papillae of the hairs, and
around the bodies of sudoriferous glands. The terminal ends of pain receptors consist of
unmyelinated nerve fibers that often anastomose into small knobs between the epithelial cells.
Any kind of stimulus, if it is intense enough, can stimulate the pain receptors in the skin and the
mucosa, but only radical changes in pressure and certain chemicals can stimulate the pain
receptors in the viscera.

30. MP is a 30-year-old patient who presented at your pharmacy for refill of Risperidone.
Pharmacist counselled him in the counseling room. The pharmacist noticed that the patient got
up several times and sat, and at one point, he went out of the counseling room but returned a
few moments later. Patient may be experiencing, which of the following condition?
A. Parkinson’s symptoms
B. Alzheimer symptoms
C. Tardive dyskinesia
D. Akathisia
Ans: D
Tips: Akathisia = restlessness or cannot sit still.

31. All are involved in the knee jerk reflex, EXCEPT:


A. Sensory nerves
B. Motor neurons
C. Quadriceps
D. Calcaneus bone
E. Patella bone
Ans: D
Tips: foot of the tarsus is a bone of the heel bone.
The calcaneus, also called the heel bone, is a large bone that forms the foundation of the rear
part of the foot. The calcaneus connects with the talus and cuboid bones. The connection
between the talus and calcaneus forms the subtalar joint.

32. Cerebrospinal fluid (CSF) circulates through all of the following, EXCEPT:
Copyright © 2000-2020 TIPS Inc. Unauthorized reproduction of this manual is prohibited. This manual is being used during
review sessions conducted by Pharmacy Prep.
3-7
Nervous System

A. Corpus callosum
B. Lateral ventricles in brain
C. 4th ventricles
D. Cerebral aqueduct
E. Central Canal
Ans: A

33. Which of the nerve plexuses serve the shoulder and arm?
A. Sacral
B. Cervical
C. Phrenic
D. Brachial
Ans: D

34. What innervates sciatic nerve?


A. Innervates whole foot
B. Innervates hips
C. Innervates large muscle of buttock
D. Innervate quadricep
Ans: C

35. Destruction of radial nerve causes:


A. Difficulty straightening elbow
B. Difficulty moving shoulders
C. Effecting knee function
D. Effecting index fingers
E. Effect on wrist function
Ans: E

36. Cell for regeneration of myelin sheath?


A. Schwan cell
B. Myalgia
C. Dendritic cells
D. Neurons
E. Nephrons
Ans: A
Tips: Demyelination is observed in multiple sclerosis. Schwan cells (oligodendrocytes) are main
glial cells of the peripheral nervous system which wrap around axons of motor or sensory
neurons to form myelin sheath.
Dendritic cells (accessory cells) are antigen presenting cells of immune system. Their main
function is to process antigen material and present it on the cell surface to the Tcell.

37. In blood brain barrier, the monoamine oxidase enzyme carries out:
A. Oxidative deamination metabolic reactions
Copyright © 2000-2020 TIPS Inc. Unauthorized reproduction of this manual is prohibited. This manual is being used during
review sessions conducted by Pharmacy Prep.
3-8
Nervous System

B. Cytochrome oxidation
C. Catalysis of proteins
D. Oxidation of lipids
E. Oxidation of carbohydrates
Ans: A
Tips: MAO enzyme catalyzes oxidative deamination metabolic reaction.

38. Lewy bodies are proteins in nerve present in?


A. Parkinsonism
B. Psychosis
C. Depression
D. Epilepsy
E. Multiple sclerosis
Ans: A

39. Nystagmus involves all, EXCEPT:


A. Involuntary movement
B. Oscillation
C. Rapid eye movement
D. Repeating
E. Eye burger
Ans: E

40. Nerve plexus that serves the shoulder and arm?


A. Cervical
B. Lumbar
C. Brachial
D. Sacral
E. Cranial
Ans: C

41. Lumbar plexus serves the thigh?


A. Cervical
B. Lumbar
C. Brachial
D. Sacral
E. Femoral
Ans: E
Nerve plexus Peripheral nerve
Cervical plexus: Phrenic nerve
Brachial plexus: Axillary, median, radial and ulnar nerve
Lumbar plexus: Obturator, femoral, saphenous nerve
Sacral Plexus: sciatic nerve, gluteal region
Copyright © 2000-2020 TIPS Inc. Unauthorized reproduction of this manual is prohibited. This manual is being used during
review sessions conducted by Pharmacy Prep.
3-9
Nervous System

Copyright © 2000-2020 TIPS Inc. Unauthorized reproduction of this manual is prohibited. This manual is being used during
review sessions conducted by Pharmacy Prep.
3-10
Cardiovascular System

PHARMACY PREP
CARDIOVASCULAR SYSTEM

1. Absolute refractory period (ARP) reflects the time during which no action potential can be
initiated regardless of how much inward current are supplies. The ARP begins and ends at.
A. Phase I to Phase 4
B. Phase 0 to phase 1
C. Phase 1 to phase 2
D. Phase 1 to phase 3
E. Phase 2 and 3 only
Ans: D

2. Pathway of blood flow from systemic circulation to liver?


A. Mesenteric vein and portal vein
B. Portal artery and hepatic vein
C. Portal vein and hepatic artery
D. Hepatic vein and portal artery
Ans: A

3. What is incorrect about lymphatic system?*


A. Has No duct vessels or lymphatic valves
B. Connect between arteries and veins
C. Plasma proteins are absent
D. Consist of White blood cells
E. Platelets and red blood cells are absent
Ans: A
Tips: Lymphatic structures are lymphatic duct, lymphatic vessels, thymus, spleen, lymph
nodes, and lymphatic capillaries.

4. Change in cardiac automaticity (spontaneous activity initiated by cardiac cells generating


periodic oscillations) is most directly associated with change in which slope of myocardial
action potential?
A. Phase 0
B. Phase I
C. Phase 2
D. Phase 3
E. Phase 4
Ans: D

5. Which of the vasoactive hormone causes arterial constriction?


A. Antihistamines
B. Bradykinin
C. Prostaglandin
D. Thromboxane A 2
Copyright © 2000-2020 TIPS Inc. Unauthorized reproduction of this manual is prohibited. This manual is being
used during review sessions conducted by PharmacyPrep.
4-1
Cardiovascular System

E. Prostacyclin
Ans: D

6. Myocardial contractility is best correlated with the intracellular concentration of


A. Free Ca2+
B. Free Na+
C. Free K+
D. Free Cl-
E. Free Mg2+
Ans: A
Tips: Sympathetic stimulations increase calcium intake thus increase concentration of
calcium around myofibrils.  Ca will increase contractility by activating Actin and Myocin
muscle fibres.

7. Aldosterone is secreted by adrenal cortex. What is incorrect about aldosterone?


A. Increases sodium chloride (NaCl) reabsorption by the renal distal tubules thereby
increase blood volume and arterial pressure
B. It causes vasoconstriction of the arterioles, thus increase TPR and mean arterial
pressure.
C. Angiotensin II stimulates the synthesis and secretion of aldosterone by the adrenal
cortex
D. K+ excretion from renal distal tubule and water retention along with Na+
E. Acts as long-term regulator or blood pressure
Ans: B

8. Repolarization process produces?


A. Negative charge in cell
B. Sodium ion influx
C. Chloride efflux
D. Excessive positive charge inside cell
E. Excessive negative and positive charge inside cells
Ans: A
Tips: in repolarization state cell goes negative charge.

9. What disease is NOT associated with thrombus in circulatory system?


A. Acute coronary syndrome
B. Acute stroke
C. Stable angina
D. Ischemic heart diseases
E. Raynaud phenomenon
Ans: E

10. Embolus is?


A. Floating blood clot
Copyright © 2000-2020 TIPS Inc. Unauthorized reproduction of this manual is prohibited. This manual is being
used during review sessions conducted by PharmacyPrep.
4-2
Cardiovascular System

B. A blood clot at site of plaque


C. Fibrin clot
D. Bulging of arteries
E. Hemorrhage
Ans: A

11. The electrical activity occurred during depolarization and repolarization transmitted
through electrodes attached to the body and transformed by an electrocardiograph (ECG)
in to series of waveforms. The "QRS" wave indicates?
A. Atrial depolarization
B. Ventricular depolarization
C. Atrial repolarization
D. Ventricular repolarization
E. Plateau
Ans: B
Tips: QRS is activation ventricles, P wave indicate activation atrium and T wave is recover
wave.

12. Q-T prolongation (prolong interval) can cause?


A. Atrial fibrillation
B. Supraventricular fibrillation
C. Ventricular fibrillation or ventricular tachycardia
D. Arrhythmias
E. None of the above
Ans: C

13. Which of the following conditions risk factor is NOT related to first degree relative (family
history)?
A. Premature cardiovascular disease
B. Type 2 Diabetes
C. Coronary artery diseases
D. Dyslipidemia
E. Hepatitis
Ans: E

14. Which of the following is the indicator of ventricular activation?


A. P wave
B. Q wave
C. Q-T wave
D. QRS wave
E. T wave
Ans: D

Copyright © 2000-2020 TIPS Inc. Unauthorized reproduction of this manual is prohibited. This manual is being
used during review sessions conducted by PharmacyPrep.
4-3
Cardiovascular System

15. Which of the following medical conditions patient is recommended to self care by using of
tight stockings?
A. Coronary Artery Disease
B. Heart failure
C. Stroke
D. Deep vein thrombosis
E. All of the above
Ans: D

16. Atrial fibrillations ECG or changes in electrode potential curve show absence or change of?
A. P wave
B. QRS wave Atrial fibrillation is the major risk factor for stroke.
C. T wave (blood clot or thrombus in brain or cerebral artery)
D. QT wave
E. U wave
Ans: A
Tips: Atrial fibrillation is regularly irregular atrial rate that can change P wave. Atrial
fibrillation is the major risk factor for stroke. Frequent P waves appears in atrial fibrillation.

17. Which one of the following ECG waves is associated with ventricular depolarization?
A. P wave
B. QRS wave
C. T wave
D. QT interval prolongation
E. U wave
Ans: B
Tips: There are three waves. QRS is activation ventricle depolarization. P wave indicate
activation atrium or atrial depolarization. T wave is recovering wave.

18. Which of the following cardiovascular disease is least commonly associated with
atherosclerotic plaques?
A. Angina
B. Myocardial infarction
C. Congestive heart failure
D. Ischemic stroke
E. Raynaud's phenomenon
Ans: E
Tips: Raynaud’s phenomenon is due to vascular constriction in limbs.
Atherosclerosis leads to formation of plaques and facilitate blood clot formation. The most
common conditions associated with atherosclerosis are angina, myocardial infarction,
deep vein thrombosis and stroke. However Raynaud’s phenomenon is a peripheral vascular
disease occurs due to vascular constriction in limbs.
If stable plaque than commonly cause angina, if plaque is ruptured than cause
inflammation can lead to MI.
Copyright © 2000-2020 TIPS Inc. Unauthorized reproduction of this manual is prohibited. This manual is being
used during review sessions conducted by PharmacyPrep.
4-4
Cardiovascular System

19. What is the most common cause of arteriosclerosis?


A. High LDL
B. Smoking
C. Hypertension
D. Emotional stress
E. All of the above
Ans: A

20. A patient is diagnosed with atrial fibrillation. What is correct?


A. P wave is absent
B. P and QRS wave is absent
C. QRS wave is affected
D. T wave is affected
E. QT prolongation
Ans: A

21. What is correct about echocardiogram?


A. Measures blood pressure
B. Measure heart rate
C. Measure heart valve abnormality and blood clot location in coronary arteries.
D. Measure dysrhythmias
E. Measures electrode potential
Ans: C

22. What is the normal blood pressure?


A. 120/80
B. 120/85
C. 120/90
D. 120/95
E. 120/100
Ans: A

23. Which acute coronary syndrome affects only subendocardial part of the myocardium and
causes ST segment depression?
A. ST segment Elevated Myocardial Infarction (STEMI)
B. Non-ST segment Elevated Myocardial Infarction
(NSTEMI)
C. Prinz mental angina
D. All of the above
Ans: B
Tips: Acute coronary syndrome

Copyright © 2000-2020 TIPS Inc. Unauthorized reproduction of this manual is prohibited. This manual is being
used during review sessions conducted by PharmacyPrep.
4-5
Cardiovascular System

24. Who should not use compression stockings?


A. To prevent varicose veins
B. Elderly with compromised venous return
C. Person with risk factors for venous thromboembolism
D. Pregnant lady with pedal edema
E. Some one immobile in hospital after knee and hip replacement surgeries.
Ans: B

25. After depolarization, cell responds in the following manners, EXCEPT:


A. Calcium channel opens
B. Sodium Channel closure
C. Chloride channel opens
D. Potassium channel opens
Ans: A

26. Abnormal persistent dilatation of blood vessels is known as:


A. Embolism
B. Aneurism
C. Emphysema
D. Starvation
E. Atherosclerosis
Ans: B
Tips: Aneurysm is an abnormal dilatation of a blood vessel while embolism is an
obstruction of airway by a mass transported in the circulation. Emphesema is a reversible
form of airway obstruction.

27. Correct statements regarding embolism include:


I. Abnormal dilatation of blood vessel
II. Obstruction of a blood vessel by a mass transported in the circulation
III. An embolus can be a clot or a foreign material transported in the circulation
A. I only
B. III only
C. I and II only
D. II and III only
E. All are correct
Copyright © 2000-2020 TIPS Inc. Unauthorized reproduction of this manual is prohibited. This manual is being
used during review sessions conducted by PharmacyPrep.
4-6
Cardiovascular System

Ans: D
Tips: Embolism is an obstruction of a blood vessel by a mass transported in the circulation.
The embolus is this mass transported in the circulation that can be a clot or a foreign
material. Abnormal dilatation of a vessel artery is known as aneurysm

28. During inspiration, the lowest blood pressure in supine position is:
A. Vena cava
B. Internal carotid artery
C. Aorta artery
D. Sciatic artery
E. Pulmonary artery
Ans: A
Tips: During inspiration the lowest blood pressure in supine position is in vena cava

29. Which of the following theories relates to the tension, radius of vessel and intraluminal
pressure achieved in a blood vessel wall?
A. Fick’s law
B. Arrhenius
C. LaPlace’s law
D. Newtonian law
E. Henderson law
Ans: C
Tips: LaPlace’s Law explains the tension achieved in a blood vessel wall to the radius of
vessel and intraluminal pressure.

30. The first location where blood is present in case of left ventricular heart failure:
A. Liver
B. Lung Left ventricular failure = pulmonary edema (lung)
C. Heart Right ventricular failure = peripheral edema
D. Kidney
E. Brain
Ans: B
Tips: Blood leaves the lung and reaches the left ventricle of the heart through pulmonary
vein. In case of left ventricular failure the tendency is accumulation of blood in the lung due
to blockage of blood circulation from lung through the heart.

31. Stenosis:
A. Dilatations of blood vessels
B. Increase in cardiac output
C. Accumulation of cholesterol in blood vessels
D. Narrowing of blood vessels
E. High pressure in blood vessels
Ans: D
Tips: Stenosis is the narrowing of blood vessels
Copyright © 2000-2020 TIPS Inc. Unauthorized reproduction of this manual is prohibited. This manual is being
used during review sessions conducted by PharmacyPrep.
4-7
Cardiovascular System

32. The first tissue that receive the greatest cardiac output at rest?
A. Lung
B. Liver
C. Kidney
D. Muscle
E. Fat
Ans: B
Tips: The first tissue that receives the greatest cardiac output at rest is liver, followed by
kidney and lung.

33. Correct statements regarding blood pressure may include:


I. Systolic pressure is the highest arterial pressure during a cardiac cycle.
II. Diastolic pressure is the lowest arterial pressure during a cardiac cycle
III. Diastolic pressure is crucial in verifying if a person suffers of high blood pressure
A. I only
B. III only
C. I and II only
D. II and III only
E. All are correct
Ans: E
Tips: The normal blood pressure in a health adult is 120/80, where 80 is the diastolic and
120 is the systolic pressure. Systolic pressure is the highest arterial pressure during a
cardiac cycle and is measured when the heart contracts while diastolic pressure is the
lowest arterial pressure during a cardiac cycle and is measured when the heart is relaxed.

34. Phase 0 is?


A. Repolarization
B. Depolarization
C. Prolong repolarization
D. Plateau
E. None of the above
Ans: B

Copyright © 2000-2020 TIPS Inc. Unauthorized reproduction of this manual is prohibited. This manual is being
used during review sessions conducted by PharmacyPrep.
4-8
Endocrine System

PHARMACY PREP
ENDOCRINE SYSTEM

1. Which hormone keeps corpus luteum after fertilization and pregnancy?


A. Human chorionic gonadotropin hormone (HCG)
B. Progestin ↑HCG = 1st three months of pregnancy
C. Estrogen ↑ LH = day 14 of menstrual cycle (ovulation)
D. Luteinizing hormone (LH) ↑ Estrogen = follicular phase
E. FSH ↑ Progestin = luteal phase
Ans: A
Tips: If fertilization occurs, the corpus luteum is rescued by regression of hCG, which is
produced by the placenta. On the first trimester of pregnancy, the corpus luteum is
stimulated by hCG for the production of estrogen & progestin.

2. The doctor is suspecting that his patient is suffering from hypothyroidism. Which assay
can be used to confirm his diagnosis?
A. Serum TSH
B. FT 4 in urine
C. T 4 increase
D. TSH in urine
Ans: A
Tips: Serum TSH test has a greater sensitivity in the detection of thyroid disease than any
other test especially if hypothyroidism is suspected.

3. Follicle Stimulating Hormone and Luteinizing Hormones levels may be mainly increased
during:
A. Pregnancy
B. Ovulation
C. Conception
D. Last trimester of pregnancy
Ans: B
Tips: FSH and LH are predominantly increased during ovulation.
Follicular phase Ovulation phase Luteal phase
Estrogen LH and FSH Progestin
Day 1 to 13 Day 14 Day 15 to 28
Menstrual bleeding (1 to 6)

4. Which is/are true about placenta?


A. The umbilical cord transport nutrients from mother to fetus passing through the
placenta.
B. The umbilical cord is formed of two umbilical arteries and one umbilical vein
C. Placenta is an organ characteristic of true mammals during pregnancy
D. All are correct

Copyright © 2000-2020 TIPS Inc. Unauthorized reproduction of this manual is prohibited. This
manual is being used during review sessions conducted by Pharmacy Prep.
5-1
Endocrine System

Ans: D
Tips: Placenta is an organ characteristic of true mammals during pregnancy, joining
mother and offspring, providing endocrine secretion and selective exchange of soluble,
but not particulate, blood-borne substances through an apposition of uterine and
thromphoblastic vascularized parts.

5. Hypothyroidism may result due to:


A. Treatment of hyperglycemia
B. Treatment overdose of hyperuricemia
C. Treatment overdose with anti-thyroid drugs.
D. Treatment of levothyroxine.
Ans: C
Tips: Overdose or excessive use of antithyroid drugs such as Methimazole or
Propylthiouracil can cause hypothyroidism. Grave’s disease is characterized by
hyperthyroidism. Hashimoto thyroiditis is a hypothyroidism.
Strategic thinking:
1. Read the question.
2. In this particular type of question, correct analysis can be achieved by
thinking of a “balance”. Here, HYPOTHYROIDISM has become a problem as a
result of overtreating its opposite condition, HYPERTHYROIDISM (an elevated
level of the thyroid hormones). Anti-thyroid drugs are used in an attempt to
bring the T4 and T3levels to normal, to achieve balance. This is why regular
monitoring of the thyroid hormones should be done for the purpose of dose
adjustment. In essence, the opposite condition is expected to occur in the
event of overtreating (use of medication overdose) the other condition.
3. Correct answer is C as other options A and B have nothing to do with the
thyroid hormones. Option D on the other hand will cause HYPERthyroidism if
a patient overdosed.

6. If patient is given high dose of Levothyroxine, he may experience?


A. Severe hypothyroidism
B. Hyperthyroidism
C. Hashimoto’s disease
D. Myxedema
Ans: B

7. Characteristics of hypoparathyroidism may include:


A. It is characterized by decrease of calcium and increase of Phosphorous.
B. It is a deficiency of thyroid gland.
C. It is characterized by an increase of calcium and decrease of phosphorous.
D. Decrease in Levothyroxine.
Ans: A

Copyright © 2000-2020 TIPS Inc. Unauthorized reproduction of this manual is prohibited. This
manual is being used during review sessions conducted by Pharmacy Prep.
5-2
Endocrine System

Tips: Hypoparathyroidism is a condition produced by greatly reduced function of the


parathyroids possibly due to an autoimmune disease or genetic factors, or by removal of
those bodies. It leads to fall in calcium levels followed by a rise in plasma phosphate
level.

8. Hyperaldosteronism, a hyper function of adrenal cortex may be characterized by:


A. Abnormality of electrolyte metabolism resulting in high secretion of aldosterone.
B. Can be treated by aldosterone antagonist: Spironolactone.
C. Hyperaldosteronism is characterized typically by hypokalemia.
D. All are correct
Ans: D
Tips: Hyperaldosteronism is an abnormality of electrolyte metabolism caused by
excessive secretion of aldosterone. The over secretion of aldosterone by adrenal cortex is
mainly characterized by hypokalemia.

9. Which is the main hormone secreted by the adrenal cortex and responsible for the
regulation of electrolytes and water balance?
A. FSH
B. Thiazides
C. ADH
D. Aldosterone
Ans: D
Tips: Aldosterone is the main mineral corticoid hormone secreted by the adrenal cortex.
Its principal activity is the regulation of electrolyte and water balance by promoting the
renal retention of sodium and the excretion of potassium.

10. Ketoacidosis is a type of acidosis accompanied by the accumulation of ketone bodies in


the body tissues and fluids and may be a result of:
A. Starvation
B. Juvenile type diabetes
C. Inadequate insulin treatment
D. All are correct
Ans: D
Tips: Ketoacidosis is a type of acidosis accompanied by the accumulation of ketone
bodies (ketosis) in the body tissues and fluids. It can be caused by juvenile diabetes,
diabetes acidosis, starvation and inadequate treatment with insulin.

11. Hyperparathyroidism causes excessive calcium in blood. This triggers secretion of which
hormone?
A. Calcitonin
B. Parathyroid hormone
C. Thyroid hormone
D. Insulin

Copyright © 2000-2020 TIPS Inc. Unauthorized reproduction of this manual is prohibited. This
manual is being used during review sessions conducted by Pharmacy Prep.
5-3
Endocrine System

Ans: A
Tips: Hypercalcemia triggers calcitonin secretion from thyroid gland.
Strategic thinking:
1. Read the question.
2. In answering this type of question, determine the relationship of electrolytes
and hormones in the body, what work against each other or together. It is
important to remember that the body is designed to keep things in balance in
order to sustain life. Rise in level of a hormone or electrolyte causes the body
to release a hormone or electrolyte that works against it to bring it to normal.
(As discussed in tips)
3. Correct answer is A.

12. Hypofunction of adrenal cortex may cause:


A. Cushing disease
HYPOCORTICOSTEROIDS HYPERCORTICOSTEROIDS
B. Gray’s disease
Addison disease Cushing syndrome
C. Addison’s disease
D. All are wrong
Ans: C
Tips: Hypofunction of adrenal cortex is well known as Addison’s disease while the
hyperfunction of adrenal cortex is known as Cushing’s syndrome.

13. Cushing’s syndrome is a disorder of adrenal cortex that is mainly due to:
A. Hyperfunction of the adrenal medulla
B. High secretion of cortisone.
C. Hypofunction of adrenal cortex.
D. All of the above
Ans: B
Tips: Cushing syndrome is characterized by an increase in cortisone levels resulting from
a hyperfunction of adrenal cortex.

14. Aldosterone is released from?*


A. Adrenal medulla
B. Adrenal cortex outer layer
C. Adrenal cortex inner layer
D. Adrenal cortex middle layer
E. Pancreas
Ans: B
Adrenal gland Controlled by Hormones released
Adrenal medulla Sympathetic(chromaffin cells) Epinephrine
Cortex: Outer layer Renin-angiotensin Aldosterone
Middle layer ACTH Corticosteroids
Inner layer ACTH Androgens

Copyright © 2000-2020 TIPS Inc. Unauthorized reproduction of this manual is prohibited. This
manual is being used during review sessions conducted by Pharmacy Prep.
5-4
Endocrine System

15. All of the following cells are normally presented in the pancreas, EXCEPT:
A. β-cell
B. α-cell
C. Sertoli cell
D. Delta cell
Ans: C
Tips: Sertoli cells are found in the tubules of the testes to which the spermatids become
attached. They provide support, protection and nutrition until the spermatids become
transformed into mature spermatozoa.

16. Which of the following statement is incorrect about vasopressin (antidiuretic hormone
ADH)?
A. It is a potent vasoconstrictor that increases Total Peripheral Resistance (TPR) in
arterioles.
B. Increases water reabsorption by the renal distal tubule and collecting ducts.
C. ADH is released when atrial receptors detect the decrease in blood volume.
D. Antidiuretic hormone deficiency cause type 2 Diabetes Mellitus
E. Excessive Antidiuretic hormone cause Syndrome of Inappropriate Antidiuretic
hormone (SIDAH)
Ans: D
Tips: Antidiuretic hormone deficiency cause diabetes insipidus.

17. Which is not secreted from the anterior pituitary?


A. Prolactin
B. Vasopressin
C. Gonadotropin hormones
D. LH
Ans: B
Tips: Vasopressin is an anti-diuretic hormone secreted from the posterior pituitary.

18. All of the following substance may be classified as steroid hormone, EXCEPT:
A. Vitamin D 3
B. Estrogen
C. Cortisone
D. Stilbestrol
E. Hydrocortisone
Ans: D
Tips: Stilbestrol is a synthetic estrogen. It does not have steroidal hormone structure. It is
used mainly used in the treatment of breast cancer.
Strategic thinking:
1. Read the question. Take note of the clues, in this case EXCEPT.

Copyright © 2000-2020 TIPS Inc. Unauthorized reproduction of this manual is prohibited. This
manual is being used during review sessions conducted by Pharmacy Prep.
5-5
Endocrine System

2. In answering this type of question, it is helpful to know from the choices the
ones that share the same structure. Although a vitamin, Vitamin D is derived
from a steroid so it has the structure similar to options B, C, E.
3. Correct answer is D as explained in TIPS.

19. Medical condition related to hypothyroidism, the decrease of thyroid hormones might
include:
A. Cushing syndrome
B. Addison’s syndrome
C. Grave’s disease
D. Hashimoto disease
Ans: D
Tips: Hypothyroidism is a hypofunction of thyroid gland that is responsible for the
development of Myxedema crisis or Gull’s disease.

20. Examples of rate limiting factors of insulin may include the following, EXCEPT:
A. Capillary passage
B. Zinc crystalline insulin
C. Protein binding
D. A and B
Ans: C
Tips: Insulin does not bind to protein.

21. Wrong statement regarding FSH and LH may include:


A. They are stimulating hormones.
B. They are secreted from anterior pituitary gland
C. Secreted specifically from posterior pituitary gland.
D. They increase their concentration during ovulation.
E. Luteinizing hormone is used to detect ovulation in test
Ans: C
Tips: FSH-Follicle Stimulating Hormone and LH-Luteinizing Hormone are pituitary
gonadotropin hormones secreted from the anterior pituitary.

22. After an insulin dose, hypoglycemia may occur due to:


A. Low carbohydrate diet
B. Increase in glucose levels
C. Increase in glycogen levels
D. High carbohydrate diet
Ans: A
Tips: After an insulin dose, hypoglycemia may occur due to low carbohydrate diet.

23. Insulin is a hormone secreted by which cells?


A. α-cells

Copyright © 2000-2020 TIPS Inc. Unauthorized reproduction of this manual is prohibited. This
manual is being used during review sessions conducted by Pharmacy Prep.
5-6
Endocrine System

B. β-cells
C. Glucagon
D. Amylase
Ans: B
Tips: Insulin is a protein hormone secreted by the beta cells of the pancreas. It is secreted
in response to elevated blood levels of glucose, amino acids, ketones, fat acids and
promotes the efficient storage and utilization of these molecules.

24. Aldosterone is secreted by the outer layer of adrenal cortex. Which of the following is
incorrect?
A. It increases NaCl reabsorption by the renal distal tubules thereby, increases blood
volume and arterial pressure.
B. It causes vasoconstriction of arterioles thereby, increases TPR and mean arterial
pressure
C. Angiotensin II stimulates the synthesis of aldosterone at adrenal cortex.
D. All of the above
E. None of the above
Ans: B
Tips: Aldosterone is NOT a vasoconstrictor. Instead Angiotensin II is strong
vasoconstrictor.
Aldosterone hormone main functions are Na, water retention in distal tubules and
Potassium excretion in late distal tubules and collecting duct

25. Correct statements regarding glucagon a polypeptide hormone, may include:


A. Secreted by the α-cells of the pancreas.
B. It increases blood glucose levels or hyperglycemia
C. Stimulates hepatic gluconeogenesis and glycogenolysis.
D. It is used in emergencies of severe hypoglycemia
E. All are correct
Ans: E
Tips: Glucagon is a polypeptide hormone secreted by the alpha cells of the pancreas in
response to hypoglycemia or due to stimulation of the growth hormone of the anterior
pituitary; it stimulates glycogenolysis in the liver by inducing activation of liver
phosphorylase.

26. Examples of therapeutically used hormones and related drugs may include:
A. Adrenocorticosteroids
B. Antidiabetic agents
C. Thyroid hormones
D. Gonadal and pituitaries hormones
E. All are correct
Ans: E

Copyright © 2000-2020 TIPS Inc. Unauthorized reproduction of this manual is prohibited. This
manual is being used during review sessions conducted by Pharmacy Prep.
5-7
Endocrine System

Tips: Adrenocorticosteroids (Hydrocortisone, Prednisone, Betamethasone,


Dexamethasone).
Antidiabetic agents (Insulin), thyroid hormones (Levothyroxine, Gonadal and pituitary
hormones (GnRH analogs: Goserilin) are all therapeutically used hormones and related
drugs.

27. Examples of gonadotropin hormone may include:


A. Adrenocorticotropic hormone (ACHT)
B. Follicle Stimulating Hormone (FSH)
C. Thyroid Stimulating Hormone (TSH)
D. Thyrotropin-releasing hormone (TRH)
Ans: B
Tips: FSH-Follicle Stimulating Hormone is a gonadotropin hormone excreted from the
anterior pituitary. Gonadotropins are glycoprotein polypeptide hormone secreted from
anterior pituitary gland.

28. False statement/s regarding VASOPRESSIN include/s:


A. It is used to treat neurogenic diabetes insipidus.
B. It is used to treat postoperative abdomen tissue.
C. It acts on proximal renal epithelium promoting the absorption of water.
D. It acts on distal convoluted renal epithelium promoting the absorption of water.
Ans: C
Tips: Vasopressin does not act on proximal but on the distal renal epithelium promoting
the absorption of water.

29. Hormone which sustains the corpus luteum in pregnancy is:


A. Estrogen
B. Follicle Stimulating Hormone (FSH)
C. Gonadotropin hormones like hCG
D. Progesterone
Ans: C

30. Characteristics of diabetes insipidus may include:


A. Polyuria
B. Thirst
C. Decrease in antidiuretic hormones levels
D. Decrease in osmolarity
E. All are correct
Ans: E
Tips: Diabetes insipidus results in a deficient quantity of antidiuretic hormones being
released or produced, and thus failure of tubular reabsorption of water in the kidney. As
a result, a large amount of urine of low specific gravity is excreted, followed by
dehydration and great thirst.

Copyright © 2000-2020 TIPS Inc. Unauthorized reproduction of this manual is prohibited. This
manual is being used during review sessions conducted by Pharmacy Prep.
5-8
Endocrine System

31. Hormone used to improve uterine contraction in labor and to control postpartum
bleeding is:
A. Vasopressin
B. Oxytocin
C. Somatropin
D. Prolactin
Ans: B
Tips: Oxytocin is therapeutically used to improve uterine contraction in labour and to
control postpartum bleeding. It also acts in milk ejection stimulation.

32. Adrenocorticosteroid hormones are naturally present in the body and are synthesized
by:
A. Hypothalamus
B. Pituitary
C. Adrenal cortex
D. Adrenal gland
Ans: C
Tips: Adrenal cortex is the one responsible for the release of cortisone-
adrenocorticosteroids.

33. Which of the following hormone may stimulate the release of TSH (Thyroid Stimulating
Hormones)?
A. Antihiuretic Hormone (ADH)
B. Parathyroid hormone
C. Thyrotropin-releasing hormone (TRH)
D. Luteinizing hormone (LH)
Ans: C
Tips: TRH is the Thyroid Regulating/Releasing Hormone, secreted from the
hypothalamus and responsible for the stimulation of TSH’s release from pituitary gland.

34. What does DEIODINATION mean?


A. The soma of tyrosyl residues with thyroglobulin proteolysis.
B. The catalyzation of T 4 to T 3 formation.
C. The production of iodine by the thyroid gland.
D. The degradation of thyroid hormones that are eliminated by feces and urine.
Ans: B
Tips: Deiodination is activation of thyroxine T 4 to active triiodothyronine T 3 in peripheral
and liver tissues. Deiodination is the degradation of thyroid hormones that are
eliminated by feces and urine.

35. What regulatory hormones are produced during periods of hypoglycemia?


A. Insulin and epinephrine
B. Glucagon and epinephrine

Copyright © 2000-2020 TIPS Inc. Unauthorized reproduction of this manual is prohibited. This
manual is being used during review sessions conducted by Pharmacy Prep.
5-9
Endocrine System

C. Aldosterone and epinephrine


D. Corticosteroids and androgen
Ans: B

36. Hyperthyroidism is characterized by high levels of thyroid hormones. Example(s) of


drugs used in hyperthyroidism treatment may include:
A. Propanolol
B. Methimazole
C. Propylthiouracil
D. All are correct
Ans: D
Tips: Propylthiouracil and Methimazole are main drugs used in the treatment of
hyperthyroidism. Propanolol is also used in the short-term preoperative management of
thyrotoxicosis crises.

37. Characteristic symptoms of hypothyroidism may include all of the following, EXCEPT:
A. Sensitivity to cold
B. Heat intolerance
C. Lethargy
D. Constipation
E. Weight gain
Ans: B
Tips: Heat intolerance or excessive sweating is one of the major side effects of
hyperthyroidism.

38. Which of the following stimulates secretion of calcitonin hormone from Thyroid gland?
A. Hypocalcemia
B. Hypercalcemia
C. Hypokalemia
D. Hyperkalemia
Ans: B

39. How long does it take to begin menstrual bleeding after ovulation?
A. 10 to 11 days
B. 11 to 12 days
C. 13 to 14 days
D. 15 to 16 days
Ans: C

40. Which of the following moves calcium from bone to blood?


A. Calcitonin
B. Bisphosphonates
C. Calcitriol

Copyright © 2000-2020 TIPS Inc. Unauthorized reproduction of this manual is prohibited. This
manual is being used during review sessions conducted by Pharmacy Prep.
5-10
Endocrine System

D. All
Ans: C
Tips: Calcitriol is vitamin D3, dissolved calcium from bone to blood in hypocalcemia
conditions.

41. A female is at risk of abnormal squamous cell proliferation of the cervical uterus due to
all, EXCEPT:
A. Multiple partner
B. Smoking
C. Early age of sexual activity
D. Oral contraceptives
E. Genetics
Ans: E
Tips: Women who take oral contraceptives for more than five years have an increased
likelihood of developing cancer.

42. A 29-year-old woman with abdominal pain was admitted to the hospital. Examination
shows retroperitoneal infections affecting endocrine gland. Which structure is affected?
A. Ovary
B. Suprarenal gland
C. Pancreas
D. Liver
E. Stomach
Ans: C

Copyright © 2000-2020 TIPS Inc. Unauthorized reproduction of this manual is prohibited. This
manual is being used during review sessions conducted by Pharmacy Prep.
5-11
Endocrine System

Copyright © 2000-2020 TIPS Inc. Unauthorized reproduction of this manual is prohibited. This
manual is being used during review sessions conducted by Pharmacy Prep.
5-12
Renal Disorder
PHARMACY PREP
RENAL DISORDERS

1. Factors that may increase the risk of renal failure may include:
A. Hemorrhage
B. Heart diseases
C. Oliguria
D. Pulmonary edema
E. All are correct
Ans: E

2. Reabsorption of drugs in kidney does NOT depend on?*


A. Flow rate
B. Tonicity FILTRATION
C. Tubular secretion ↓
REABSORPTION
D. pH

E. Metabolism TUBULAR SECRETION
Ans: C
Tips: Tubular secretion occurs after reabsorption in late distal convoluted tubules.
Strategic thinking:
1. Read the question.
2. In answering this type of question, remembering the SEQUENCE of a process is
helpful in addition to knowing the factors that have effect on each phase. In this
case, two other phases, namely Tubular secretion and Metabolism are mentioned.
However, Metabolism takes place before a drug’s reabsorption. So in essence, it does
affect reabsorption. Tubular secretion on the other hand follows after reabsorption
thus it does not affect it.
3. Correct answer is C.

3. Both glomerular filtrate rate and tubular secretion rate reduced in infants compared to adults.
Which list of drugs is most likely to have a prolonged clearance during infancy because of
immature renal function alone?
A. Amikacin, Caffeine, Retinoic Acid
B. Acetaminophen, Atomoxetine, Retinoic Acid
C. Acetaminophen, Benzoyl Alcohol, & Vancomycin
D. Amikacin, Gentamicin, Vancomycin
E. Atomoxetine, Lopinavir, & Vancomycin
Ans: D
Tips: Nephrotoxic drug that require dose adjustment in in infants. As infants and children have
reduced glomerular filtration rate and tubular secretion rate.

4. A patient has been diagnosed with hypercalcemia. Which of the following laboratory
investigation is used to determine hypercalcemia?
A. High parathyroid hormone (PTH) levels
Copyright © 2000-2020 TIPS Inc. Unauthorized reproduction of this manual is prohibited. This manual is being used during
review sessions conducted by PharmacyPrep.
6-1
Renal Disorder
B. Low parathyroid hormone (PTH) levels
↓ PTH= ↓ Ca and ↑ phosphate
C. High K levels
D. High phosphate levels ↑ PTH = ↑ Ca and ↓ phosphate
Ans: A

5. All of the following statements about renal failure are correct, EXCEPT:
A. Increased total body clearance
B. Increased drug half life
C. Decrease body clearance
D. Affect drugs that excrete via kidneys
E. Drugs metabolized via liver are the least to be affected
Ans: A

6. Secretion of potassium ions by the distal tubule will be decreased by:


A. Metabolic alkalosis
B. A high potassium diets
C. Hyperaldosteronism
D. Spironolactone administration
E. Thiazide administration
Ans: D
Tips: Spironolactone is an aldosterone antagonist, thus decrease potassium secretion into urine.
This will cause hyperkalemia.

7. Azotemia means?
A. Increase in nitrogen waste in blood
B. Increase in nitrogen waste in urine
C. Decrease in nitrogen waste in body
D. Decrease in nitrogen waste in urine
E. No change in nitrogen
Ans: A

8. Proteinuria is?
A. Proteins in urine
B. Proteins in blood
C. Proteins in food
D. Proteins in liver
E. Proteins in the brain
Ans: A

9. Which of the following conditions is the possible cause of Pre-renal acute renal failure?
A. Hemorrhage
B. High Blood pressure
C. Cancers
D. Creatinine clearance
Copyright © 2000-2020 TIPS Inc. Unauthorized reproduction of this manual is prohibited. This manual is being used during
review sessions conducted by PharmacyPrep.
6-2
Renal Disorder
E. Filtration
Ans: A

10. Intracellular alkalosis is caused by?


A. Spironolactone
B. Thiazides
C. Loop diuretics
D. ACEi
E. Mannitol
Ans: A
Tips: Intracellular alkalosis produced when increasing extracellular potassium above its normal
levels.

11. All of the following drugs cause metabolic alkalosis, EXCEPT:


A. Acetazolamide
B. Thiazides
Metabolic acidosis Metabolic alkalosis
C. Hydrochlorothiazide
Acetazolamide Thiazides,
D. Loop diuretics
K+ sparing Loop
E. Furosemide
ASA overdose osmotic
Ans: A

12. A man has a history of severe diarrhea. His condition led to the loss of HCO 3 - from GI tract that
causes a decrease in blood [HCO 3 -], and increase CO 2 . Laboratory values reveal the following:
pH = 7.25, pCO 2 = 24 mmHg, [HCO 3 -] = 10 mEq/L (22 mEq/L). Which of the following metabolic
disturbances best describes the man’s condition?
A. Metabolic alkalosis
B. Metabolic acidosis ↑ CO2 = Metabolic acidosis
C. Respiratory acidosis
D. Respiratory alkalosis
E. Normal acid base status
Ans: B
CO 2 HCO 3 pH ventilation
Metabolic acidosis ↑ ↓ ↓ normal
Metabolic alkalosis ↓ ↑ ↑ Normal
Respiratory acidosis ↑ normal ↓ Hypoventilation
Respiratory alkalosis ↓ normal ↑ Hyperventilation

13. Prerenal ARF is characterized as?


A. Inadequate blood circulation (perfusion) to the kidneys
B. Excessive blood perfusion to the kidney
C. Excessive drug elimination
D. Bladder cancer
E. All of the above

Copyright © 2000-2020 TIPS Inc. Unauthorized reproduction of this manual is prohibited. This manual is being used during
review sessions conducted by PharmacyPrep.
6-3
Renal Disorder
Ans: A
Tips: Prerenal ARF is characterized by inadequate blood circulation (perfusion) to the kidneys,
which leaves them unable to clean the blood properly. Many patients with prerenal ARF are
critically ill and experience shock (very low blood pressure). There often is poor perfusion within
many organs, which may lead to multiple organ failure.

14. A drug is metabolized in the liver and is renally eliminated. If patient has renal failure, what is
correct?
A. Drug accumulates in the blood and can cause toxicity
B. Drug eliminates from stools
C. Drug excretes in urine, and not metabolized
D. Drug have first pass metabolism
E. Drug has faster elimination and reabsorption
Ans: A
Strategic thinking:
1. Read the question. Identify and organize the information.
2. In answering this type of question, note of the process and organs involved and
establish a relationship between the processes in the absence or presence of a
disease of the organs involved. In this particular case, the drug is METABOLIZED in
the LIVER. After metabolism, the drug has been transformed into its form that can
be eliminated. Following the case, it is ELIMINATED via the KIDNEY. Now, the
patient has renal failure which means that the organ involved in the drug’s
elimination cannot do its job as it’s supposed to. So what happens to the drug?
Apparently, there is no elimination or excretion that can be expected. Instead, the
drug metabolite will remain in the system which is dangerous to the body.
In contrast, if the patient has liver failure instead, the answer will be different.
3. Correct answer is A.

15. What is incorrect about potassium levels?


A. Chronic renal disease may cause hyperkalemia
B. Adrenal cancer may cause hypokalemia
C. ACE Inhibitors like captopril may cause hypokalemia
D. Spironolactone may cause hyperkalemia
E. Hydrochlorothiazide may cause hypokalemia
Ans: C
Tips: ACE Inhibitors and ARBs cause hyperkalemia and thus captopril is an ACEi, and cause
hyperkalemia. Acute renal failure cause hyperkalemia and hyperphosphatemia and avoided in
acute renal disease.

16. What is correct about renal failure?


I. Renal failure not only alters the renal elimination, but also the non-renal disposition of
drugs that are extensively metabolized by the liver.
II. Renal failure reduces the drug clearance of renally eliminated drugs.

Copyright © 2000-2020 TIPS Inc. Unauthorized reproduction of this manual is prohibited. This manual is being used during
review sessions conducted by PharmacyPrep.
6-4
Renal Disorder
III. Chronic renal failure (CRF) has been shown to significantly reduce the non-renal
clearance and alter bioavailability of drugs predominantly metabolized by the liver and
intestine
A. I only
B. III only
C. I and II only
D. II and III only
E. All are correct
Ans: C

17. The shock and airway edema of anaphylaxis may be best treated by administration of which of
the following agents?
A. Salbutamol
B. Diphenhydramine
C. Adrenaline or epinephrine
D. Acetazolamine
E. Aminiphyline
Ans: C

18. Renal nephritis causes:


I. Glomerular to be permeable
II. Leak the proteins to urine
III. The capillary membrane becomes impermeable
A. I only
B. III only
C. I and II only
D. II and III only
E. All of the above
Ans: C
Tips: Renal nephritis = excessive secretion of proteins in urine.

19. Glomuronephritis symptoms:


I. Protein urea
II. Decrease in Na ions
III. Azotemia
A. I only
B. III only
C. I and II only
D. II and III only
E. All are correct
Ans: A

20. What is nephritic syndrome?


I. Damage of renal tubule
Copyright © 2000-2020 TIPS Inc. Unauthorized reproduction of this manual is prohibited. This manual is being used during
review sessions conducted by PharmacyPrep.
6-5
Renal Disorder
II. Excretion of protein in urine
III. Blood in gut
A. I only
B. III only
C. I and II only
D. II and III only
E. All are correct
Ans: C

21. A patient has renal disease. Which of the following medication used is the concern for the
pharmacist?
A. Opioids
B. Statins
C. ACE Inh.
D. NSAIDs
E. Codeine
Ans: D
Tips: Sulindac have least renal problems. However, all NSAIDs can cause chronic renal disease
and should be avoided.

22. Acute pre-renal failure is caused by all, EXCEPT:


A. Sepsis
B. Hypertension
C. Heart problems
D. Congestive heart failure
E. Gentamycin
Ans: E

23. A regular customer of your pharmacy, age 55 yo, medication profile includes Ramipril 10 mg,
valsartan 5 mg, and recently doctor added spironolactone. Patient diet include excessive intake
of bananas. All of the following are pharmacist concern, except?
A. Ramipril
DRUGS that cause HYPERKALEMIA
B. Valsartan ACEI
C. Spironolactone ARBs
D. Age 55 year old POTASSIUM SPARIN DIURETICS
BANANAS.
E. Bananas
Ans: D

24. Which of the following is the least preferable combination antihypertensive in a patient whose
K+ levels are 5.5 mEq/L?
A. Ramipril/Hydrochlorothiazide
B. Valsartan/Amlodipine
C. Perindopril/spironolactone
D. Losartan/Hydrochlorothiazide
Copyright © 2000-2020 TIPS Inc. Unauthorized reproduction of this manual is prohibited. This manual is being used during
review sessions conducted by PharmacyPrep.
6-6
Renal Disorder
E. Amlodipine/Atorvastatin
Ans: C
Tips: Precaution in combining two drugs with hyperkalemia side effects. Perindopril is an ACEi
cause hyperkalemia and spironolactone is aldosterone antagonist cause hyperkalemia.

25. Characteristics of diabetic nephropathy?


A. Albumin creatinine ratio > 3 mg/mL
B. Hypoalbuminemia
C. Edema
D. Hyperalbuminemia
E. hyperlipidemia
Ans: A

26. A patient eGFR declined significantly and doctor suspect chronic kidney diseases. All of the
following are associated with chronic kidney disease, EXCEPT:
A. Intravenous radiographic contrast media
B. NSAIDs or COX-II inhibitors
C. Volume depletion
D. Strict blood pressure or blood glucose control
E. Aminoglycosides
Ans: D

27. Severe renal disease stage 5. The creatinine clearance?


A. <70 mL/min
B. <50 mL/min
C. <30 mL/min
D. <15 mL/min
E. <0 mL/min
Ans: D

28. Which can be given to patient with renal problems to treat bowel movement problem
(constipation)?
A. Polyethylene glycol (PEG)
B. Sodium phosphate
C. Magnesium oxide
D. Laxative
E. Senna
Ans: A
Tips: polyethylene glycol (PEG) is safe laxative in renal disease patients.

29. Hemorrhoid patients should avoid all, EXCEPT:


A. Heavy lifting
B. Longtime sitting
C. Acidic food
Copyright © 2000-2020 TIPS Inc. Unauthorized reproduction of this manual is prohibited. This manual is being used during
review sessions conducted by PharmacyPrep.
6-7
Renal Disorder
D. Sitz bathing
E. Straining
Ans: D

30. JM is a patient in the last stage renal failure. Drug is excreted 10% unchanged in the urine.
What should be done regarding the dose?
A. No need to adjust the dose
B. Reduce the dose half
C. Reduce the dose to quarter
D. Increase the interval to double
E. Discontinue renal eliminated drugs
Ans: E

31. A 50-year-old male patient is using Hydrochlorothiazide for the past 4 years. Recently blood
pressure was uncontrolled. Doctor is considering to add antihypertensive therapy. Which of the
following is appropriate?
A. Amlodipine
Type I Type II
B. Metoprolol
ACE Inhibitors Thiazide
C. Ramipril ARBs Beta blockers
D. Verapamil Alpha blockers CCB
E. Chlorothiazide K sparing
Ans: C
Tips: If patient is using drug from type II if blood pressure is NOT controlled then add new drug
from type I. If patient first drug is hypokalemia drug than add another with hyperkalemia.

32. ASA over dose cause?


A. Hyperventilation
B. Hypoventilation
C. Metabolic alkalosis
D. Respiratory acidosis
E. Respiratory Acidosis And Metabolic Alkalosis
Ans: A
Tips: ASA overdose cause metabolic acidosis and respiratory alkalosis.

33. Hyperventilation is a condition due to?


A. Lowered CO 2 concentration followed in rise pH
B. High CO 2 concentration followed in rise pH
C. Lowered CO 2 concentration followed in lowered pH
D. Lowered CO 2 concentration
E. Lowered CO 2 without change in pH
Ans: A

34. A doctor prescribed high dose of Naproxen to treat osteoarthritis to a patient, with chronic
kidney disease stage 4. What is appropriate action?
Copyright © 2000-2020 TIPS Inc. Unauthorized reproduction of this manual is prohibited. This manual is being used during
review sessions conducted by PharmacyPrep.
6-8
Renal Disorder
A. Dispense Naproxen and tell patient that it is not safe to use in renal diseases
B. Tell patient, this is not safe to use in renal disease and will talk to doctor change safer
option
C. Tell patient to use low dose of Naproxen because of CKD
D. Change to Acetaminophen and tell the patient the reason of change of medication
Ans: B
Tips: Stage 4 chronic renal disease is CrCl 29 to 15 mL/min. NSAIDs are contraindicated in
patient with CrCl <30 mL/min.

35. An oral anticoagulant drug Dabigatran 75-100% renally eliminates. Before initiating Dabigatran
250 mg daily drug therapy what is monitored?
A. Liver function test
B. International normalized ratio
C. Creatinine Clearance
D. Blood urea nitrogen
E. Creatinine kinase
Ans: C

36. Serum creatinine is less in elder men compared to young because:


A. Protein binding is greater in elder.
B. Decreased body protein in muscles.
C. Renal clearance is greater in elder (low muscle mass).
D. Increased body protein.
Ans: B

37. Diuretic used in treatment of edema in a patient of renal failure:


A. Loop diuretic (furosemide)
B. Spironolactone
C. Thiazide
D. Chlorthalidone
Ans: A

Copyright © 2000-2020 TIPS Inc. Unauthorized reproduction of this manual is prohibited. This manual is being used during
review sessions conducted by PharmacyPrep.
6-9
Renal Disorder

Copyright © 2000-2020 TIPS Inc. Unauthorized reproduction of this manual is prohibited. This manual is being used during
review sessions conducted by PharmacyPrep.
6-10
Liver Function and Diseases

PHARMACY PREP
LIVER PHYSIOLOGY AND CHRONIC LIVER DISEASES

1. Which of the following is a common cause of liver cirrhosis?


A. Chronic alcoholism
B. Hepatitis A
C. Hepatic encephalopathies
D. Hepatitis B and C
E. Hypertension
Ans: A

2. All of the following can cause cirrhosis as a complication, EXCEPT:


A. Ascites
B. Liver cancer
C. Hepatic encephalopathy
D. Edema
E. Diabetes
Ans: E

3. The signs of alcoholic liver cirrhosis


I. Accumulation of fluid in peritoneal cavity (ascites)
II. Microcytic anemia
III. Blood in gut
A. I only
B. III only
C. I and II only
D. II and III only
E. I, II, III
Ans: E
Tips: Excess iron is stored as Ferritin in liver, spleen, bone marrow. Ferritin bind with
Fe2+ (ferrous) ion, which are oxidized to F3+ and deposited in the interior sphere as
ferrihydrite thus cause microcytic anemia.

4. Which of the following is NOT a chronic hepatitis infection?


A. Hepatitis A
B. Hepatitis B
C. Hepatitis C
D. Hepatitis B and C
E. None of the above
Ans: A

5. Which of the following test is the most specific to liver cirrhosis?


A. Bilirubin
B. AST

Copyright © 2000-2020 TIPS Inc. Unauthorized reproduction of this manual is prohibited. This manual is being
used during review sessions conducted by PharmacyPrep.
7-1
Liver Function and Diseases

C. ALT
D. Alkaline phosphatase (ALP)
E. Creatinine Kinase
Ans: D
Tips: Serious liver problems like liver cirrhosis alkaline phosphatase (ALP) increases.

6. Obstruction of bile secretion into duodenum from liver is (extra hepatic blockade)
referred as? *
A. Cholestasis or cholestatic jaundice
B. Cholecystitis
C. Cholelithiasis
D. Hepatic encephalopathy
E. Ascites
Ans: A
Tips: Cholestasis is a reduction or stoppage of bile flow. The cause of cholestasis is acute
hepatitis, alcoholic liver disease, primary biliary cirrhosis, hormonal changes in pregnancy,
stones in bile duct, cancer of bile duct, pancreatitis.

7. What is the drug treatment for chronic hepatitis C?


A. Ribavirin
B. Peginterferon
C. Interferon B
D. Lamivudine
E. Adefovir
Ans: B
Hepatitis A Hepatitis B Hepatitis C
No treatment needed Interferon shots (4 mo) Peginterferon combination
(infected get well in their Lamivudine po (1yr) with antiviral ribavirin
own) Adefovir po 2y

8. Which of the following OTC drug is most likely associated with hepatotoxicity?
A. Acetyl salicylic acid
B. Ibuprofen
C. Acetaminophen
D. Naproxen
E. All of the above
Ans: C

9. Initial symptoms of patient with liver toxicity?


A. Polydipsia and polyphagia
B. Shortness of breath
C. Abdominal pain and swelling
D. GI bleeding
E. Metallic taste in mouth
Ans: C

Copyright © 2000-2020 TIPS Inc. Unauthorized reproduction of this manual is prohibited. This manual is being
used during review sessions conducted by PharmacyPrep.
7-2
Liver Function and Diseases

10. Which of the following vaccine is least likely used among travelers?
A. Hepatitis A
B. Hepatitis B
C. Dukoral
D. Gardasil
E. None of the above
Ans: D
Tips: Gardasil vaccine is given for cervical cancer, from the age 13 to 26. this prevents
cervical cancer caused by papilloma virus.

11. Acetaminophen hepatotoxicity can be reduced by activation of which process?


A. Glucuronidation
B. Glutathione conjugation
C. Acetylation
D. Sulfate conjugation
E. Methylation
Ans: B
Tips: Glutathione conjugation activation reduces hepatotoxicity of acetaminophen.

12. Which of the following non-toxic end product of glutathione conjugation?


A. Acetylcysteine
B. cysteine
C. Mercapturic acid
D. Glutathione conjugation
E. Glutathione only
Ans: C

13. If a patient takes hepatitis B vaccine, this vaccine also protects?


A. Hepatitis A
B. Hepatitis C
C. Hepatitis D
D. Hepatitis A, B & C
E. Hepatitis A, B & D
Ans: C
Tips: Hepatitis B is a DNA type of virus whereas hepatitis D is RNA type of virus.
However, hepatitis D in order to grow requires assistance of hepatitis B. Thus, in
principle by preventing hepatitis B, infection of hepatitis D is also achieved.

14. Liver cirrhosis is a type of end stage chronic liver disease (liver cirrhosis). What clinical
laboratory test is used to confirm diagnosis?
A. ALT
B. AST
C. Bilirubin
D. Albumin

Copyright © 2000-2020 TIPS Inc. Unauthorized reproduction of this manual is prohibited. This manual is being
used during review sessions conducted by PharmacyPrep.
7-3
Liver Function and Diseases

E. Liver biopsy
Ans: E
Tips: The liver cirrhosis diagnosis can be confirmed by a liver biopsy (removal of a tissue
sample for examination under microscope). CT scan and ultrasound may show that liver
is shrunken or abnormalities.

15. In case of chronic alcoholism, which of the following liver enzymes are affected?
A. ALP
B. AST
C. ALT
D. Bilirubin
E. None of the above
Ans: B
Tips: Trans aminases AST to ALT ratio >2:1 strongly suggest alcohol abuse.

16. Hepatitis A is mainly caused by:


A. Sexual contact
B. Blood transfusion
C. Food and drink contamination
D. Traveling abroad
E. Drugs
Ans: C

17. A Canadian travelling to south East Asia should take what hepatitis vaccine?
A. Hepatitis A only
B. Hepatitis B only
C. Hepatitis A and B
D. Hepatitis A, B and C
E. Hepatitis B and C
Ans: C
Tips: Travel vaccinations are recommended by destination. Hepatitis A and B are
widespread in south East Asia.
Reference: www.travel.gc.ca

18. MK is a 50-year-old man diagnosed with ascites. What is the first line therapy for
ascites?
A. Peginterferons
B. Interferons
C. Lamivudine
D. Mineralocorticoids antagonist
E. Metolazone
Ans: D
Tips: Mineral corticosteroids are aldosterone. Spironolactone is an aldosterone
antagonist.

Copyright © 2000-2020 TIPS Inc. Unauthorized reproduction of this manual is prohibited. This manual is being
used during review sessions conducted by PharmacyPrep.
7-4
Liver Function and Diseases

19. Which of the following drugs causes Cholestatic jaundice as a side effect?
A. Cotrimoxazole
B. Erythromycin base
C. Erythromycin lactobacillus
D. Probiotics
E. Erythromycin estolate
Ans: E

20. Liver enzymes like cytochrome CYP450 causes drug metabolism in the liver. What is
NOT related to drugs that are metabolized in the liver?
A. Metabolism effects by tissue binding
B. Metabolism changes with liver blood circulation
C. Metabolism changes with intrinsic activity
D. Metabolism effected by chronic liver diseases
E. Oxidative metabolism is catalyzed by cytochrome CYP450.
Ans: A

21. Which of the following drugs metabolism decreased in chronic hepatic diseases?
A. Proton pump inhibitors
B. Warfarin
C. antacids
D. Lipid soluble drugs
E. Parenteral drugs
Ans: B
Tips: warfarin mechanism of action takes place in liver

22. A patient is on treatment of ascites using diuretics. Patient has osteoarthritis and using
pain medications. Which of the following drug use is the pharmacist’s concern?
A. Spironolactone
B. NSAIDs
C. Acetaminophen
D. Ibuprofen
E. Codeine
Ans: C
Tips: acetaminophen is hepatotoxic. It is concern in chronic liver diseases like ascites.

23. Lactulose, a laxative is used to treat which of the following chronic liver disease?
A. Constipation
B. Ascites
C. Encephalitis
D. Liver cirrhosis
E. Hepatitis
Ans: C

Copyright © 2000-2020 TIPS Inc. Unauthorized reproduction of this manual is prohibited. This manual is being
used during review sessions conducted by PharmacyPrep.
7-5
Liver Function and Diseases

Tips: Lactulose traps excessive ammonia that is produced by chronic liver disease
encephalitis.

24. Travelers should receive which of the following vaccine to prevent water and food
contaminated hepatitis?
A. Hepatitis A
B. Hepatitis B
C. Hepatitis A and B
D. Hepatitis C
E. Hepatitis D
Ans: A
Strategic thinking: in answering this type of question:
1. Read the question.
2. In answering this type of question, it is important to know that mode of
transmission of the disease and make an association of the information included in
the question. This question mentions WATER and FOOD which go into the mouth.
Among the hepatitis conditions, Hepatitis A and E are acquired by orofecal route.
3. Thus, correct answer is A only.
.
25. What type of hepatitis can be prevented by prophylaxis?
A. Hepatitis A
B. Hepatitis A and B
C. Hepatitis B and C
D. Hepatitis A, B, C
E. Hepatitis A, B, C, D and E
Ans: B

26. Hepatitis B is transmitted by all, EXCEPT:


A. Contact with blood and its products
B. Sexual transmission
C. Contact oral secretion
D. Sharing needle
E. Food and water
Ans: E
Tips: Food water, oro fecal can cause hepatitis A.

27. Phase II metabolic reaction Glucuronidation is affected by Enterohepatic recirculation.


Which of the following drugs are affected?
A. Drugs that undergoes phase II metabolism
B. Estrogen/progestins oral contraceptive pills
C. Drugs with phase I metabolism
D. Drug with first pass metabolism
E. Lipid soluble drugs
Ans: A

Copyright © 2000-2020 TIPS Inc. Unauthorized reproduction of this manual is prohibited. This manual is being
used during review sessions conducted by PharmacyPrep.
7-6
Liver Function and Diseases

28. What is the most common cause of alcoholic liver diseases?


A. Corticosteroids therapy
B. Acetaminophen toxicity
C. Alcohol abuse
D. Ascites
E. Wilson's disease
Ans: C

29. A customer at your pharmacy presents with severe pain after motor vehicle accidents.
His current medical condition includes chronic liver disease. Which of the following is
least likely recommended for pain?
A. Ibuprofen
B. Codeine with Acetaminophen
C. Naproxen
D. ASA
E. Codeine with ASA
Ans: B

30. A man who uses large dose of anabolic steroid is at risk of?
A. Anemia
B. Hyperprolactinemia
C. Hirsutism
D. Cholestatic jaundice
E. Testicular enlargement
Ans: C
Tips: Anabolic steroids are androgen hormone or testosterone. These hormones cause
manly features such as excessive hair growth and muscle growth.

31. What type of chronic liver disease is caused by buildup of ammonia in the brain and
causes dementia-like symptoms such as confusion, forgetfulness, agitation, poor
concentration and judgment and slow speech?
A. Ascites
B. Alcoholic liver cirrhosis
C. Hepatic encephalopathies
D. Cholestatic jaundice
E. Hepatitis
Ans: C

32. Which occurs in liver cirrhosis?


A. Hemorrhage
B. Jaundice
C. Ascites
D. All of the above
Ans: D

Copyright © 2000-2020 TIPS Inc. Unauthorized reproduction of this manual is prohibited. This manual is being
used during review sessions conducted by PharmacyPrep.
7-7
Liver Function and Diseases

33. MK is a 50-year-old man diagnosed with ascites. All of the following are causes of
ascites, EXCEPT:
A. Tuberculosis
B. Abdominal surgeries
C. Congestive heart failure
D. Stroke
E. Liver cirrhosis
Ans: D
Tips: Stroke is cerebrovascular conditions and not associated with chronic liver disease
ascites.

34. Pathway of blood from liver to systemic circulation?


A. Mesenteric vein and portal vein
B. Portal artery and hepatic vein
C. Portal vein and hepatic artery
D. Hepatic vein  hepatic artery mesenteric
E. Coronary artery
Ans: D
Tips: Liver  Heart (hepatic vein) HEART Hepatic artery  mesenteric artery to GI
area

35. A patient’s creatinine clearance is 300 mL/hour, it is a?


A. Chronic renal disease
B. Normal renal function
C. Excessive renal function
D. Severe renal disease
E. Acute renal disease
Ans: D
Tips: Patient CrCl per min is 5 mL/min. Normal creatinine clearance is 80-120
mL/minutes. If this is per hours 100 mL x 60 min = 6000 ml/hour.

36. In Acetaminophen overdose, the first to be elevated is?


A. Lactate dehydrogenase (LDH)
B. Alanine transaminase (ALT)
C. Aspartate Transaminase (AST)
D. Gama glutamyl Trans peptidase (GGP)
E. Alkaline phosphatases (ALP)
Ans: C
Tips: AST is elevated because it is more sensitive.

37. Jaundice in newborns is caused by a deficiency of:


A. Glucose
B. UDP glucose
C. Glucuronate

Copyright © 2000-2020 TIPS Inc. Unauthorized reproduction of this manual is prohibited. This manual is being
used during review sessions conducted by PharmacyPrep.
7-8
Liver Function and Diseases

D. UDP glucuronate
E. UDP glucuronyl transferase
Ans: E
Tips: Hyperbilirubin or high bilirubin levels causes jaundice. The high bilirubin inhibits the
action of disphosphoglucuronic acid (UDPGA) a glucuronyl transferase responsible for
conjugation and subsequent excretion of bilirubin.

38. A young man comes to the pharmacy complaining of headache. He has history of
hepatitis because of using a particular drug in the past. What would be your
recommendation?
A. Acetaminophen 500 mg
B. Acetaminophen plus codeine
C. Ibuprofen
D. Enteric coated Acetyl salicylic acid ASA
E. Acetaminophen plus methocarbamol
Ans: C
Tips: patient with liver disease avoid acetaminophen. NSAID can be used instead.

39. Correct statement regarding α-Fetoprotein-AFP includes:


I. Produced by the fetal liver
II. May cause hepatocellular carcinoma
III. Causes death of fetus
A. I only
B. III only
C. I and II only
D. II and III only
E. All of theabove
Ans: C
Tips: α-Fetoprotein-AFP is produced by the fetal liver and may cause hepatocellular
carcinoma of fetus, however it does not cause the death of fetus.

40. What is the first line therapy for Hapatitis C?


A. Peginterferon
B. Lamivudine
C. Ribavirin
D. Sovaldi (sofosbuvir)
E. Harvoni (ledipasvir+sofosbuvir)
Ans: A

41. Jaundice in newborns is caused from a deficiency of:


A. Glucose
B. UDP glucose
C. Glucuronate
D. UDP glucuronate

Copyright © 2000-2020 TIPS Inc. Unauthorized reproduction of this manual is prohibited. This manual is being
used during review sessions conducted by PharmacyPrep.
7-9
Liver Function and Diseases

E. UDP glucuronyl transferase (UDPGT)


Ans: E
Tips:
Bilirubin-UGT enzyme performs a phase 2 metabolic reaction Glucuronidation. During
this reaction enzyme transfer a glucuronic acid to unconjugated bilirubin it to conjugated
bilirubin. Glucuronidation dissolvable in water so it removes from body. UDPGT is also
used to treat or prevent jaundice.
Glucose-6-phosphate dehydrogenase (G6PD) deficiency increase production of bilirubin
in jaundice in newborn.

Copyright © 2000-2020 TIPS Inc. Unauthorized reproduction of this manual is prohibited. This manual is being
used during review sessions conducted by PharmacyPrep.
7-10
Respiratory System

PHARMACY PREP
RESPIRATORY COMPLICATIONS

1. The capacity of inhalation is depending on the volume of?


A. Alveoli
B. Total lung capacity
C. Inspiratory capacity
D. Tidal volume
E. Vital capacity
Ans: C
Tips: Inspiratory capacity = the sum of TV + IRV
Total lung capacity is the volume in the lungs after a maximal inspiration.
Total lung capacity includes tidal volume, inspiratory reserve volume. Expiratory volume
and residual volume.
Inhalation (Inspiration) Expiration (Exhaling)
Lung volume increases Lung volume decreases

2. Dyspnea means:
A. Painful muscle spasms
B. Pain in the heart
C. Pain in extremities
D. Painful breathing or shortness of breath
E. Painful menstruation
Ans: D

3. Emphysema is?
A. COPD
B. Fibrosis
C. Bronchitis
D. Airway disease
E. None of the above
Ans: A

4. Condition characterized by progressive, partially irreversible form of airflow obstruction


is known as:
A. Aneurism
B. Emphysema
C. Embolism
D. Cirrhosis
E. Jaundice
Ans: B
Tips: Emphysema is chronic and irreversible permanent enlargement of alveoli.
Aneurysm is an abnormal dilatation of a blood vessel while embolism is an obstruction

Copyright © 2000-2020 TIPS Inc. Unauthorized reproduction of this manual is prohibited. This manual is being
used during review sessions conducted by PharmacyPrep.
8-1
Respiratory System

of airway by a mass transported in the circulation. Cirrhosis and jaundice are liver
related complications.

5. Advantages of nasal route over oral route absorption of drugs may include:
I. Nasal route has a higher surface area than GI tract
II. Nasal route has more blood vessels
III. Nasal route has a higher onset of absorption
A. I only
B. III only
C. I and II only
D. II and III only
E. All are correct
Ans: D
Tips: Although nasal route has a smaller surface area than intestine drugs administrated
by nasal route will have faster onset of action due to large quantity of blood vessels in
the nasal route.

6. A 3-year-old child was newly diagnosed with asthma. All of the following questions are
important to ask his mother, EXCEPT:
A. Any cardiovascular problems?
B. Does other family member have asthma?
C. How often symptoms come?
D. Old asthma medication he used before?
E. How many times wake up in night due to symptoms
Ans: A

7. Example of leukotriene antagonist drug includes:


A. Salbutamol
B. Ipratropium
C. Zafirlukast
D. Montelukast
E. C and D are right
Ans: E
Tips: Zafirlukast and Montelukast are classics leukotriene antagonists used in the oral
treatment of asthma

8. Which reaction can ozone cause in the lungs?


A. Oxidation
B. Hydration
C. Reduction
D. Complexation
E. Hydrolysis
Ans: A

Copyright © 2000-2020 TIPS Inc. Unauthorized reproduction of this manual is prohibited. This manual is being
used during review sessions conducted by PharmacyPrep.
8-2
Respiratory System

Tips: Ozone is a gas that may cause irritation and toxicity of pulmonary system. The
main reaction of ozone in the body is oxidation.

9. Correct statements regarding glossitis include which of the following?


I. Inflammation of the tongue
II. Characterized by presence of pinkish-red central lesions on the dorsal surface
of the tongue
III. It can be a chronic condition of the tongue seen in pernicious anemia.
A. I only
B. III only
C. I and II only
D. II and III only
E. All are correct
Ans: E
Tips: Glossitis is an inflammatory disease of the tongue of unknown etiology
characterized by multiple annular areas of desquamation of the filiform papillae on the
dorsal surface of the tongue. Usually presenting pinkish-red central lesions, normally
seen in pernicious anemia.

10. Respiratory acidosis is a result of:


A. Increased partial pressure of CO 2
B. Reduced partial pressure of CO 2
pCO2  (>45 mmHg)
C. Increased ventilation CO 2 pH  (<7.35)
D. Reduced ventilation of CO 2
E. A and D
Ans: E
Tips: Respiratory acidosis is a condition that occurs when the lungs cannot expire enough
of the CO 2 produced by the body. Excess CO 2 causes pH of blood and other body fluids
decrease and become acidic.

11. Initial treatment of chronic obstructive pulmonary disease may include:


A. Salbutamol
B. Short Acting Branchodilators (Salbutamol or/and Ipratropium)
C. Salmereol
D. Sodium cromoglycate
E. Fluticasone
Ans: B
Tips: Short acting bronchodilators salbutamol or/and Ipratropium is the initial treatment
of COPD-Chronic Obstructive Pulmonary Disease by blocking the bronchoconstriction
caused by activation of the parasympathetic nervous system.

12. Oxygen is carried in blood by hemoglobin iron. The iron state that binds to oxygen is?
A. Ferric
B. Ferrous

Copyright © 2000-2020 TIPS Inc. Unauthorized reproduction of this manual is prohibited. This manual is being
used during review sessions conducted by PharmacyPrep.
8-3
Respiratory System

C. Ferritin
D. Hematocrit
Ans: B
Tips: The iron in the ferrOus state (Fe2+) binds with O2. If iron is in the ferric (Fe3+)
state, it is methemoglobin which does not bind with oxygen.

13. Asthma is a condition of respiratory tract that may be aggravated by?


A. Allergens
B. Cold weather
C. Exercise
D. Emotional stress
E. All are correct
Ans: E
Tips: Asthma is a chronic inflammatory disorder of the airway that may be induced by
allergens, occupational exposure, exercise, emotion, weather, drugs and so on.

14. Which of the following is considered as the best treatment of asthma?


A. β 2 -agonists
B. β-Blockers
C. α-agonists
D. Anticholinergic drugs
E. Cholinergic drugs
Ans: A
Tips: Inhaled short-acting β-agonists are the most effective drugs available for all the
kinds of asthma treatment.

15. Which of the following agents or combinations may be appropriate to treat severe
intermittent asthma?
A. High doses of inhaled corticosteroid
B. Long acting β2-agonists
C. Short acting β2-agonists
D. Oral corticosteroids
E. Combination of all the above medication
Ans: E
Tips: Severe intermittent asthma treatment is best done by short and long-acting β-
agonists plus inhaled and oral corticosteroids.

16. Examples of drug(s) that may precipitate asthma include all of the following, EXCEPT:
A. Aspirin
B. β-Blockers
C. Ipratropium
D. NSAIDs
E. Cholinergic drugs
Ans: C

Copyright © 2000-2020 TIPS Inc. Unauthorized reproduction of this manual is prohibited. This manual is being
used during review sessions conducted by PharmacyPrep.
8-4
Respiratory System

Tips: Ipratropium is an anticholinergic drug that be used in adjunct treatment of asthma.

17. All of the following are pathophysiologic characteristics of asthma, EXCEPT:


A. Airway obstruction and inflammation
B. Destructive changes in the alveolar walls
C. Thickened smooth muscle of airway
D. Mucosal edema
E. Narrowed lumen of airway
Ans: B
Tips: Destructive changes in the alveolar walls are characteristics of a respiratory
complication called “Emphysema”

18. The MOST well-known characteristic symptom of asthma include:


A. Wheezing
B. Mucosal edema
C. Cough
D. Chest tightness
E. Tachycardia
Ans: A
Tips: Mainly symptoms of asthma are: Wheezing, shortness of breathing, chest
tightness, cough, tachycardia, and tachypnea and pulses paradoxus.

19. Which of the following drugs is considered as the drug of choice in a specific asthma
emergency treatment?
A. Salmeterol
B. Adrenaline
C. Salbutamol (Albuterol)
D. Inhaled corticosteroides
E. Zafirlukast
Ans: C
Tips: Salbutamol (Albuterol) is the fastest acting β-agonist used in the treatment of
bronchoconstriction emergencies-asthma.

20. Salbutamol is a short acting β 2 -agonists that may be used in the treatment of which of
the following asthma type?
A. Mild asthma
B. Emergency asthma
C. Severe intermittent asthma
D. Moderate asthma
E. All kind of asthma
Ans: E
Tips: Salbutamol is a fast acting β-agonist and considered the drug of choice in all kind of
asthma.

Copyright © 2000-2020 TIPS Inc. Unauthorized reproduction of this manual is prohibited. This manual is being
used during review sessions conducted by PharmacyPrep.
8-5
Respiratory System

21. All are correct concerning the action of corticosteroids in asthma status, EXCEPT:
A. Suppress the inflammatory response
B. Decrease production of inflammatory mediators
C. Decrease airway responsiveness to inflammation
D. Relieve bronchoconstriction
E. Increase β-agonist receptors response
Ans: D
Tips: The use of corticosteroids in asthma suppresses the inflammatory response and
decreases the airway hyperesponsiveness by decreasing the production of inflammatory
mediators and mucus production. Combination therapies of ICS+LABA like budesonide +
formoterol have additive effect.

22. All are examples of corticosteroids inhalers used in asthma, EXCEPT:


A. Beclomethasone
B. Sodium cromoglycate
C. Flunisolide
D. Fluticasone
E. Budesonide
Ans: B
Tips: Sodium cromoglycate is an anti-inflammatory, antiallergenic and mast cell
stabilizer drug.

23. What drugs cause bronchoconstriction?


A. β2-agonists
B. Anticholinergic
C. Methyl xanthines
D. Beta blockers
E. Leukotriene modifiers
Ans: D
Tips: beta blockers cause bronchoconstriction. Beta 2agonist (salbutamol, formoterol,
salmeterol), anticholinergics (ipratropium, tiotropium), and methylxanthine
(theophylline) and Leukotriene inhibitors (Montelukast) cause bronchodilatation.

24. Best drug used in the prevention of exercise-induced asthma may include:
A. Long acting β2-agonists
B. Short acting β2-agonists
C. Anticholinergics
D. Leukotriene modifiers
E. Corticoids
Ans: B
Tips: Salbutamol, a fast acting β-agonist, is the most effective drug in the prevention of
exercise-induced asthma.

Copyright © 2000-2020 TIPS Inc. Unauthorized reproduction of this manual is prohibited. This manual is being
used during review sessions conducted by PharmacyPrep.
8-6
Respiratory System

25. All are β2-agonists used as brochodilators in the treatment of bronchospasm, EXCEPT:
A. Albuterol
B. Salmeterol
C. Terbutaline
D. Pindolol
E. Levalbuterol
Ans: D
Tips: Pindolol is a β-blocker used mainly in the treatment of hypertension.

26. Example of methyl xanthine used in the treatment of respiratory complications includes:
A. Theophylline
B. Terbutaline
C. Ipratropium
D. Zafirlukast
E. Salmeterol
Ans: A
Tips: Theophylline is chemical class of Methyl xanthine. Ipratropium is an anticholinergic
drug used in COPD as the drug of choice.

27. Agents considered as precursor of leukotriene include:


A. Tyrosine
B. Cyanide
C. Arachidonic acid
D. Prostaglandins
E. Histamine
Ans: C
Tips: Cysteinyl leukotrienes are products of arachidonic acid metabolism. Leukotriene
modifiers drugs inhibit leukotriene synthesis by inhibiting 5-lipoxygenase that catalyzes
the conversion of arachidonic acid to leukotriene.

28. Indication of leukotrienes modifiers in the treatment of asthma includes which of the
following?
I. Acute treatment
II. Prophylaxis
III. Chronic treatment
A. I only
B. III only
C. I and II only
D. II and III only
E. All are correct
Ans: D
Tips: Leukotriene modifiers are approved for oral prophylaxis and chronic treatment of
asthma.

Copyright © 2000-2020 TIPS Inc. Unauthorized reproduction of this manual is prohibited. This manual is being
used during review sessions conducted by PharmacyPrep.
8-7
Respiratory System

29. Drug known as the new relative cromolyn like drug includes:
A. Cromolyn sodium
B. Cromoglycate sodium
C. Cromolyn disodium
D. Disodium Cromoglycate
E. Nedocromil sodium
Ans: E
Tips: Nedocromil sodium is the new relative Cromolyn like drug acting in a very similar
way as sodium Cromoglycate does.

30. Leukotriene antagonist is referred to as steroid sparing agents, thus it is indicated to:
A. Reduce corticosteroid doses
B. To avoid increase of corticosteroid dose
C. Do not used together with corticosteroids
D. It increases toxicities of steroids
Ans: A

31. Emphysema is?


A. Asthma
B. Acute bronchitis
C. Chronic bronchitis
D. Permanent enlargement of alveoli
E. None of the above
Ans: D

32. Montelukast, a Leukotriene antagonist’s therapeutic use includes?


I. Acute asthma exacerbations
II. Asthma maintenance
III. ASA induced asthma
A. I only
B. III only
C. I and II only
D. II and III only
E. All are correct
Ans: D

33. Patient has a relevant increase in his asthma symptoms. He is currently using salbutamol
and fluticasone inhalers. What would you advise as an adjunct medication to treat his
symptoms?
A. Oral corticosteroids
B. Higher dose of corticosteroid inhalers
C. Salmeterol
D. Montelukast
E. Anticholinergic agents

Copyright © 2000-2020 TIPS Inc. Unauthorized reproduction of this manual is prohibited. This manual is being
used during review sessions conducted by PharmacyPrep.
8-8
Respiratory System

Ans: D
Tips: Leukotriene antagonist is added to patient whose asthma is not controlled by
SABA+ICS, then add LTRAs.

34. In the treatment of bronchoconstriction all of the following drugs can be used, EXCEPT:
A. Beta 2 agonist
B. Anticholinergics
C. Methylxanthines
D. Xanthine oxidase inhibitors
E. Leukotriene modifiers
Ans: D

35. What drugs are used to acute asthma?


A. Inhaled corticosteroids
B. Salbutamol
C. Salmeterol
D. Prednisone po
E. Hydrocortisone iv
Ans: B

36. Peak flow meter is a test to determine asthma severity for patient at home. This test
measures?
A. The highest forced expiratory flow
B. Volume that has been exhaled at the end of the first second (FEV 1 ).
C. Total lung capacity
D. Expiration rate
E. Inspiration rate
Ans: A

37. Spirometer is a device used in clinics to diagnose asthma. Spirometer measures?


A. The highest forced expiratory flow
B. Volume that has been exhaled at the end of the first second (FEV 1 ).
C. Total lung capacity
D. Expiration rate
E. Inspiration rate
Ans: B
Tips: 80-120% of predicted is normal value
70-80% demonstrates mild reduction/restriction
50-70% demonstrates moderate reduction
<50% demonstrates severe reduction

38. Which of the following is the most serious respiratory complication associated with
smoking?
A. Asthma

Copyright © 2000-2020 TIPS Inc. Unauthorized reproduction of this manual is prohibited. This manual is being
used during review sessions conducted by PharmacyPrep.
8-9
Respiratory System

B. COPD
C. Upper respiratory tract infections
D. Otitis media
E. Pneumonia
Ans: B
Strategic thinking:
1. Read the question.
2. In answering this type of question, you can apply multiple strategies.
Eliminate options that do not have direct association with smoking (Otitis
media and Asthma). Another, look for your clue, in this case, MOST SERIOUS
and COMPLICATION. URTI and Pneumonia although they are serious, they
can develop from other causes, mostly bacterial of origin. Smoking may
aggravate the conditions but they are not complications. The question asks of
a complication and COPD develops in people who are long-term smokers.
3. Thus, answer is B.

39. Drugs used to treat asthma causes:


A. Bronchodilatation
B. Vasodilatation
C. Bronchoconstriction
D. Arterial dilatation
E. Vasoconstriction
Ans: A

40. Factors that decrease the respiration, EXCEPT:


A. Mucosal edema
B. Increase brachial secretion
C. Increase ventilation
D. Bronchospasm
Ans: C
Tips: Hyperventilation cause deep and fast breathing and increase respiration.

41. Defined as the gaseous exchange that occurs at alveoli capillary membrane:
A. Respiration
B. Ventilation
C. Inhalation
D. Expiration
E. Inspiration
Ans: A

42. Bronchodilators for asthma:


A. Are often used as meter-dose inhalers, diskus, turbuhaler
B. Are obsolete since the introduction of nedocromil
C. Include inhaled theophylline, B 2 agonists, and oral corticosteroids

Copyright © 2000-2020 TIPS Inc. Unauthorized reproduction of this manual is prohibited. This manual is being
used during review sessions conducted by PharmacyPrep.
8-10
Respiratory System

D. Are ineffective by inhaled route.


Ans: A
Tips: Nedocromil is not a bronchodilator.

43. A customer presents with symptoms of cough, fever, fatigue, and weight loss. What is
correct?
A. Refer to doctor because symptoms likely suggest asthma.
B. Refer to doctor because symptoms likely suggest COPD.
C. Refer to doctor because symptoms likely suggest tuberculosis.
D. Refer to doctor because symptoms likely suggest emphysema.
Ans: C

44. What is the normal breathing rate?


A. 30 to 60 breaths/min
B. 12 to 16 breaths/min
C. 1 to 2 breaths/min
D. 10 to 12 breaths/min
E. 80 to 120 breaths/min
Ans: B

45. What is the cause of sialorrhea in children?


A. Poor oral and facial muscle control
B. Poor oral hygiene
C. Gingival hyperplasia
D. Teething pain
E. Undeveloped organ
Ans: A

46. Which of the following test confirms the diagnosis and assesses severity of Chronic
Obstructive Pulmonary Disorder (COPD)?
A. Arterial blood gas test
B. Peak flow meter
C. Spirometer
D. Bronchoprovocation test
E. Metacholine challenge
Ans: C

47. Cystic fibrosis treated by Dapsone is taken by:


A. IV
B. IM
C. Inhalation
D. Nasal inhaler
Ans: A

Copyright © 2000-2020 TIPS Inc. Unauthorized reproduction of this manual is prohibited. This manual is being
used during review sessions conducted by PharmacyPrep.
8-11
Respiratory System

Copyright © 2000-2020 TIPS Inc. Unauthorized reproduction of this manual is prohibited. This manual is being
used during review sessions conducted by PharmacyPrep.
8-12
Urinary System
PHARMACY PREP
URINARY SYSTEM

1. A 54-year-old patient was recently diagnosed with BPH. He is using the following
medications: Captopril, sildenafil for his erectile dysfunction and
Hydrochlorothiazide. He has no allergy but has postural hypotension but not high.
What drug you should recommend to treat the shrinkage of prostate in BPH?
A. Finasteride
B. Tamsulosin
C. Dutasteride
D. Doxazosin
E. Prazosin
Ans: A
Tips: Finasteride inhibits type II and dutasteride types I and II isoenzymes of 5-alpha-
reductase, which blocks the metabolism of testosterone to dihydrotestosterone.
Reduces prostatic volume. Alpha1 antagonist Tamsulosin, doxazosin, prazosin
increase urine flow rate by used to block alpha1-adrenergic receptors that mediate
smooth muscle activity in the bladder neck, prostate and prostatic capsule, reducing
the dynamic component of bladder outlet obstruction.

2. Which of the following is not associated with pelvic area?


A. Cystitis
B. Urethritis
C. Pyelonephritis
D. Abdomen
E. Inguinal hernia
Ans: D

3. Which of the following is NOT a symptom of urinary tract infection?


A. Urinary urgency
B. Painful urination
C. Fever
D. Lower back pain
E. Weight loss
Ans: E
UTI signs and symptoms include: urinary frequency, urgency, voiding in small
amount, inability empty bladder, painful urination, cloudy urine, foul smelling urine,
hematuria, lower abdominal discomfort, lower back pain, and fever.

4. All of the following drugs can be used in patient with urinary incontinence, EXCEPT:
A. Duloxetine
B. Bethanechol
C. Diphenhydramine
D. Furosemide
Copyright © 2000-2020 TIPS Inc. Unauthorized reproduction of this manual is prohibited. This manual is being
used during review sessions conducted by PharmacyPrep.
9-1
Urinary System

E. Oxybutynin
Ans: D

5. The drug of choice for the treatment of urinary incontinence is?


A. Diphenhydramine
B. Ipratropium
C. Tiotropium
D. Furosemide
E. Oxybutynin
Ans: E

6. Which of the following is NOT a symptom of Benign Prostatic Hyperplasia?


A. Frequent urine
B. Nocturia
C. Irritation
D. Jet urination
E. All of the above
Ans: D
Tips: jet urination or urine stream is not possible in benign prostatic hyperplasia.

7. Which of the following is the most common symptom of benign prostatic


hyperplasia?
A. Frequent urine due to incomplete voiding of urine
B. Fever
C. Flank pain
D. Jet urination
E. Low back pain
Ans: A
Tips: in BPH normal stream of urination is not possible.

8. What is the most common type of Urinary Tract Infection (UTI)?


A. Complicated UTI
B. Uncomplicated UTI
C. Cystitis
D. Pyelonephritis
E. Urethritis
Ans: C
Tips: cystitis is bladder infection, and the most common type of urinary tract
infections.

9. What is the most common pathogen that causes of urinary tract infection (UTI)?
A. S. aureus
B. P. aeruginosa
C. E. coli
Copyright © 2000-2020 TIPS Inc. Unauthorized reproduction of this manual is prohibited. This manual is being
used during review sessions conducted by PharmacyPrep.
9-2
Urinary System

D. Syphilis
E. Gonorrhea
Ans: C

10. What is the drug of choice to treat cystitis in patient with renal disease CrCl <30
ml/min?
A. Amoxicillin
B. Cefixime
C. Cotrimoxazole
D. Nitrofurantoin
E. Trimethoprim
Ans: C
Tips: First line therapies for uncomplicated cystitis is Cotrimoxazole, trimethoprim,
nitrofurantoin. Fosphomycin. However patient with renal disease avoid
nitrofurantoin is avoided. Since Cotrimoxazole is combination of sulphamethoxazole
and trimethoprim. Is preferred over just trimethoprim alone. Amoxicillin alone is not
used anymore in UTI due to resistance. Cefixime is not used anymore for UTI. If
patient has sulfa allergy and renal disease then choose second line option cephalexin
or fluoroquinolones.

11. What is the distinguishing symptom of complicated UTI?


A. Fever
B. Turbid urine (blood in urine)
C. Dysuria
D. Frequent urination
E. Burning on urination
Ans: B

12. What drug is least effective in the treatment of uncomplicated cystitis?


A. Fosfomycin
B. Cephalexin
C. Cotrimoxazole
D. Nitrofurantoin
E. Moxifloxacin
Ans: E
Tips: Moxifloxacin and levofloxacin is respiratory tract fluoroquinolones are often
used for the treat pneumonia.
Cystitis first line therapy is Cotrimoxazole 3days, or trimethoprim 3 days, or
Nitrofurantoin 5 days or Fosfomycin single dose of po.

13. Which of the following is distinguishing symptoms of pyelonephritis?


A. Urine frequency
B. Urine urgency
C. Hematuria
Copyright © 2000-2020 TIPS Inc. Unauthorized reproduction of this manual is prohibited. This manual is being
used during review sessions conducted by PharmacyPrep.
9-3
Urinary System

D. Flank pain
E. Nausea and vomiting
Ans: D
Tips: Flank pain or pelvic in urinary tract infection is identification of pyelonephritis.

14. What is the drug of choice to treat benign prostatic hyperplasia?


A. Doxazosin
B. Tadalafil
C. Finasteride
D. Saw palmetto
E. Oxybutynin
Ans: C

15. All of the following used to treat benign prostatic hyperplasia (BPH), EXCEPT:
A. Doxazosin
B. Tadalafil
C. Finasteride
D. Saw palmetto
E. Oxybutynin
Ans: E
Tips: Oxybutynin is the drug of choice for urinary incontinence.

16. Which of the following runs through the centre of prostate?


A. Bladder
B. Ureter
C. Urethra
D. Kidney
E. Arteries
Ans: C

17. Which of the following causes stress incontinence?


A. Relaxed pelvic floor
B. Urethral blockade
C. Inability void urine
D. Urgency of urine
E. Emotional stress
Ans: A

18. A 70-year-old Parkinson's disease patient often has to be in the office and airport for
business travels. Often, he is bothered by involuntary passage of urine. His condition
can be categorized as:
A. Involuntary incontinence
B. Urge incontinence
C. Stress incontinence
Copyright © 2000-2020 TIPS Inc. Unauthorized reproduction of this manual is prohibited. This manual is being
used during review sessions conducted by PharmacyPrep.
9-4
Urinary System

D. Overflow incontinence
E. None
Ans: B

19. Which of the following conditions have NO urinary burning sensation?


A. Urinary tract infections
B. Prostatitis
C. Benign prostatic hyperplasia
D. Urinary incontinence
E. Cystitis
Ans: D

20. Which of the following medical conditions causes difficulty in stream urination?
A. Urinary tract infections
B. Prostatitis
C. Benign prostatic hyperplasia
D. Urinary incontinence
E. Cystitis
Ans: C

21. Hyperplasia is?


A. Increased in cell size
B. Increased in cell numbers
C. Decreased in cell size
D. Decreased in cell numbers
E. Increase cell size and numbers
Ans: B

22. Hypertrophy is:


A. Increased in cell size
B. Increased in cell numbers
C. Decreased in cell size
D. Decreased in cell numbers
E. Increase cell size and numbers
Ans: A

23. Which of the following causes tumours?


A. Hypertrophy
B. Hyperplasia
C. Atrophy
D. Hyperacidity
E. Urinary incontinence
Ans: B
Strategic thinking: in answering this type of question:
Copyright © 2000-2020 TIPS Inc. Unauthorized reproduction of this manual is prohibited. This manual is being
used during review sessions conducted by PharmacyPrep.
9-5
Urinary System

1. Read the question.


2. In answering this type of question, determine the meaning of your
KEYWORD. In this case, tumours, that results from rapid cell proliferation.
Try to associate the meaning to the choices and pick the one that has
similar connotation.
3. Thus, answer is B.

24. Urinary incontinence in children is defined as repeated daytime or night time voiding
urine into the bed or cloths. It is termed as?
A. Bed sores
B. Bed wetting
C. Wetting Phenomenon
D. All of the above
Ans: B
Tips: Enuresis or bed wetting can occur in children ages between 6 to 7 year old.
(over 7 concern). Most children bladder control occurs typically at age of 5 to 6.

25. Enuresis is bed wetting is treated by?


A. Diuretics
B. Desmopressin
C. Fluoxetine
D. Diphenhydramine
Ans: B
Tips: Desmopressin is antidiuretic hormone analog.

26. Tamsulosin is used to treat benign prostatic hyperplasia symptoms. It blocks alpha 1a
receptors and facilitates urine flow in what vessel?
A. Coronary arteries
B. Bladder arteries
C. Prostate arteries
D. Cerebral arteries
E. All of the above
Ans: B
Tips: Alphazosin is selectived alpha1a receptor antagonist. These receptors are
mainly present on bladder artries.
To avoid first-dose syncope (severe hypotension), start doxazosin and terazosin at a
low dosage and gradually increase until symptomatic improvement or intolerance
occurs. Dose titration is not necessary with alfuzosin, silodosin and tamsulosin.

27. Testosterone to dihydrotestosterone is catalyzed by 5-alpha reductase. The


dihydrotesterone causes prostate growth. What drugs inhibit 5-alpha reductase and
reduce prostate size?
A. Alpha blockers
B. Tamsulosin
Copyright © 2000-2020 TIPS Inc. Unauthorized reproduction of this manual is prohibited. This manual is being
used during review sessions conducted by PharmacyPrep.
9-6
Urinary System

C. Terazosin
D. Androgen agonist
E. Androgen antagonist
Ans: E
Tips: Finasteride and Dutasteride are androgen antagonist act by inhibiting 5-alpha
reductase. The drug of choice for BPH.

28. Which of the following is least likely associated with uncomplicated Urinary Tract
Infections?
A. Fever
B. Turbid urine
C. Frequent urination
D. Burning
E. Urinary Frequency
Ans: B

29. In differential diagnosis of urinary tract symptoms assessment, which of the


symptoms are least likely associated with urinary tract infection?
A. Fever
B. Turbid urine
C. Frequent urination
D. Itch on vaginal area with white discharge
E. Urinary Frequency
Ans: D
Tips: Itch on vaginal area and white discharge is the most common symptoms of vaginal
candidiasis.
Strategic thinking: in answering this type of question:
1. Read the question.
2. Look for your clues. In this question, LEAST LIKELY. As many conditions have
overlapping symptoms, it is important to be familiar with ONE SYMPTOM
unique to a particular disease. From the question, all but one is compatible
with UTI. Option D indicates a condition specific to women affecting their
genitals. (a fungal infection)
3. Thus, answer is D.

30. MP is a 50-year-old man currently using Ramipril 10 mg for high blood pressure for
the past 10 years, and Atorvastatin 20 mg daily for hyperlipidemia for the past 5
years. His triglycerides were elevated so his doctor added Fenofibrate therapy to
manage this. Recently, he has been diagnosed with renal calculi. Which of the
following drugs could be cause of his renal calculi?
A. Ramipril
B. Losartan
C. Fenofibrate
D. Atorvastatin
Ans: C
Copyright © 2000-2020 TIPS Inc. Unauthorized reproduction of this manual is prohibited. This manual is being
used during review sessions conducted by PharmacyPrep.
9-7
Urinary System

31. Where is sperm produced from?


A. Prostate gland
B. Seminal vesicle
C. Seminiferous tubules
D. Testes
Ans: C
Tips: The tests contain coiled structure called seminiferous tubules which are the
sites of sperm production.

Copyright © 2000-2020 TIPS Inc. Unauthorized reproduction of this manual is prohibited. This manual is being
used during review sessions conducted by PharmacyPrep.
9-8
The Eye
PHARMACY PREP
The Eye

1. What is the other name of blind spot?


A. Optic disc
B. Retina
C. Cornea
D. Optic nerve
E. None of the above
Ans: A

2. Which of the following is associated with aqueous humor?


A. Retina
B. Lens
C. Ciliary gland
D. Sweat glands
E. Tear duct
Ans: C
Tips: Aqueous humor is produced by ciliary gland in anterior eye chamber.

3. Vitreous humor is present in?


A. Knee joint
B. Shoulder join
C. Eye
D. Hormone glands
E. Ear
Ans: C
Tips: The clear gel fills the space between the lens and the retina of eyeball of human.

4. What is the rate-limiting step in ophthalmic drops?


A. Optic disc
B. Retina
C. Cornea
D. Optic nerve
E. None of the above
Ans: C
Strategic thinking: in answering this type of question:
1. Read the question.
2. Look for your clues and keywords. Your clue is rate-limiting which means
that the kinetics of the drug is largely dependent on it. Keyword, opthalmic
drops. Since this is topically applied, we are looking for a part that comes in
contact with the drug FOREMOST.
3. Thus, answer is C.

Copyright © 2000-2020 TIPS Inc. Unauthorized reproduction of this manual is prohibited. This manual is being
used during review sessions conducted by PharmacyPrep.
10-1
The Eye
5. What drug causes contact lens discoloration side effect?
A. Amiodarone
B. Hydroxy chloroquine
C. Rifampin
D. Ethambutol
E. Isotretinoin
Ans: C
Tips: drugs that cause ocular toxicities are Amiodarone (corneal deposits),
Hydroxychloroquine (ocular toxicity), Ethambutol and Isotretinoin (vision changes).
Rifampin causes discoloration of contact lens.

6. JP is a 50-year-old asthma patient using Salbutamol PRN. He is also receiving


treatment for hypertension which is Metoprolol 50 mg daily. Recently, JP was
diagnosed with glaucoma. Doctor is considering prescribing Latanoprost to treat
glaucoma instead of Timolol. Why does the doctor choose prostaglandin analog?
A. Patient has asthma & using salbutamol
B. Patient has hypertension using metoprolol
C. Latanoprost is the first line therapy for open angle glaucoma
D. Patient is a 50-year-old, asthma and hypertension
E. Patient has open angle glaucoma
Ans: A
Tips: salbutamol is beta 2 agonist and timolol is non-selective beta blockers. CAT. X.
Should not combine these medications.
Strategic thinking:
1. Read the question carefully. Make sense of what is being asked.
2. Questions like these may present choices that are all correct in sense. Use
multiple strategies to increase your chance of arriving at the best option.
Look for clues. DOCTOR has already chosen a drug suitable for the patient.
Then, eliminate options less relevant with the doctor’s action. So, B, C and
E are excluded. Between A and D, A is a better choice on the basis that it
directly cites Salbutamol which has interaction with Timolol as explained
in TIPS.
3. Correct answer is A.

7. At night dark vision (dim light), the most sensitive photoreceptors in retina for
motion?
A. Cones R = Rods, so dark
B. Rods C = Cone, for Color
C. Cones and rods
D. Optic nerve
E. Cornea
Ans: B

Copyright © 2000-2020 TIPS Inc. Unauthorized reproduction of this manual is prohibited. This manual is being
used during review sessions conducted by PharmacyPrep.
10-2
The Eye
8. What photoreceptors are active in color light?
A. Cones
B. Rods
C. Cones and rods
D. Optic nerve
E. Cornea
Ans: A
Tips: C for color and C for cones

9. What is myopia?
A. Night vision
B. Far sight
C. Near sight
D. Color vision
E. Color blindness
Ans: C

10. Drugs that cause pupil dilatation, except?


A. Morphine
MYDRIASIS = DILATATION
B. Atropine (YARD)
C. Benztropine
D. Ipratropium MIOSIS = CONSTRICTION
E. Trihexyphenidyl (INCH)
Ans: A
Tips: Morphine overdose cause pinpoint pupil. All anticholinergic drugs cause mydriasis
or pupil dilatation. All anticholinergic drugs cause pupil dilatation

11. Hyperopia is?


A. Far sight
B. Near sight
C. Glaucoma
D. None of the above
Ans: A

12. The sclera and cornea constitute this layer of the eye:
A. Retinal tunic
B. Vascular tunic
C. Fibrous tunic
D. Nervous tunic
Ans: C
Tips: The sclera and cornea form the fibrous tunic of bulb of the eye, the sclera is
opaque and constitutes the posterior part of tunic. The cornea is transparent and
forms the anterior part.

Copyright © 2000-2020 TIPS Inc. Unauthorized reproduction of this manual is prohibited. This manual is being
used during review sessions conducted by PharmacyPrep.
10-3
The Eye
13. "Night blindness" is an early sign of:
A. Cataracts
B. Glaucoma
C. Vitamin a deficiency
D. Myopia
E. Cones photoreceptors
Ans: C
Tips: In dark or night rods are more sensitive. Active pigmentation of rods
photoreceptor is rhodopsin, which vitamin A derivative. So, vitamin A deficiency cause
night blindness

14. Sympathetic stimulation of the iris causes:


A. Astigmatism
B. Pupillary constriction
C. Pupillary dilation
D. Glaucoma
Ans: C

15. Cone cells are photoreceptors that:


A. respond to low light levels
B. respond to colored light
C. are found in the optic disc
D. are found in the vascular tunic
Ans: B

16. The region where the lens focuses the image onto the retina is the:
A. Optic nerve
B. Fovea
C. Pupil
D. Blind spot
Ans: B
Tips: Fovea in the eye a tiny pit located in the macula of the retina that provides clear
vision. This area concentrated with cones. Light falls directly on cone. The cones are
active in day light and color.

17. The fluid that fills the posterior chamber of the eye is the:
A. Lacrimal fluid
B. Vitreous humour
C. Aqueous humour
D. Jocular humour
Ans: B

18. The major light absorbing pigment in retinal rod photoreceptors is:
A. Rhodopsin

Copyright © 2000-2020 TIPS Inc. Unauthorized reproduction of this manual is prohibited. This manual is being
used during review sessions conducted by PharmacyPrep.
10-4
The Eye
B. Melanin
C. Glutamate
D. Chlorophyll
E. Carotenoids
Ans: A

19. Photoreceptors release more neurotransmitters:


A. in brighter light
B. in darkness
C. only when stimulated by bipolar cells
D. none of the above
Ans: B

20. What is incorrect about sympathetic blockers, example Timolol in the treatment of
glaucoma?
A. Decrease glaucoma
B. Decrease IOP by inhibiting formation of aqueous humor.
C. Decrease IOP by increase outflow of aqueous humor.
D. First line therapy for glaucoma
E. Ophthalmic drops are available
Ans: C
Tips: Timolol decreases IOP by inhibiting formation of aqueous humor.

21. Optic nerve is a cranial nerve and function covers vision. What is correct?
A. Serves muscle of the eye
B. A sensory nerve only for vision
C. A motor and sensory nerve
D. Serves sensory from face, mouth, motor to mastication
Ans: B

22. MP is a 55-year-old male using Timolol 1 gtts bid ou. If patient medication profile has
the following drugs, which of these medications are the pharmacist concern?
A. Hydrochlorothiazide
B. Salbutamol
C. Ramipril
D. Metformin
E. Levothyroxine
Ans: B
Tips: Timolol is beta2 blocker and salbutamol is b2 agonist and therefore can cause
drug -drug interactions.

23. In prescription sig. Timolol 1 gtt bid ou what is correct direction?


A. Instill one drop in both eyes
B. Instill one drop two times daily in both eyes

Copyright © 2000-2020 TIPS Inc. Unauthorized reproduction of this manual is prohibited. This manual is being
used during review sessions conducted by PharmacyPrep.
10-5
The Eye
C. Instill two drops once daily in both eyes OU= both eye
D. Instill one drop two times daily in right eye OD = right eye
E. Instill one drop two times daily in left eye OS = left eye
Ans: B

24. A prescription of ciprofloxacin + dexamethasone OTIC prep? What is correct


statement?
A. Instil in eye only
B. Instil in ear only
C. Can be instilled in eye and ear
D. Taken orally
E. Applied topically
Ans: B
Tips: Eye drops preparation should be sterile. However OTIC ear drop can be sterile or
no sterile. Eye drops can instill in eye and ear. But OTIC instilled only in ear.

25. What is the drug of choice to treat open angle glaucoma in patients preferring once
daily dose?
A. Latanoprost
B. Latanoprost + timolol
C. Timolol
D. Ciprofloxacin ophthalmic
E. Dexamethasone
Ans: A
Tips: Latanoprost once daily one drop at 9pm or bedtime and is the first line therapy.
Timolol is used bid.

26. A doctor prescribed Timolol 1 gtt daily as directed. What is correct?


A. Instill one drop in both eyes
B. Instill one drop in left eye
C. Instill one drop in right eye
D. Instill one drop in affected eye
E. Instill one drop in eye and ear
Ans: D

27. MK is a 55-year old asthma patient and taking salbutamol. Recently diagnosed for
open angle glaucoma. Doctor is considering prescribing therapy for glaucoma. Which
of the following is least likely suitable?
A. latanoprost
B. Dorzolamide
C. Acetazolamide
D. Timolol
E. Pilocarpine
Ans: D

Copyright © 2000-2020 TIPS Inc. Unauthorized reproduction of this manual is prohibited. This manual is being
used during review sessions conducted by PharmacyPrep.
10-6
The Eye
Tips: Patient with Asthma avoid non-selective beta blockers like Timolol increase
asthma.

28. What are the more in number and more sensitive photoreceptors in eye?
A. Cones
B. Rods
C. Rods and cones
D. Cataract
E. Pupil
Ans: B

29. What is correct statement about photo receptors in eye?


A. Rods are sensitive to color light
B. Cones are active in dark
C. Rods are less in number than cones
D. Cones are less in number than rods
E. Cones and rods photo cells are equally sensitive
Ans: D

30. Which of the following drugs can cause dry eye side effect?
A. Salbutamol
B. Timolol
C. Latanoprost
D. Epinephrine
E. Atropine
Ans: E

31. Emergency medication in glaucoma?


A. Pilocarpine
B. Acetazolamide
C. Atropine
D. Timolol
E. Latanoprost
Ans: A

32. MP, a60-year-old women has been prescribed Latanoprost 1 drop daily at bedtime to
treat glaucoma. Pharmacist counsels patient on how to instill eye drop. She also
reminded MP to apply gentle pressure on inner cantus after instilling eye drop
because:
A. This increased absorption of eye drops from cornea
B. This minimizes systemic absorption and avoid systemic side effects
C. This increases drug viscosity and increase contact time
D. This delays absorption of drug and improve drug effectiveness
Ans: B

Copyright © 2000-2020 TIPS Inc. Unauthorized reproduction of this manual is prohibited. This manual is being
used during review sessions conducted by PharmacyPrep.
10-7
The Eye
Tips: Applying gentle pressure in inner cantus reduces systemic absorption and side
effects of eye drops.

33. Which of following describes proper technique of administering eye drops?


A. Till the head forward. With the forefinger, pull down the lower eyelid to form a
pocket.
B. Touch the tip of the dropper against the eye when administering the drops.
C. Immediately after administering the drops, press a forefinger against the inside
corner of the eye.
D. Blink eye lids after instilling eye drops
Ans: C

34. A mother of an 8-year-old boy comes at your pharmacy with a concern. Her son has
ear pain, fever and red swelling in ear. The pain started after swimming lessons
yesterday. What is appropriate?
A. Refer to doctor
B. Recommend OTC acetaminophen and if no improvement in 2 days see the doctor
C. Recommend antibiotics and so ask to see doctor
D. All external ear problem can cause infection, so refer to doctor
Ans: A

35. Total power of the eye is 59 diopters. Main role is maintenance of this power as
performed by:
A. Anterior surface of cornea
B. Lens
C. Vitreous Humor
D. Retina
E. Posterior surface of cornea
Ans: B
Tips: Diopter is a unit of measurement of the optical power of lens.
Or
Diopter is a unit of refractive power that is equal to the reciprocal of the focal length
(in meter) of given lens.
The total optical power of lens in human eye is 60 diopters. The cornea accounts for
approximately two thirds of this refractive power (about 40 diopters) and crystalline
lens contributes to remaining one third (20 diopters).

Copyright © 2000-2020 TIPS Inc. Unauthorized reproduction of this manual is prohibited. This manual is being
used during review sessions conducted by PharmacyPrep.
10-8
Blood and Anemia

PHARMACY PREP
BLOOD AND ANEMIA
1. Which of the following plasma glycoproteins controls the levels of the free iron in biological
fluids?
A. Total iron binding capacity
B. Transferrin protein
C. Serum ferritin
D. Hemoglobin
E. Myoglobin
Ans: B

2. Microcytic anemia is characterized as?


A. Hypochromic anemia
B. Vitamin B 12 deficient anemia
C. Folate deficient anemia
D. Sickle cell anemia
E. Decreased total iron binding capacity
Ans: A

3. In microcytic anemia (iron deficient), all of the following decrease, EXCEPT:


A. Mean cell volume (MCV)
B. Hematocrit (Hct)
C. Hemoglobin
D. Serum ferritin
E. Total iron binding capacity (TIBC)
Ans: E
Tips: In microcytic anemia, total iron binding capacity increase. Hematocrit is actual volume of
RBCs in a unit volume of whole blood).

4. Megalobastic anemia is characterized as?


A. Iron deficient anemia
B. Vitamin B 12 or folic acid deficient anemia
C. Folate deficient anemia
D. Sickle cell anemia
E. Pernicious anemia
Ans: B

5. Pernicious anemia is characterized as?


A. Iron deficient anemia
B. Vitamin B 12 deficiency only
C. Folate deficient anemia
D. Sickle cell anemia
E. Vitamin B 12 and folic acid deficiency

Copyright © 2000-2020 TIPS Inc. Unauthorized reproduction of this manual is prohibited. This manual is being used during
review sessions conducted by PharmacyPrep.
11-1
Blood and Anemia

Ans: B
Tips: Pernicious anemia is due to deficiency of intrinsic factor. This factor is essential for vitamin
B12 absorption.

6. KP, a 35-year-old woman is using ferrous gluconate to treat anemia. What decreases absorption
of ferrous gluconate that requires to be separately taken?
A. Taking Food
B. Proton pump inhibitors
C. Citrus juice
D. Apple juice
E. Grapefruit juice
Ans: B
Tips: Proton pump inhibitors may reduce the absorption of oral iron salts. Antacids aluminum,
magnesium, calcium decrease absorption of iron. So separate as much as possible.

7. Intrinsic factors are secreted from?


A. Esophagus
B. Stomach
C. Small intestine
D. Large intestine
E. Parietal cells
Ans: E

8. Pernicious anemia patient should NOT take vitamin B 12 supplement by?


A. Oral
B. Subcutaneous injection
C. Intra muscular injection
D. Intravenous
E. None
Ans: A

9. Which of the following screening tests is used to confirm Iron Deficiency Anemia (IDA)?*
A. Mean cell volume
B. Serum folate levels
C. Serum ferritin levels
D. Red blood cells levels
E. Transferrin saturation
Ans: C
Tips: The screening test for IDA is ferritin level. IDA is typically defined as ferritin level <30
ng/mL. However, infections, inflammation and obesity can influence ferritin level, in these
situations, value changes to <50 ng/mL.
Transferrin saturation <16%.

Copyright © 2000-2020 TIPS Inc. Unauthorized reproduction of this manual is prohibited. This manual is being used during
review sessions conducted by PharmacyPrep.
11-2
Blood and Anemia

10. MP had motor vehicle accident and lost large volume of blood. Which of the following anemia
is possible?
A. Megaloblastic anemia
B. Normocytic anemia
C. Microcytic anemia
D. Sickle cell anemia
E. Hemolytic anemia
Ans: C
Tips: excessive blood loss can cause iron deficiency anemia.

11. A woman planning to be pregnant should take which of the following supplements?
A. Vitamin B 12 supplements
B. Iron supplements
C. Folic acid supplements
D. Intrinsic factor
E. Vitamin B 12 or folic acid
Ans: C
Tips: Folic acid supplement should begin 3 months before pregnancy and continue for first 3
months or pregnancy.

12. Megaloblastic anemia is due to deficiency of?


A. Vitamin B 12
B. Iron
C. Folic acid
D. Intrinsic factor
E. Vitamin B 12 and folic acid
Ans: E

13. A 75-year-old frequent customer of your pharmacy has been diagnosed with megaloblastic
anemia. The most likely cause of megaloblastic anemia in elderly persons is due to the
deficiency of?
A. Vitamin B 12
B. Iron
C. Folic acid
D. Hemolytic anemia
E. Genetic defect
Ans: A

14. Filgrastim is:


A. Interleukin
B. Colony stimulating factor
C. Tumor necrosis factor
D. Oprelvekin
E. Erythropoietin's

Copyright © 2000-2020 TIPS Inc. Unauthorized reproduction of this manual is prohibited. This manual is being used during
review sessions conducted by PharmacyPrep.
11-3
Blood and Anemia

Ans: B

15. The MOST common cause of megaloblastic anemia is


A. Gastric bleeding
B. Lack of dietary iron
C. Lack of dietary calcium
D. Lack of dietary pyridoxine
E. Lack of absorption of vitamin B 12
Ans: E

16. Hematocrit is?


A. Circulating iron storage
B. Measures the total iron binding capacity
C. Measures the total free iron
D. Packed volume of RBCs
E. It is serum ferritin
Ans: D
Tips: The percentage of red blood cells to the blood volume is the Hct. (Packed cell volume).

17. Which of the following blood cells cannot be distinguished from basophils?
A. Neutrophil
B. White blood cells
C. Eosinophils
D. Red blood cells
E. Mast cells
Ans: E

18. Low mean cell volume (MCV) indicates?


A. Megaloblastic anemia
B. Microcytic anemia
C. Hemolytic anemia
D. Normocytic anemia
E. Aplastic anemia
Ans: B

19. Pernicious anemia is


A. Due to deficiency intrinsic factor thus no oral vitamin B 12 absorption
B. Due to deficiency of vitamin B 12 , administer oral vitamin B 12
C. Due to deficiency of folic acid supplements, administer folic acid supplements
D. Due to deficiency of iron supplements, administer iron supplements
E. Due to deficiency of vitamin B 12 and folic acid supplements
Ans: A

Copyright © 2000-2020 TIPS Inc. Unauthorized reproduction of this manual is prohibited. This manual is being used during
review sessions conducted by PharmacyPrep.
11-4
Blood and Anemia

20. A customer of your pharmacy calls and expresses a concern that his 2-year-old daughter
swallowed 10 iron capsules this morning past 4 hours ago. She showed no unusual symptoms.
What should you do?
A. Wait and watch
B. Ask to go emergency
C. Contact poison control centre and provide phone number
D. Recommend to take excessive fluids
E. Ask to see doctor
Ans: B

21. What is true about microcytic anemia?


A. It may cause due deficiency of iron during pregnancy
B. Deficiency of folic intake can cause microcytic anemia
C. Deficiency of folic acid can cause microcytic anemia
D. Deficiency of vitamin B 12 can cause microcytic anemia
E. Deficiency of intrinsic factor
Ans: A

22. Laboratory tests for iron deficiency anemia, EXCEPT:


A. Hemoglobin (HgB)
B. Hematocrit (Hct)
C. Erythrocyte Sedimentation Rate
D. Serum Ferritin
E. Iron binding capacity
Ans: C
Tips: Erythrocytes sedimentation rate (ESR) is not screening test for microcytic anemia. ESR is
used acute, chronic infections, malignancies, rheumatic factors, and seizures.

23. A patient needs something for anemia as confirmed in his diagnosis. He did not hoever take any
medication and is now overwhelmed by sensation of fainting and complaining of black stool.
What should you do?
A. Recommend to see his doctor right away
B. Symptoms may due to GI bleeding
C. Ask if pink color stool as maybe lower GI bleeding
D. Ask about allergy before giving him something
E. Recommend iron supplements
Ans: A

24. Which of the following iron supplements have the highest elemental iron?
A. Ferrous sulfate
B. Ferrous gluconate
C. Ferrous fumarate
D. Combination of iron supplements
E. All of the above

Copyright © 2000-2020 TIPS Inc. Unauthorized reproduction of this manual is prohibited. This manual is being used during
review sessions conducted by PharmacyPrep.
11-5
Blood and Anemia

Ans: C
Tips: Elemental iron: Ferrous fumarate 33%>Ferrous sulfate 20%> Ferrous gluconate 12%.

25. The most accurate screening for Iron Deficiency Anemia (IDA) is serum ferritin. What is true
about serum ferritin in IDA?
A. Increases because serum ferritin stores iron.
B. Increases because serum absorbs water in iron deficiency.
C. Decreases because serum ferritin stores iron.
D. Decreases because serum ferritin increases protein and iron stores
E. No change in serum ferritin
Ans: C
Tips: Serum ferritin is protein that stores iron. In case of iron deficiency, tissue absorbs water and dilutes
iron stores thus decrease serum ferritin.

26. A 59-year-old lady comes to the pharmacy to buy some iron supplements. She is currently using
ibuprofen 400 mg qid for her joint pains. She feels very weak and has sensations of fainting. She
observed occult blood in stool, however she is NOT sure. What is your appropriate action?
A. Recommend ferrous fumarate because high elemental iron
B. Recommend ferrous glucose because it is light on stomach
C. Tell her to stop taking ibuprofen and refer her to doctor to determine cause of bleeding.
D. Ask her to stop taking ibuprofen
E. Ask her to take low dose of ibuprofen
Ans: C

27. Pregnancy related anemia is due to?


A. Deficiency of iron
B. Decrease in blood volume of plasma
C. Deficiency of vitamin b 12
D. Deficiency of folic acid
E. Deficiency of folic acid and vitamin b 12
Ans: A

28. Which of the following is the cause of megaloblastic anemia?


A. Due to impaired synthesis of serum ferritin
B. Impaired DNA synthesis
C. Impaired red blood cells formation
D. Due genetic defects in formation of rbc
E. Due to deficiency G6PD enzyme
Ans: B
Tips: Megaloblastic anemia arise because of impaired DNA synthesis caused by deficiency of
vitamin B 12 (Cobalamine) or folate or due to impaired DNA and RNA metabolism.

29. Which of the following is correct about ferrous gluconate 300 mg dose?
A. Available By Prescription
B. Pharmacist recommended

Copyright © 2000-2020 TIPS Inc. Unauthorized reproduction of this manual is prohibited. This manual is being used during
review sessions conducted by PharmacyPrep.
11-6
Blood and Anemia

C. Over the counter or unscheduled


D. Not available in Canada
Ans: B
Tips: According to NAPRA Iron supplements are placed under schedule II (behind the counter
drugs. These drugs can be prescribed or recommended by pharmacist. Iron supplements
(elemental iron <30 mg) is placed under over the counter drugs and present in self selection area
of pharmacy.

30. KP, a 30-year-old woman walks into your pharmacy. She wants to buy iron supplements
because she feels tired and fatigued. What is your appropriate action?
A. Recommend ferrous gluconate 300 mg daily.
B. Recommend ferrous sulfate because has the highest elemental iron.
C. Pharmacist should assess patient by asking her if she had seen doctor for anemia
symptoms.
D. Ask her take more rest and diet.
Ans: C
Tips: Pharmacist should assess patient about anemia symptoms before recommending iron
supplements.

31. Which of the following is the common deficiency in pregnancy?


A. Iron
B. Vitamin B 12
C. Folic acid
D. Multivitamin
E. Dimenhydrinate
Ans: A

32. A 35-year-old woman who is a frequent customer of your pharmacy has been diagnosed with
megaloblastic anemia. She is on treatment of epilepsy and using phenytoin for the past 5
months. Her megaloblastic anemia is due to deficiency of?
A. Vitamin B 12
B. Iron
C. Folic acid
D. Intrinsic factor
E. Genetic defect
Ans: C

33. What it the dose of folic acid supplement patient taking carbamazepine?
A. 1 mg
B. 100 mg
C. 200 mg
D. 1 to 5 mg
E. 300 mg
Ans: D

Copyright © 2000-2020 TIPS Inc. Unauthorized reproduction of this manual is prohibited. This manual is being used during
review sessions conducted by PharmacyPrep.
11-7
Blood and Anemia

Tips: To minimize neurological side effects of carbamazepine, folic acid 5 mg is supplemented.

34. Blood hemoglobin transport oxygen in systemic circulation. The form of iron present in
hemoglobin is?
A. Ferric Fe3+ Hemoglobin iron complex consist
B. Ferrous Fe 2+ ferrous Fe2+ iron.
C. Oxidized form
Ferric (Fe3+) containing hemoglobin
D. Oxyhemoglobin
condition is know as
E. Methemoglobin
methemoglobinemia.
Ans: B

35. Transferrin?
A. Stores iron in blood
B. Transports iron in blood
C. Oxidized form of iron in blood
D. Preserve iron ionic form in blood
E. Reduced iron ion in blood
Ans: B

36. A 75-year-old elderly person has deficiency of Cyanocobalamin (Vit B 12 ). This can lead to:
A. Megaloblastic anemia
B. Microcytic anemia
C. Intrinsic factor
D. Iron deficient anemia
Ans: A

37. What glycoproteins measures the free iron in blood?


A. Haemoglobin
B. Transferrin Transferrin: iron Transport protein in blood
C. Serum ferritin measures free iron.
D. Hematocrit Serum ferritin: Iron Storage proteins and used
Ans: B for iron deficiency anemia

38. All of the following causes iron deficiency anemia, EXCEPT:


A. GI bleeding
B. Chronic diseases
C. Thalassemia
D. Folate deficiency
E. Peptic ulcers
Ans: D

39. Choose the statement that ranks diagnostic test in order of specificity of iron deficiency
anemia. The least to most?
A. Ferritin, MCV, hemoglobin count

Copyright © 2000-2020 TIPS Inc. Unauthorized reproduction of this manual is prohibited. This manual is being used during
review sessions conducted by PharmacyPrep.
11-8
Blood and Anemia

B. Hemoglobin count, ferritin and MCV


C. Hemoglobin count, MCV and serum ferritin
D. MCV, hemoglobin count and serum ferritin
Ans: D
Tips: Anemia is most commonly defined as reduction in hemoglobin concentration <135 g/L in men and
Hgb <120 g/L in women. Serum ferritin <30 ng/mL.
Strategic thinking:
1. Read the question carefully. Make sense of what is being asked.
2. In questions asking for a set or group of answers, be cautious in your selection by being
guided by your clues, as these provide insight on how the set should be arranged. In this
case, LEAST to MOST. In some questions, it could ask for the reverse (most to least). Also,
to improve your answering ability, it helps to know either the MOST specific or most
sensitive test in the confirmation of a diagnosis as there are several tests that can be done
but not all are equally specific or sensitive. Finally, you can easily to eliminate other
options that do NOT NAME/MEET your first criterion of the set.
3. Thus, D is the answer and all other options are eliminated because they do not begin with
MCV.

40. Vitamin B12 and folic acid supplementation in megaloblastic anemia leads to the improvement
of anemia due to:
A. Increased DNA synthesis in bone marrow
B. Increased hemoglobin production
C. Erythroid hyperplasia
D. Increased iron absorption
E. None of above
Ans: A

41. What is normally present freely in Plasma?


A. Thrombin
B. Prothrombin
C. Fibrin
D. Plasminogen
Ans: B
Tips: Prothrombin is protein produced by liver, blood clotting. Prothrombin produce thrombin
by activation of clotting factor X.

Clotting Factor X Fibrinogen


Prothtrombin --------------->Thrombin ------------>Fibrin  blood clot
All of the following contains iron and are related to non-heme iron (haemoglobin), EXCEPT:
A. Ferritin
B. Transferrin
C. Superoxide dismutase
D. Cytochrome oxidase
E. Methane monooxygenase
Ans: E

Copyright © 2000-2020 TIPS Inc. Unauthorized reproduction of this manual is prohibited. This manual is being used during
review sessions conducted by PharmacyPrep.
11-9
Blood and Anemia

Tips: Methane monooxygenase or MMO is an enzyme capable of oxidizing C-H bond in methane
as well as other alkanes. The active site in MMO contains a di-iron centre bridge by oxygen (Fe-
O-Fe).

42. What serum proteins transport iron from GI track to bone marrow?
A. Serum ferritin
Serum ferritin: stores iron
B. Transferrin
Transferrin; transports iron
C. Hematocrit
Hematocrit: RBC volume
D. Ferritin Ferritin: iron binding protein
Ans: B
Tips: Transferrin is the iron transport protein in serum.

43. The high transferrin may indicate?


A. Iron deficiencies
B. Excessive iron transport
C. Megaloblastic anemia
D. Normocytic anemia
Ans: A
Tips: The higher transferrin level indicates an iron deficiency anemia. Transferrin is also
measured by total iron binding capacity.

44. A 13-year-old girl presented to doctor with fatigue, anorexia, pallor of skin, mucus membrane,
conjunctiva, some time shortness of breath and chest pain. Her lab test shows serum ferritin
<20 and Hemoglobin < 100. She should take?
A. Folic acid supplements
B. IV Iron infusion
C. Vitamin B 12
D. All of the above
Ans: B
Tips: symptoms of severe iron deficient anemia and laboratory test serum ferritin <30 ng/mL
and Hemoglobin is low so iron infusion is helpful.

45. Which of the blood cells are most affected by anti-cancer drugs therapy?
A. Neutrophils
B. Thrombocytes
C. Lymphocyte
D. Erythrocytes
E. Basophils
Ans: A

46. Highest iron absorption, if taken?


A. Empty stomach
B. With food
C. With orange juice

Copyright © 2000-2020 TIPS Inc. Unauthorized reproduction of this manual is prohibited. This manual is being used during
review sessions conducted by PharmacyPrep.
11-10
Blood and Anemia

D. With Or Without Food


E. With milk
Ans: C
Strategic thinking:
1. Read the question carefully.
2. Look for clues and keywords. In this case, clue is HIGHEST while the keyword, ABSORPTION.
In light of the question, Iron as you learned should be taken on an empty stomach.
However, question is asking you what can help achieve maximum absorption of the drug;
with iron it needs an acidic environment.
3. Thus, C is the correct answer.

47. Sustained-release iron tablets are ineffective in patient because of following reason:
A. High side effects
B. Chance of dose dumping
C. Does not get release in gastric environment
D. Site specific action
E. All of above
Ans: C

48. Decrease in platelet count is caused by:


A. Malignancy
B. Rheumatoid Arthritis
C. Systemic Lupus Erythematosus
D. Polycythemia vera
E. Postsplenectomy syndrome
Ans: A

49. Patient has A blood antigens and Rh. What is his blood type?
A. A positive
B. A negative
C. AB negative
D. O negative
E. O positive
Ans: A
Tips: blood group A antigens positive means, A group with RhD positive. Group A antigens negative
means RhD negative. Universal donor “O with negative”. Whereas O with positive is common. Blood
group O has neither A nor B antigens in plasma, but both A and B antibodies are present in plasma.

50. Important characteristics regarding the blood formed elements may include:
I. Red Blood Cells (RBC) is originated from erythroblast and has a life span of 120 days (4
months)
II. Platelets are involved in blood clot and have a life span is 7 to 10 days
III. Platelets are originated from megakariocyts
A. I only
B. III only

Copyright © 2000-2020 TIPS Inc. Unauthorized reproduction of this manual is prohibited. This manual is being used during
review sessions conducted by PharmacyPrep.
11-11
Blood and Anemia

C. I and II only
D. II and III only
E. All are correct
Ans: E
Tips: Blood formed elements are erythrocytes, leukocytes and platelets. Erythrocytes also known as Red
Blood Cells form 2% of the whole blood, has a half-life of approximately 120 days and play important
rules in the O2 and CO2 transportation. Platelets are originated from megakariocytes and are involved in
blood clotting.

51. The blood weighs 7 to 8% of the total body weight of a health adult. How much of these
percentages are represented approximately in liter of blood?
A. 4 to 6 L
B. 7 to10 L
C. 7 to 8 L
D. 10 L
E. 15 L
Ans: A
Tips: The blood is composed of 90% water and 8% plasma protein. A health adult has
approximately 5 L of blood circulating in the body.

52. Anemia due to iron deficiency includes all, EXCEPT:


A. Peptic ulcer
B. Pregnancy
C. Menstruation
D. GERD
E. NSAID associated GI bleeding
Ans: D

53. Which of the following is not related to lymphocytes?


A. Lymph nodes
B. Thymus gland
C. Thoracic duct
D. Spleen
E. Thyroid gland
Ans: E
Tips: Thyroid gland is not part of lymphocytes. Lymphocytes include: Tonsils and adenoid gland,
thymus, lymph nodes, spleen, bone marrow and lymphatic vessels.

Copyright © 2000-2020 TIPS Inc. Unauthorized reproduction of this manual is prohibited. This manual is being used during
review sessions conducted by PharmacyPrep.
11-12
Biochemistry

PHARMACY PREP
BIOCHEMISTRY

1. Proteases are?
A. Carbohydrate metabolizing enzymes
B. Digestive tract enzymes
C. Protein metabolizing enzymes
D. Nucleic acid metabolizing enzymes
E. Catabolism enzymes
Ans: C
Tips: Any enzyme that breaks down protein into its building blocks, amino acids, is called
a protease, which is a general term. Your digestive tract produces a number of these enzymes,
but the three main proteases are pepsin, trypsin and chymotrypsin.

2. What increases in burns?


A. Albumin
B. Globulins
C. Bilirubin
D. Platelets
E. Red blood cells
Ans: B
Tips: Albumin decreases and globulin increases in burns. Antibodies are types of globulins
proteins.

Plasma proteins concentration that changes with some conditions


Conditions Albumin (60% a major Alpha 1 acid
protein) glycoprotein
Renal Failure ↓ (hypoalbuminemia) ↑
Hepatic Failure ↓ ↑
Arthritis - ↑
BURNS ↓ -
Pregnancy ↓ -
Stress/Trauma ↓ ↑

3. Glycine reaction is normally mediated by:


A. Acetyl Coenzyme A
B. UDP Glucoronyl tranferase
C. PAPS- Phosphoadenosine-5-Phosphosulfate
D. Sulfotranferase
E. GST-Glutathione S-Transferase
Ans: A
Tips: Glycine reaction is a reaction mediated by acetyl Coenzyme A.

Copyright © 2000-2020 TIPS Inc. Unauthorized reproduction of this manual is prohibited. This manual is being used during
review sessions conducted by PharmacyPrep.
12-1
Biochemistry

4. Abundant element in the interstitial fluid may include:


A. Potassium
B. Sodium
C. Magnesium
D. Calcium
E. Chlorine
Ans: B
Tips: Interstitial fluid is situated between parts or in the interspaces of a tissue therefore outside
the cells. Sodium is the most abundant element outside the cells therefore in the interstitial
fluid.
Strategic thinking:
1. Read the question carefully. Make sense of what is being asked.
2. In answering this type of question, refrain from being carried away by the
shortness of the question. Contain the excitement. As in answering other questions,
look for your CLUES and KEYWORDS. Here, clue is INSTERSTITAL. You may easily
confuse it with INSIDE, so be careful. Interstitial refers to spaces. Meaning,
question is asking for the element abundant OUTSIDE the cell. Remember you
Mnemonic (for elements intra- and extracellular): PISO (Potassium In, Sodium Out)
3. Thus, the correct answer is B.

5. Which is responsible for nitric oxide (NO) in vivo?


A. Arginine
B. Tryptophan
C. Guanine
D. Thiamine
E. Leucine
Ans: A
Tips: Arginine is an essential amino acid responsible for nitric oxide (NO).

6. Enzyme that enables DNA fragments from different sources to be joined?


A. DNA polymerase
B. DNA gyrase
C. DNA ligase
D. RNA transferase
E. None of the above
Ans: C
Tips: DNA ligase is an enzyme that enables DNA fragments from different sources to be joined.

7. Glycogenolysis is?
A. Glucose to glycogen
B. Glycogen to glucose
C. Fat and proteins to glucose
D. Glucose to CO 2 and H 2 O
Ans: B
Copyright © 2000-2020 TIPS Inc. Unauthorized reproduction of this manual is prohibited. This manual is being used during
review sessions conducted by PharmacyPrep.
12-2
Biochemistry

Strategic thinking:
1. Read the question carefully.
2. In biochemistry, there are a number of common and suffixes attached to at
beginning or at the end the rootword respectively to quickly define a process. An-,
Cat-, –genesis, -lysis, etc. So, as in Anatomy, it is important to be familiar with
these affixes.so if question is simply asking you to define a word, first find the
rootword. In this case, Glycogen, a polysaccharide. Then, look for the affix. Here,
it’s a suffix –lysis, which refers to the disintegration of a compound. Putting them
together, answer should match a definition from glycogen to a simpler unit, like
glucose.
3. Thus, the correct answer is B.

8. Gluconeogenesis is?
A. Glucose to glycogen
B. Glycogen to glucose
C. Fat and proteins to glucose
D. Glucose to CO2 and H2O
Ans: C
Tips: Fats & proteins break down to glucose is gluconeogenesis.

9. Which of the following vitamin is not water-soluble?


A. Vitamin B
B. Vitamin C
C. Vitamin D
D. Folic acid
E. Riboflavin
Ans: C
Tips: Vitamin A, D, E and K are fat-soluble vitamins.

10. The most abundant metal ion in human body


A. Iron
B. Magnesium
C. Aluminum
D. Phosphorous
E. Potassium
Ans: A
Tips: Iron is the most abundant metal ion in human body.

11. Which of the following is considered the normal potassium serum in human body?
A. 2.0 to 3.5 mEq/L
B. 3.5 to 5.0 mEq/L
C. 5.0 to 7.5 mEq/L
D. 3.5 to 7.5 mEq/L
E. 3.0 to 6.0 mEq/L
Copyright © 2000-2020 TIPS Inc. Unauthorized reproduction of this manual is prohibited. This manual is being used during
review sessions conducted by PharmacyPrep.
12-3
Biochemistry

Ans: B
Tips: Normal potassium serum in human body range between 3.5 to 5.0mEq/L.

12. Which of the following is considered normal sodium serum in human body?
A. 35 to 47 mEq/L
B. 147 to 150 mEq/L
C. 135 to 147mEq/L
D. 75 to 135 mEq/L
E. 125 to 145 mEq/L
Ans: C
Tips: Normal sodium serum in human body range between 135 to 147mEq/L

13. Which of the following is considered as essential vitamin for breast-feed babies?
A. Vitamin A
B. Vitamin B
C. Vitamin C
D. Vitamin D
E. Vitamim E
Ans: D
Tips: The most essential vitamin for breastfed babies is vitamin D.

14. Which of the following hormone acts on cell membrane?


A. Insulin
B. Prolactin
C. Cortisol
D. Estrogen
E. Alanine
Ans: A
Tips: Insulin is a hormone that acts on cell membrane

15. Which of the following vitamins has tetrahydrofolate coenzyme activity based on its pteridine
ring?
A. Thiamine
B. Vitamin D
C. Vitamin K
D. Folic acid
E. Pyridoxine
Ans: D
Tips: Folic acid is a vitamin B9 that has tetrahydrofolate coenzyme activity based on its pteridine
ring.

16. What is correct regarding the formation of proteins?


A. Formed by condensation of peptide bond of amino acids.
B. Geometric sequence of amino acids
Copyright © 2000-2020 TIPS Inc. Unauthorized reproduction of this manual is prohibited. This manual is being used during
review sessions conducted by PharmacyPrep.
12-4
Biochemistry

C. The formation occurs by complexation of amino acids


D. Proteins are formed from carbohydrates
E. None of the above is right
Ans: A
Tips: Proteins are formed by a reaction between amino acids called condensation between
peptide bonds.

17. Which is the strongest endogenous pain producer in human body?


A. Leukotriene
B. Enkephalin
C. Bradykinin
D. Cytokine
E. Angiotensin
Ans: C
Tips: Bradykinin is the strongest endogenous analgesic-pain producer in human body.

18. Which of the following is considered the FIRST precursor of vitamin A formation?
A. Retinoic acid
B. Carotene
C. Cobalt
D. Calciferol
E. Pantothenic acid
Ans: B
Tips: Vitamin A is formed from retinoic acid that is formed from beta-carotene.

19. Biological catalysts responsible for supporting almost all of the chemical reactions that
maintain human life process:
A. Nucleic acid
B. Amino acids
C. Carbohydrates
D. Oligopeptides
E. Enzymes
Ans: E
Tips: Enzymes are biological catalysts responsible for supporting almost all of the chemical
reactions that maintain human life process, and accelerate reactions by lowering the energy of
the transition state.

20. Which of the following statements is/are correct regarding enzymes?


I. Enzymes are proteins
II. Enzymes are catalyst because they are never altered during a reaction
III. Michaelis-Menten theory describe the enzymatic reactions
A. I only
B. III only
C. I and II only
Copyright © 2000-2020 TIPS Inc. Unauthorized reproduction of this manual is prohibited. This manual is being used during
review sessions conducted by PharmacyPrep.
12-5
Biochemistry

D. II and III only


E. All are correct
Ans: E
Tips: Enzymes are proteins or biological catalysts responsible for supporting almost all of the chemical
reactions. They are called catalyst because are never altered during a reaction. Michaelis-Menten theory
describes the enzymatic reactions.

21. Zwitter ion reacts with a substrate as:


A. Neutral ion
B. Proton acceptor ion
C. Proton donor ion
D. Anionic
E. Cationic
Ans: A
Tips: Zwitter ion is a neutral ion with two charges, positive and negative, that reacts neutrally
with a substrate

22. Creatinine is excreted in the urine and increased in kidney failure because filtration is
decreased. Creatinine is formed from?
A. Creatine phosphate
B. Glycerol
C. Phosphates CPK muscle
D. Proteins Creatine -------------- Creatine phosphate ----------- creatinine
E. Transferases ATPADP
Ans: A

23. Which pyrimidine base is not present in DNA?


A. Thiamine
B. Adenine
C. Uracil
D. Cytosine
E. Guanine
Ans: C
Tips: Uracil is only present in RNAs.

24. Arginine is precursor of?


A. Serotonin
B. Histamine
C. Nitric oxide
D. Epinephrine
E. Tryptophan
Ans: C

25. Shaped network present in the cell between nucleus and cell wall:
A. Ribossomes
Copyright © 2000-2020 TIPS Inc. Unauthorized reproduction of this manual is prohibited. This manual is being used during
review sessions conducted by PharmacyPrep.
12-6
Biochemistry

B. Endoplasmic reticulum
C. Mitocondria
D. Cell membrane
E. Nucleotide
Ans: B
Tips: Endoplasmic reticulum is a shaped network present in the cell between nucleus and cell
wall.

26. The Michaelis-Menten equation-Enzymatic reactions will appears ZERO order when:
A. The substrate concentration is much smaller than K m
B. When K m is much smaller than the substrate concentration
C. When V max is much smaller than K m
D. When V max is much larger than K m
E. When K m approaches V max
Ans: E
Tips: The Michaelis-Menten equation-Enzymatic reactions will appear ZERO order when Km approaches
Vmax and FIRST order when the substrate concentration is smaller than Km.

27. Tryptophan is a precursor of?


A. Nitric oxide Tryptophan  Serotonin
B. Serotonin Arginine  nitric oxide
C. Hemoglobin Glycine  hemoglobin & myoglobin
D. Myoglobin Tyrosine  epinephrine
E. Cytochrome oxidase
Ans: B

28. In Ketoacidosis, all of the following are found in excess amount in the urine, EXCEPT:
A. Lactic acid
B. Keto bodies
C. Acetone
D. Acetoacetate
E. Beta hydroxy butyrate
Ans: A

29. Hydrolysis of fixed oils may yield:


A. Saturated + Unsaturated acids
B. Glycerol + Fatty acids
C. Glucose + Fatty acids
D. Fatty acids + Triglycerides
E. Galactose + sphingosine
Ans: B
Tips: Glycerol and fatty acids are the end products of hydrolysis of fixed oils.
30. Example of compound that is phospholipid includes:
A. Glycogen
Copyright © 2000-2020 TIPS Inc. Unauthorized reproduction of this manual is prohibited. This manual is being used during
review sessions conducted by PharmacyPrep.
12-7
Biochemistry

B. Leucine
C. Lecithin
D. Casein
E. Phenylalanine
Ans: C
Tips:
Lecithin = Lipid + phosphoric acid
Casein = proteins + phosphoric acid

31. In an enzymatic reaction, addition of competitive inhibitor leads to:


A. Increase the rate of reaction
B. Decrease the rate of reaction
C. Same rate of reaction
D. Decrease rate and increase substrate concentration
E. Decrease both rate and substrate concentration
Ans: C
Tips: Addition of competitive inhibitor in an enzymatic reaction does not alter the rate of
reaction.

32. Which of the following is the end product of ANAEROBIC glycolysis?


A. Pyruvic acid
B. Urea
C. Uric acid
D. Lactic acid
E. Proteins
Ans: D
(aerobic)
Glucose Pyruvate CO 2 + H20

(anaerobic)

Lactate (lactic acid)

33. Which of the following is the intermediate product of AEROBIC reactions?


A. Pyruvic acid
B. CO 2 AEROBIC  PYRUVATE
C. CO 2 + H 2 O ANAEROBIC  LACTATE
D. Lactic acid
E. Proteins
Ans: A
Tips: CO 2 and H 2 O are the end products of aerobic reactions.

Copyright © 2000-2020 TIPS Inc. Unauthorized reproduction of this manual is prohibited. This manual is being used during
review sessions conducted by PharmacyPrep.
12-8
Biochemistry

34. Linolenic acid is type of essential fatty acid?


15 12 9 1
COOH

Alpha-Linolenic Acid (omega-3)


A. Omega 3
B. Omega 6
C. Omega 9
D. Eicospentaenoic acid (EPA)
E. Decosahexaenoic acid (DHA)
Ans: A
Tips: Lenic = omega 3 and Leic = omega 6

35. Which of the following vitamins deficiencies can occur in patient using low protein diet?
A. Vitamin A
B. Vitamin B 1 Proteins  Tryptophan  Niacin (vitamin B3)
C. Vitamin B 3
D. Vitamin B 12
E. Vitamin C
Ans: C

36. All of the following can cause denaturation of proteins, EXCEPT:


A. Acids
B. Heating
C. Bases
D. Mechanical mixing
E. Buffers
Ans: E
Tips: Buffers are safe to prepare proteins preps such as insulins are prepared in buffers.

37. False statement regarding vitamin D 3 metabolism include:


A. Vitamin C facilitates absorption of vitamin D 3 in the liver.
B. Hydroxylation of D 3 is in kidney.
C. Hydroxylation of is D 2 in the liver.
D. Parathyroid hormone activates metabolism of vitamin D 3 in the kidney.
E. Skin activates synthesis of vitamin D by sunlight.
Ans: A
Tips: Vitamin C does not interfere with absorption of vitamin D3 in the liver.

38. What is the end product of carbohydrates metabolism?


A. Lactose
B. Sucrose
Copyright © 2000-2020 TIPS Inc. Unauthorized reproduction of this manual is prohibited. This manual is being used during
review sessions conducted by PharmacyPrep.
12-9
Biochemistry

C. Maltose
D. Fructose
E. Glucose
Ans: E

39. What is a primary protein structure?


A. Linear Sequence Of Amino Acids
B. Alpha helix
C. Beta pleated sheet
D. Protein consisting more than one protein chain
E. Three-dimensional structure
Ans: A

40. The protein three-dimensional structure and linked with disulphide bonds is?
A. Primary
B. Secondary
C. Tertiary
D. Quaternary
Ans: C

41. Which of the following protein structure is the linear chain of amino acids?
A. Primary
B. Secondary
C. Tertiary
D. Quaternary
E. Alpha helix
Ans: A

42. Which of the following protein structure describes more than one amino acid chain?
A. Primary
B. Secondary
C. Tertiary
D. Quaternary
E. Alpha helix
Ans: D

43. What amino acid is precursor for synthesis of –CH2- in tetrapyrrole structure of porphyrin ring
in hemoglobin?
A. Alanine
B. Tyrosine
C. Glycine
D. Phenyl alanine
E. Acetyl CoA
Ans: C
Copyright © 2000-2020 TIPS Inc. Unauthorized reproduction of this manual is prohibited. This manual is being used during
review sessions conducted by PharmacyPrep.
12-10
Biochemistry

44. Heme containing enzymes are formed from protoporphyrin. What are the examples of heme
containing enzymes or proteins?
A. Hemoglobin
B. Myoglobin
C. Cytochrome oxidase
D. All of the above
Ans: D

45. Which of the following is the rate limiting step in atherosclerosis formation?
A. HMG-COA to mevalonate formation
B. Mevalonate to cholesterol formation
C. Low density lipoprotein formation'
D. Lipid synthesis
Ans: A

46. What is present in cerebroside?


A. Mucin
B. Casein
C. Sphingosine
D. Lipids
E. Phospholipid
Ans: C
Tips: Cerebroside = carbohydrate + sphingosine

47. Essential amino acid present in infants?


A. Arginine
B. Glycine
C. Phenylalanine
D. Tryptophan
Ans: A
Tips: histidine and arginine are present in infants.
Histidine  histamine

48. What amino acid is present in porphyrin ring?


A. Arginine
B. Glycine
C. Phenylalanine
D. Tryptophan
E. Valine
Ans: B

49. Which of the following plasma protein plays role in disease resistance?
A. Globulin
B. Albumin
Copyright © 2000-2020 TIPS Inc. Unauthorized reproduction of this manual is prohibited. This manual is being used during
review sessions conducted by PharmacyPrep.
12-11
Biochemistry

C. Hemoglobin
D. Myoglobin
E. Fibrinogen
Ans: A
Tips: globulin act as immune protein in plasma

50. Which disaccharide is abundant in the body?


A. Lactose
B. Fructose
C. Dextrose
D. Mannose
E. Glucose
Ans: A

51. Lecithin is?


A. Protein
B. Carbohydrate
C. Hydrocarbon
D. Phospholipid
E. Glycolipid
Ans: D

53. Choose correct answer about linolenic acid and linoleic acid:
A. Linolenic acid is should be from dietary sources
B. Linolenic acid and linoleic acid have only difference of double bond in chemical structure.
C. Linolenic acid is omega 3 essential fatty acid
D. Linoleic acid omega 6 essential fatty acid
E. All of the above
Ans: E
Tips: Linolenic is omega 3 essential fatty acids. Omega 3 has three double bonds at position, 9,
12 and 15. Whereas omega 6 has two double bonds at position 9 and 12.

52. Which is NOT a natural fiber?


A. Starch
B. Pectin
C. Lignin
D. Cellulose
E. Chitin
Ans: E
Soluble fibers Insoluble fiber
Gelatin, fruits, vegetables, legumes, Wheat bran, grains, nuts and seeds
seaweeds
Tips: Chitin is fibrous substance consisting of polysaccharides and forming the major constituent
of exoskeleton arthropods and cell wall of fungi.
Copyright © 2000-2020 TIPS Inc. Unauthorized reproduction of this manual is prohibited. This manual is being used during
review sessions conducted by PharmacyPrep.
12-12
Biochemistry

53. All contain glucans, EXCEPT:


A. Bone
B. Neurons
C. Muscle
D. Fungi cell wall
E. Bacteria cell wall
Ans: A
Tips: glucans are fungal glycoproteins. A glucan molecule is a polysaccharide of D-glucose
monomers, linked by glycosidic bonds. Many beta-glucans are medically important. They
represent a drug target for antifungal medications of the echinocandin class.

54. Amino acids that undergo dimerization?


A. Serine
B. Cysteine
C. Threonine
D. Glycine
E. Hemoglobin
Ans: B
Tips: Cysteine form dimer and attached with disulfide bond (S-S)

55. Sphingosine is?


A. Protein
B. Carbohydrate
C. Hydrocarbon
D. Phospholipid
E. Glycolipid
Ans: E

56. Alpha-lipoic acid?


A. Improve functions of neurons in diabetic patient
B. Potent antioxidants
C. Potentiate vitamin E and Vitamin C
D. All of the above
Ans. D
Tips: Lipoic acid or alpha lipoic acid (ALA) and thioctic acid is an organosulfar compound derived
from caprylic acid (octanoic acid) is made in body and is essential for aerobic metabolism. It is
marketed as antioxidant.

57. Kinase enzymes are considered:


A. Lyase
B. Hydroxylase
C. Hydrolase
D. Transferase
E. Oxidoreductase
Copyright © 2000-2020 TIPS Inc. Unauthorized reproduction of this manual is prohibited. This manual is being used during
review sessions conducted by PharmacyPrep.
12-13
Biochemistry

Ans. D
Tips: Kinase is an enzyme that catalyzes the transfer of phosphate group from high energy
phosphate-donating molecule to specific substrates. This process also known as
phosphorylation.

Copyright © 2000-2020 TIPS Inc. Unauthorized reproduction of this manual is prohibited. This manual is being used during
review sessions conducted by PharmacyPrep.
12-14
Nutrition

PHARMACY PREP
NUTRITION

1. Precursor of vitamin A:
A. Carotenoids
B. Cholesterol
C. Tryptophan
D. Pyridoxine
E. Eicosanoids
Ans: A

2. What is incorrect about vitamins deficiency?


A. Vitamin A deficiency cause night blindness
B. Vitamin B1 deficiency cause beriberi
C. Vitamin B3 deficiency cause pellagra
D. Vitamin C deficiency cause scurvy
E. Vitamin K deficiency cause clotting of blood
Ans: E
Vitamin A Night blindness
Vitamin B 1 Beriberi (nervousness, paralysis of muscles)
Vitamin B 2 Ariboflavinosis
Vitamin B 3 Pellagra
Vitamin B 5 Paresthesia
Vitamin B 6 Sideroblastic anemia
Vitamin B 12 Megaloblastic anemia
Vitamin C Scurvy, swelling gum
Vitamin D Rickets & osteomalacia
Vitamin K Non-clotting of blood or bleeding

Strategic thinking:
1. Read the question.
2. In Nutrition Chapter, many questions will be about excess and deficiencies
of vitamins and minerals and their accompanying symptoms. First, know
the FUNCTION of the vitamins. Being familiar with these including their
important sources can benefit you a whole lot. From there, look for your
clue. A deficiency or excess. If deficient, choose an option that is OPPOSITE
of its function. In this scenario, Vitamin K as you learned in needed in
normal blood clotting. Thus deficiency of this will lead to bleeding as
explained in the table above.
3. Thus, the correct answer is E.

Copyright © 2000-2020 TIPS Inc. Unauthorized reproduction of this manual is prohibited. This manual is being
used during review sessions conducted by PharmacyPrep.
13-1
Nutrition

3. Rhodopsin is eye pigmentation present in rods photoreceptor. It is transformed from


the following conversion of 11-retinal to produce night vision to what compound?
A. 11-Cis-retinal to 11-trans retinal
B. 11-Trans retinal to 11-cis retinal
C. 13-Cis retinal to 13- trans retinal
D. 13 Trans retinal to trans retinal
E. 12 cis retinoic acid to trans retinoic acid
Ans: A

4. If mother cannot breast feed an infant 3 months old, what is the closest nutrition
recommended?
A. Cow milk-based formula
B. Soy milk-based formula
C. Lactose free formula
D. Condensed milk formula
Ans: A

5. What formula milk product is suitable for infants 1 to 6 months of age?


A. Cow milk based formula
B. Cow milk based formula fortified with iron
C. Cow milk
D. Solid food is good
Ans: B

6. Topical acne treatment tretinoin (Retin A) is:


I. 13-cis retinoic acid
II. 12-isomer of retinoic acid
III. 13-trans retinoic acid
A. I only
B. III only
C. I and II only
D. II and III only
E. I, II and III
Ans: B
Tips: Topical Tretinoin is 13-Trans retinoic acid (RetinA) TTT; Oral isotretinoin is 13-cis
retinoic acid (Accutane)

7. Which of the following drugs that cause vitamin B 6 deficiency:


A. Isoniazid
B. Penicillamine
C. Concomitant use of Cholestyramine
D. All of the above
Ans: D
Tips: Penicillamine, Isoniazid, and concomitant use of cholestyramine may cause vitamin B 6
deficiency.

Copyright © 2000-2020 TIPS Inc. Unauthorized reproduction of this manual is prohibited. This manual is being
used during review sessions conducted by PharmacyPrep.
13-2
Nutrition

8. Which of the following should be avoided with vitamin B 6 ?


A. Isoniazid
B. Penicillamine
C. Levodopa
D. Dopamine
E. Carbidopa
Ans: C
Tips: Levodopa peripheral conversion to dopamine is increased by vitamin B6 thus the
dopamine increase risk of nausea and vomiting.

9. Which of the following vitamin decreases in chronic alcoholics?


A. Vitamin A
B. Thiamine
C. Vitamin C
D. Vitamin B 12
E. Vitamin E
Ans: B

10. Which of the following vitamin should you recommend for smokers?
A. Vitamin A
B. Vitamin B
C. Vitamin C
D. Vitamin D
E. Vitamin E
Ans: C

11. Which vitamin structure is related to steroidal hormone structures?


A. vitamin A
B. vitamin B
C. Vitamin D
D. Vitamin K
E. Vitamin E
Ans: C

12. Ergocalciferol is:


A. Vitamin C
B. Vitamin D2
C. Vitamin D3
D. Vitamin E
E. Vitamin K
Ans: B
13. Cholecalciferol:
A. Vitamin C
B. Vitamin D 2

Copyright © 2000-2020 TIPS Inc. Unauthorized reproduction of this manual is prohibited. This manual is being
used during review sessions conducted by PharmacyPrep.
13-3
Nutrition

C. Vitamin D 3
D. Vitamin E
E. Vitamin K
Ans: C

14. Vitamin:
A. It is cofactor
B. It is inorganic substance
C. It is catalyst
D. It is organic substance
E. It is enzyme
Ans: A
Tips: co-factors are non-proteins chemicals compound that is required for proteins
biological activity.

15. Chemical ring structure in folic acid that binds to para aminobenzoic acid is;
A. PABA ring
B. Pteridine ring
C. Glutamate
D. Quinolone ring
E. Epoxide ring
Ans: B

16. Seniors age over 70 year old have deficiency of vitamins?


A. Vitamin A
B. Vitamin B 12
C. Vitamin C
D. Vitamin D
E. Vitamin E
Ans: B
Tips: Vitamin B 12 absorption decreases in elderly ages due to decrease in intrinsic factor.
Vitamin B12 IM/IV injections are used to treat deficiency.

17. What vitamin overdose could cause toxicity?


A. Vitamin A
B. Vitamin B
C. Vitamin C
D. Vitamin D
E. Vitamin E
Ans: A

18. Vegetarians have deficiency of vitamin?


A. Vitamin A
B. Vitamin B 12

Copyright © 2000-2020 TIPS Inc. Unauthorized reproduction of this manual is prohibited. This manual is being
used during review sessions conducted by PharmacyPrep.
13-4
Nutrition

C. Vitamin C
D. Vitamin D
E. Vitamin E
Ans: B

19. A breast fed infant should recieved which of the following vitamin drops?
A. Vitamin A drops
B. Vitamin B 12
C. Vitamin C
D. Vitamin D drops
E. Vitamin E drops
Ans: D

20. What reaction converts beta carotene to vitamin A?


A. Oxidative cleavage
B. Reduction
C. Hydrolysis
D. Esterification
E. Cytochrome
Ans: A

 Vitamin A
Oxidation

21. Peripheral neuropathy is caused by the deficiency of?


A. Vitamin A
B. Vitamin B1
C. Vitamin C
D. Vitamin B6
E. Vitamin D
Ans: D

22. Chronic renal disease can cause deficiency of?


A. Vitamin D 2
B. Vitamin B 12
C. Vitamin C
D. Vitamin D 3
E. Vitamin D
Ans: D
Tips: Chronic renal deficiency can cause deficiency of vitamin D 3 i.e 1, 25 dihydroxy
ergocalceferol which is active vitamin D.

Copyright © 2000-2020 TIPS Inc. Unauthorized reproduction of this manual is prohibited. This manual is being
used during review sessions conducted by PharmacyPrep.
13-5
Nutrition

23. Which of the following condition have deficiency of vitamin B 1 (thiamine)


A. smoking
B. old age
C. chronic alcoholism
D. Infants
E. Pregnancy
Ans: C

24. Vitamin B 12 deficiency is more common in?


A. Smoking
B. Old age
C. Chronic alcoholism
D. Infants
E. Pregnancy
Ans: B

25. What is incorrect combination about vitamin deficiencies in:


A. Smokers  Vitamin C
B. Seniors  vitamin B 12
C. Chronic alcoholics  Vitamin B 1
D. Infants (on breast milk)  vitamin D
E. Chronic renal failure  Vitamin E
Ans: E

26. What are essential fatty acids?


A. Linoleic acid (omega-6) and Linolenic acid (omega-3)
B. Arachidonic acid and linoleic acid
C. Cholesterol and arachidonic acid
D. Saturated fats and polyunsaturated fats
E. Saturated fats and monounsaturated fats
Ans: A

27. What is omega-6 essential fatty acid called?


A. Linoleic acid
B. Linolenic acid
C. Arachidonic acid
D. Transfat
E. Vegetable fats
Ans: A

28. What fats are the safe or the best fats to use?
A. Trans fats
B. Saturated fats
C. Poly unsaturated essential fats

Copyright © 2000-2020 TIPS Inc. Unauthorized reproduction of this manual is prohibited. This manual is being
used during review sessions conducted by PharmacyPrep.
13-6
Nutrition

D. Animal fats
E. Vegetable fats
Ans: C
Tips: unsaturated or mono saturated fats are recommended.

29. A customer of your pharmacy is allergic to gluten, which of the following should not
recommend?
A. Milk
B. Rice
C. Wheat
D. Wine
E. Eggs
Ans: C

30. A person have milk allergy, which may be due to?


A. Protein
B. Gluten
C. Enzyme
D. vitamins
E. None of the above
Ans: A

31. What is incorrect about calcium carbonate supplements?


A. Calcium carbonate, take it with meals. This type of supplement is absorbed best with
food.
B. Take no more than 500-600 mg of calcium at a time.
C. If you take both iron and calcium supplements, do not take them at the same
time. This is because calcium affects your body’s ability to absorb iron.
D. If milk allergy, there can cross allergy to calcium supplement, so calcium carbonate
should be avoided in milk allergy.
E. Calcium carbonate regimen should taken with food 2 to 4 times a day
Ans: D
Tips: Ref: Eatrightontario.ca

32. Folic acid structure consists of?


A. Methotrexate and PABA and glutamate
B. Pteridyl- PABA -glutamate
C. PABA-pteridyl- glutamate
D. Pteridyl-glutamate-PABA
Ans: B

33. Excessive dose of Niacin can be associated will all, EXCEPT:


A. Facial flushing
B. Hyperglycemia (diabetes)

Copyright © 2000-2020 TIPS Inc. Unauthorized reproduction of this manual is prohibited. This manual is being
used during review sessions conducted by PharmacyPrep.
13-7
Nutrition

C. Hyperuricemia (Gout)
D. Pellagra
Ans: D
Tips: Pellagra caused by the deficiency of Niacin. (Vitamin B3).

34. Essential amino acid deficiency can cause?


A. Deficiency of protein
B. Deficiency of vitamins
C. Deficiency of nucleic acids
D. No change in primary metabolites
E. All of the above
Ans: A

35. Rhodopsin is essential for vision. Its chemical structure is related to?
A. Eicosanoids
B. Steroid hormone
C. Carotenoids
D. Omega fats
E. Saturated fat
Ans: C

36. Which of the following fats in the diet has the greatest negative influence on blood
cholesterol?
A. Polyunsaturated fats
B. Saturated fats
C. Cholesterol
D. Omega 3
E. Omega 6
Ans: B

37. Beta carotene is precursor of?


A. Retinol
B. Retinoic acid
C. Vitamin A
D. Retinal
E. All of the above
Ans: E

38. Which of the following has the highest glycemic index (GI)?
A. Whole Grain
B. Vegetables
C. Fruits Like Apple
D. White bread
E. Green beans

Copyright © 2000-2020 TIPS Inc. Unauthorized reproduction of this manual is prohibited. This manual is being
used during review sessions conducted by PharmacyPrep.
13-8
Nutrition

Ans: D
Tips: The glycemic index is relative ranking of carbohydrate in foods according to how
they affect blood glucose levels. Carbohydrates with low GI value (55 or less) are more
slowly digested, absorbed and metabolized and cause a lower and slower rise in blood
levels.

39. What is the closes option to breast milk for infant at age of 3 months?
A. Formula milk
B. Cow based formula milk
C. Soy based formula milk
D. Lactose free formula milk
E. Cow milk
Ans: B

40. What is correct about iron absorption?


A. Iron is transported across the intestinal mucosa by active transport.
B. It is absorbed as ferrous iron and is converted to ferric iron within the mucosal cell.
C. Iron is absorbed better on empty stomach
D. Ferrous gluconate has highest absorption
E. All of the above
Ans: E
Tips: Iron is transported across the intestinal mucosa by active transport. It is absorbed
as ferrous iron and is converted to ferric iron within the mucosal cell.

41. Vitamin K clotting factors are formed in?


A. Blood
B. Liver
C. Colon
D. Skin
E. Renal
Ans: B

42. Where does warfarin an oral anticoagulation mechanism action takes place?
A. Blood
B. Liver
C. Colon
D. Skin
E. Renal
Ans: B

43. Inadequate intake of vitamin D over 50 year’s age can cause?


A. Renal disease
B. Osteoporosis
C. Osteoarthritis

Copyright © 2000-2020 TIPS Inc. Unauthorized reproduction of this manual is prohibited. This manual is being
used during review sessions conducted by PharmacyPrep.
13-9
Nutrition

D. Hypercalcemia
E. Hyper vitamin D syndrome
Ans: B

44. Nephrolithiasis caused by?


A. Deficiency of vitamin D
B. Deficiency of Vitamin A
C. Overdose of vitamin A
D. Overdose of vitamin B 12
E. Overdose of vitamin D
Ans: E

45. What is the best source vitamin D


A. Dairy product
B. Milk product
C. Dark green leafy vegetable
D. Sunlight
E. Sunscreen
Ans: D

46. What is the best dietary source vitamin D


A. Vitamin D fortified dairy product
B. Fish product
C. Dark green leafy vegetable
D. Sunlight
E. Sunscreen
Ans: A

47. Functional food examples?


A. Cereals added with multivitamins
B. Orange juice added with vitamin D
C. Formula milk fortified with iron
D. Milk fortified with vitamin D
E. All of the above
Ans: E

48. In chronic renal failure which of the following increase?


A. Vitamin D 3
B. Calcium Renal failure
C. Phosphates ↓ alpha 1 hydroxylase
D. Potassium ↓vitamin D3
E. Alpha1 hydroxylase ↓ Calcium
Ans: C Phosphate increase

Copyright © 2000-2020 TIPS Inc. Unauthorized reproduction of this manual is prohibited. This manual is being
used during review sessions conducted by PharmacyPrep.
13-10
Nutrition

49. All of the following can cause hypocalcemia, EXCEPT:


A. Chronic renal disease
B. Low para thyroid hormone
C. vitamin D deficiency
D. Over dose of vitamin D
E. Inadequate intake of calcium
Ans: D

50. Considering Vitamin D and its precursor molecules, which of the following is found at
the highest concentration in the plasma?
A. Cholecalciferol
B. Ergocalciferol
C. 25-hydroxy vitamin-D
D. 1,25-dihydroxy vitamin-D
E. 24,25-dihydroxy vitamin-D
Ans: D

51. Hypercalcemia is due to excessive calcium in blood. It may be caused by all except?
A. Hyperparathyroidism
B. Paget’s disease
C. Overdose of vitamin C
D. Overdose of vitamin D
Ans: C

52. Which is false about vitamin D metabolism?


A. Skin converts dehydrocholesterol to pre-vitamin by sunlight
B. Hydroxylation at C25 occurs in the liver
C. Hydroxylation at C1 occurs in the kidney
D. Metabolism of vit D in the kidney is controlled by parathormone
E. Vitamin C activates vit D metabolism in the liver
Ans: E
DIET
7-Dehydrocholesterol (Skin)

Cholecalciferol Skin (ultraviolet)

↓ [ Ca2+] Liver
↑ PTH 25-OH-cholecalciferol (Vit. D 2 ) Storage from of vitamin D
↓ [ phosphate]
(+) Kidney

1, 25-(OH) 2 -cholecalciferol 24, 25-(OH) 2 -cholecalciferol


(Active vit. D 3 ) (Inactive)

Copyright © 2000-2020 TIPS Inc. Unauthorized reproduction of this manual is prohibited. This manual is being
used during review sessions conducted by PharmacyPrep.
13-11
Nutrition

53. What is vitamin is unstable upon exposure to heat? Or heating will destroy?
A. Vitamin B12
B. Vitamin B1
C. Vitamin B2
D. Vitamin B3
E. Vitamin B7
Ans: B
Tips: vitamin B1 is the most unstable vitamin upon on heating. Vitamin B2 is the stable
on heating.

54. Institutionalized elderly usually have deficiency of which of the following?


A. Vitamin B1
B. Vitamin C
C. Vitamin D
D. Vitamin A
E. Vitamin K
Ans: C

55. Case of old women who fractured her hip. Why would not test bone mineral density?
A. She is old.
B. She already fractured her hip.
C. She already did surgery.
D. She does not have history of fracture.
E. She is female.
Ans: B

56. How long can breast milk be preserved in the refrigerator?


A) 1 hour
B) 2 hours
C) 24 hours
D) 1 year
E) 1 month
Ans: C
Tips: Breast milk can be preserved in fridge for 24 hours, room temperature for 2 hours
and freezer for 1 year.

57. The parenteral iron injections cause anaphylactic shocks. Which of the following
medications should be made available to counteract anaphylactic-type reactions that
may occur?
A) Epinephrine
B) Deferoxamine
C) Diphenhydramine inj pretreatment
D) Dobutamine
E) Dexamethasone intramuscular

Copyright © 2000-2020 TIPS Inc. Unauthorized reproduction of this manual is prohibited. This manual is being
used during review sessions conducted by PharmacyPrep.
13-12
Nutrition

Ans: A
Tips: Injection site reaction can be managed by prophylactic use of antihistamines. But
anaphylactic reactions should be managed by epinephrine.
CPS. Iron dextran (DexIron) Monograph. The parenteral use of complexes of iron and
carbohydrates has resulted in anaphylactic-type reactions. Deaths associated with such
administration have been reported. Thus prior to receiving the first iron dextran
therapeutic dose, all patients should be given an IV test dose of 0.5mL at a gradual rate
over at least 5 minutes with the remainder of the initial dose administered at least 1
hour has passed. Alternatively, give a 25 mg test dose 1 h before the initial dose.
In both cases, epinephrine should be immediately available in the event of acute
hypersensitivity reactions. The usual adult dose of epinephrine is 0.5mL of a 1:1000
solution by SC or IM.

Copyright © 2000-2020 TIPS Inc. Unauthorized reproduction of this manual is prohibited. This manual is being
used during review sessions conducted by PharmacyPrep.
13-13
Nutrition

Copyright © 2000-2020 TIPS Inc. Unauthorized reproduction of this manual is prohibited. This manual is being
used during review sessions conducted by PharmacyPrep.
13-14
Microbiology
PHARMACY PREP
MICROBIOLOGY
1. Alternating residues B-(1, 4)-linked N-acetyl group glucosamine and N-acetyl muramic acid is
found in?
A. Plasma membrane
B. Cell wall
C. Cytoplasm
D. Mitochondria
Ans: B
Tips: Bacterial cell wall predominantly gram +VE peptidoglycans also known as murein, is
polymer consist of the sugar component with alternating residues of B-(1, 4)-linked N-acetyl
group glucosamine (NAG) and N-acetyl muramic acid (NAM).

2. Which of the following is the correct statement about Staphylococcus aureus?


I. It contains teichoic acid.
II. Gram stain test is positive
III. It contains lipopolysaccharide layer.
A. I only
B. III only
C. I and II only
D. II and III only
E. All are correct
Ans: C

3. Obligate anaerobic bacteria:


I. Generate hydrogen peroxide
II. Generate superoxide dismutase
III. Do not generate superoxide dismutase
A. I only
B. III only
C. I and II only
D. II and III only
E. All are correct
Ans: B
Strategic thinking:
1. Read the question.
2. Look for your clues and keywords. In this case, clue is Obligate, meaning require.
Then, the keyword. In this case, Anaerobe. In the keyword, there might be a clue in
the form of an affix. In the questions, An- meaning absence, absence of oxygen. So,
in light of the question, these bacteria require no oxygenation to live because they
die in the presence of oxygen. Therefore, there is no way they can generate
anything with oxygen.

Copyright © 2000-2020 TIPS Inc. Unauthorized reproduction of this manual is prohibited. This manual is being used during
review sessions conducted by PharmacyPrep.
14-1
Microbiology
3. Thus, the correct answer is B.

4. Colon is large intestine which contains which of the following type of bacteria:
A. 95% to100% anaerobic
B. 95% to 100% aerobic
C. 30 to 50% anaerobic
D. 30 to 50% aerobic
E. There are no bacteria in lower gut
Ans: A

5. A patient was diagnosed with serious infection of sepsis by the Staph. Infections (gram
positive). What are the toxins likely present in blood?
A. Endotoxins
B. Exotoxin
C. No toxin is produced in blood
D. Pyrogens
Ans: B
Tips: Staph infections are gram positive bacteria cell wall produce exotoxins.

6. Which of the following clinical condition is the prophylactic use of anti-bacterial NOT required?
A. Preoperative treatment for GI conditions, tooth surgeries or tooth removed.
B. Frequent upper respiratory infections
C. Preoperative treatment of hip surgeries
D. Prevention of meningitis among individuals in close contact with infected patients
E. opportunistic infections in HIV patient
Ans: B
Tips: Certain clinical situations require the use of antibiotics for prevention rather than
treatment of infections, such as:
Prevention of strep infections in patient with rheumatic heart disease
Patient undergoing dental extraction, who have prosthetic devices such as heart valve etc.
Prevention of Tuberculosis, meningitis (close contacts) Prior to certain surgical procedures.

7. Drugs used in the treatment of bacterial meningitis are all of the following, EXCEPT:
A. Penicillins
B. Cephalosporins
C. Gentamicin
D. Streptomycin
E. Sulfonamides
Ans: D
Tips: Streptomycin is not used in the treatment of meningitis. The main treatment of bacterial
meningitis may include penicillins, cephalosporins and vancomycin while viral meningitis is mainly
treated by acyclovir IV.

8. The most common causative organism of community acquired pneumonia (CAP) is?

Copyright © 2000-2020 TIPS Inc. Unauthorized reproduction of this manual is prohibited. This manual is being used during
review sessions conducted by PharmacyPrep.
14-2
Microbiology
A. S. pneumonia
B. M. pneumonia
C. H. influenza
D. S. aureus
E. E. coli
Ans: A

9. The following organism least likely causes pneumonia:


A. S. pneumonia
B. M. pneumonia
C. H. influenza
D. M. catarrhalis
E. E. coli
Ans: E

10. Infections may occur due to mountain stream?


A. E. coli
B. S. aureus
C. Giardia
D. Yeast infections
E. S. pneumonia
Ans: C
Tips: Giardia is type of protozoa and is found in mountain stream.

11. A patient was diagnosed with gonococci bacterial infection. Which of the following bacteria is
caused by gonococcal infections?
A. Gonorrhea
B. Chlamydia Gonococcal Non-gonococcal
C. E. coli N. gonorrhea Chlamydia
D. Shigella Cefixime 800 mg Azithromycin
Ans: A Ceftriaxone

12. A diabetic patient diagnosed with skin infection cellulitis. The lab test shows gram positive
coagulase positive. Which of the following causative organism?
A. E. coli
B. S. viridans
C. S. pyogenes
D. S. aureus
E. Gonorrhea
Ans: D
Tips: cellulites mainly caused by S. aureus and S. pyogenes. However, the questions coagulase
positive infections, so answer is S. aureus.

Copyright © 2000-2020 TIPS Inc. Unauthorized reproduction of this manual is prohibited. This manual is being used during
review sessions conducted by PharmacyPrep.
14-3
Microbiology
13. Which of the following is the least likely sexually transmitted infections causative
microorganism?
A. Gonorrhea
B. Shigella
C. Syphilis
D. Chlamydia
E. Herpes
Ans: B

14. Which of the following can be sexually transmittable infection?


A. Hepatitis A
B. Hepatitis B
C. Hepatitis c
D. Hepatitis B and C
E. Hepatitis A B C
Ans: D

15. The most common cause of urinary tract infections includes:


A. N. gonnorhea
B. E. coli
C. Chlamydia
D. S. pneumonia
E. M. catarrhalis
Ans: B

16. The most common cause of traveller’s diarrhea is:


A. Protozoa
B. E. coli
C. Salmonella sp.
D. Shigella
E. Corynebacterium
Ans: B

17. Lyme disease a tick born infections commonly occur in the months of July-August and in the
area of Mississippi river valley, is caused by?
A. Treponema pallidum
B. B. burgdorferi
C. Dermatophyte
D. Giardia lamblia
E. E. coli
Ans: B

18. Which of the following is the most common cause of nosocomial infections?
A. S. aureus
Copyright © 2000-2020 TIPS Inc. Unauthorized reproduction of this manual is prohibited. This manual is being used during
review sessions conducted by PharmacyPrep.
14-4
Microbiology
B. P. aeruginosa
C. E. coli
D. S. pneumonia
Ans: B
Tips: nosocomial infections = hospital acquired infections.

19. Drug of choice for nosocomial infection that caused by P. aeruginosa.


A. Ciprofloxacin
B. Aminoglycosides
C. Amoxicillin
D. Erythromycin
E. Cephalosporins
Ans: A
Tips: P. aeruginosa is treated with ciprofloxacin and aminoglycosides. Ciprofloxacin is more
effective than aminoglycosides.

20. Clindamycin, may cause severe diarrhea, this condition can be treated by:
A. Bismuth Subsalicylate
B. Loperamide
C. OTC antidiarrheal
D. Metronidazole
E. Oral rehydration salts
Ans: D
Tips: Clindamycin may cause severe blood diarrhea due Pseudomembranous colitis. The drug of
choice is metronidazole or vancomycin

21. Which of the following is mycolic acid synthesize inhibitor?


A. Rifampin
B. Ethambutol
C. Isoniazid
D. Cotrimoxazole
E. Ketoconazole
Ans: C

22. Syphilis is a sexually transmitted infection that can caused by?


A. E. coli
B. Salmonella
C. T. pallidum
D. Spirochete
E. Diphtheria
Ans: C
Tips: The drug of choice for Syphilis? Penicillin G inj. Or doxycycline
Syphilis is a STI caused by T. pallidum

Copyright © 2000-2020 TIPS Inc. Unauthorized reproduction of this manual is prohibited. This manual is being used during
review sessions conducted by PharmacyPrep.
14-5
Microbiology
23. The fungal infection that commonly occurs on skin between foot toes is:
A. Jack itch
B. Onychomycosis
C. Athletes foot
D. Thrush
E. Ringworm
Ans: C
Tips: Athlete’s foot occurs on skin between foot toes and caused by T. pedis. Jack itch is fungal infections
commonly occurs on foot toe. Onychomycosis is fungal nail infections. Thrush is fungal infections mainly
caused by Candida albicans. Ringworm is a fungal infection, it caused by dermatophytes (it is not a
worm).

24. Toxic shock syndrome (TSS) is caused by:


A. E. coli
B. Gonorrhea
C. S. aureus
D. Syphilis
E. Chlamydia
Ans. C

25. Pseudomembranous colitis is a condition that occurs as result of antibiotics use, is caused by:
A. C. tetani
B. C. difficile
C. C. botulinum
D. C. perfringens
E. All of the above
Ans: B

26. All of the following cause food poisoning, EXCEPT:


A. E. coli
B. Salmonella sp
C. Shigella sp
D. Neisseria
E. C. botulinum
Ans: D

27. What is the drug of choice to treat a sexually transmitted infection Syphilis?
A. Ciprofloxacin
B. Doxycycline
C. Penicillin G inj.
D. Cefuroxime
E. Cephalexin
Ans: C

Copyright © 2000-2020 TIPS Inc. Unauthorized reproduction of this manual is prohibited. This manual is being used during
review sessions conducted by PharmacyPrep.
14-6
Microbiology
28. Cervical cancer is associated with?
A. Herpes viral infection
B. Gonorrhea infection
C. Syphilis infections
D. Papilloma virus
E. H. influenza
Ans: D

29. The common cold (runny nose) is caused by:


A. Influenza A
B. Influenza B
C. Rhinovirus
D. Pox virus
E. Papilloma virus
Ans: C
Tips: Rhino has runny nose

30. Common pathogenic intestinal parasites in mountain streams?


A. E. coli
B. Giardiasis
C. Ascariasis
D. Amebiasis
E. Trichomonas
Ans: B

31. Which of the following NOT associated with Toxic shock syndrome?
A. Tampons
B. Condoms
C. Contraceptive sponges
D. Intra uterine devices (IUD)
E. None of the above
Ans: B

32. Endotoxins are product of?


A. Lipopolysaccharides
B. Teichoic acid
C. Proteins
D. Fats
E. bacterial cell wall
Ans: A
Tips: Gram negative bacteria cell wall produce endotoxins, thus this bacteria cell wall have
lipopolysaccharide.

Copyright © 2000-2020 TIPS Inc. Unauthorized reproduction of this manual is prohibited. This manual is being used during
review sessions conducted by PharmacyPrep.
14-7
Microbiology
33. Which of the following methods are used for removal endotoxins?
A. Lyophilization
B. Rabbit test
C. LAL test
D. Sterilization
E. None of the above
Ans. E
Tips: Distillation method is used for removal of endotoxins.

34. A person with H. pylori positive may have -->


I. Peptic ulcer
II. Ulcerative colitis
III. Crohn’s disease
A. I only
B. III only
C. I and II only
D. II and III only
E. All are correct
Ans: A

35. Which of the following is least detected in meningococcal meningitis?


A. S. pneumonia
B. Neisseria meningitis
C. H. influenza
D. S. aureus
E. E. coli
Ans: D

36. Pneumococcal vaccination prevents all except?


A. Meningitis
B. Otitis media
C. Community acquired pneumonia
D. Osteomyelitis
E. Sinusitis
Ans: D

37. The drug of choice in treatment of Lyme disease?


A. Doxycycline in children
B. Doxycycline adults
C. Amoxicillin in children
D. B and C
Ans: D
Tips: Amoxicillin in the first line children because doxycycline is not used in children under the age of 18
year old. Otherwise amoxicillin is the first line to treat Lyme disease.

Copyright © 2000-2020 TIPS Inc. Unauthorized reproduction of this manual is prohibited. This manual is being used during
review sessions conducted by PharmacyPrep.
14-8
Microbiology
38. Which of the following drug is tuberculosis prophylaxis?
A. Isoniazid (INH)
B. Rifampin
C. Ethambutol
D. Ciprofloxacin
Ans: A

39. Which is true about Chancre disease?


A. Primary symptoms of syphilis infection
B. Painful ulcer at infection site
C. Patient infected with Yersinia pestes
D. Patient infected with urinary tract infections
E. Patient infected with E. coli
Ans: A
Tips: A chancre is a painless genital ulcer most commonly formed during the primary stage of
syphilis. This infections lesions forms approximately 21 days after initial exposure to Treponema
pallidum. The chancre transmit the sexually transmitted infections through direct physical
contact. The chancres may diminish between 4 to 8 weeks without application of medication.
The chancres usually formed around the penis, anus, mouth and vagina. The drug of choice for
syphilis is Penicillin G IM inj.

40. Which of the following symptoms are vaginal candidiasis?


I. Fishy odor discharge
II. Itchy, burning sensation
III. Cottage cheese discharge
A. I only
B. III only
C. I and II only
D. II and III only
E. All are correct
Ans: D
Tips: Vaginal candidiasis presentation symptoms are itchy vaginal area, burning sensation,
cottage cheese discharge. But fishy odor discharge is associated with Trichomonas infections.

41. A patient receiving treatment of gonorrhea infections. Should also get treatment of?
A. Meningitis
B. Chlamydia
C. Syphilis
D. Herpes
E. UTI
Ans: B

42. What infection is more common to newborn in Canada?


A. Meningitis
Copyright © 2000-2020 TIPS Inc. Unauthorized reproduction of this manual is prohibited. This manual is being used during
review sessions conducted by PharmacyPrep.
14-9
Microbiology
B. Chlamydia
C. Syphilis
D. Herpes
E. UTI
Ans: B

43. Chancre or chancroids on genitals are symptoms of?


A. Athletes foot
B. Gonorrhea
C. Syphilis
D. Herpes
E. Warts
Ans: C
Tips: Syphilis infections have (genital ulcers or chancres), single large ulcer, and painless.
Penetrate through broken skin or mucus membrane usually through sexual contact.

44. Which of the following is NOT sexually transmitted that does not require treatment of the
partner?
A. Syphilis
B. Trichomonas
C. Candida
D. Bacterial vaginitis
E. Herpes
Ans: C

45. Which of the following viral strain infection can cause cervical cancer?
A. Herpes
B. Papilloma virus
C. Hepatitis
D. Warts
E. All
Ans; B

46. What is first line treatment for C. difficile diarrhea?


A. Vancomycin IV
B. Metronidazole po
C. Metronidazole iv
D. Ciprofloxacin
Ans: B

47. Which of the following infections require prophylaxis of all close contact?
A. Chlamydia
B. Yeast
C. Athletes foot
Copyright © 2000-2020 TIPS Inc. Unauthorized reproduction of this manual is prohibited. This manual is being used during
review sessions conducted by PharmacyPrep.
14-10
Microbiology
D. Meningitis
E. Salmonella
Ans: D
Tips: rifampicin is used as prophylaxis to prevent meningitis.

48. Cold sore treatment if it reoccurs >6 times a year includes all of the following, EXCEPT:
A. Topical acyclovir
B. Oral acyclovir
C. It heals by itself
D. Topical protectant docosanol
E. Very hard violent surgery
Ans: E

49. What is true about cervical cancer?


I. Caused by oral contraceptives pills
II. Caused by papilloma virus
III. It is sexually transmitted infection
A. I only
B. III only
C. I and II only
D. II and III only
E. All are correct
Ans: D

50. Which of the following infections is NOT related to virus?


A. Prion disease
B. Measles
C. Shingles
D. Keratoconjunctivitis
E. Cold sore
Ans: A
Tips: Prion is a small infectious particle composed of abnormal folded protein. Prion disease.
(Creutzfeldt-Jakob) caused by prions. Or transmissible spongiform encephalitis. It can occur in
human and animals (mad cow disease).

51. MK is a 27-year-old man presents with persistent ear pain for last 2 days and associated with
drainage. Pharmacist refer patient to physician because?
A. Otitis externa caused by P. aeruginosa and S. aureus
B. Because it is associated with drainage
C. Because ruptured tympanic membrane
D. Because ear pain with drainage could be perforation of tympanic membrane or drainage
from middle ear.
E. Because it can cause hearing loss
Ans: D
Copyright © 2000-2020 TIPS Inc. Unauthorized reproduction of this manual is prohibited. This manual is being used during
review sessions conducted by PharmacyPrep.
14-11
Microbiology
52. E. coli is?
A. Gram negative bacilli
B. Gram positive bacilli
C. Gram negative cocci
D. Gram positive cocci
Ans: A

53. Which of the following bacteria can cause chronic peptic ulcer disease?
A. E. coli
B. H. pylori
C. S. aureus
D. S. pneumonia
Ans: B

54. Which of the following organisms is an obligate intracellular bacterium making it resistant to
cell wall antibiotics?
A. E. coli
B. Legionella pneumophilia
C. S. aureus
D. Chlamydophilia pneumonia
Ans: D

Copyright © 2000-2020 TIPS Inc. Unauthorized reproduction of this manual is prohibited. This manual is being used during
review sessions conducted by PharmacyPrep.
14-12
Cell and Molecular Biology

PHARMACY PREP
CELL AND MOLECULAR BIOLOGY

1. Centrosomes?
A. Contain acid hydrolyzing enzyme
B. Are main organizing microtubule center
C. Contain protein (rough) and lipid (smooth) needed for synthesis
D. Are the primary sites for biological protein synthesis.
E. Are enzymes that produce and decompose hydrogen peroxides.
Ans: B
Tips: Centrosomes are organizing microtubule center. Lysosome contains acid
hydrolyzing enzyme. Endoplasmic reticulum contains protein (rough) and lipid (smooth)
synthesis. Ribosomes are primary site for biological protein synthesis. Peroxisomes
contain enzymes that produces and decomposes hydrogen peroxides. Peroxisomes
breakdown long chain fatty acids.

2. Genetic information is stored in?


A. RNA
B. DNA
C. Gene
D. Chromosome
E. Ribosome
Ans: B

3. What base is NOT found in ribonucleic acid (RNA)?


A. Thymine
B. Adenine
C. Guanine
D. Cytosine
E. Uracil
Ans: A

4. The DNA exists as a double helix in which polynucleotide chains consist of a sequence of
nucleotides linked together by phosphodiester bonds, joining adjacent deoxyribose
moieties. The hydrogen bond holds the two polynucleotide strands together of the
bases from the opposing strands, called complementary base-pairing. What is incorrect
about complimentary base-pairing?
A. Adenine (A) is always paired with Thymine (T)
B. Guanine (G) is always paired with Cytosine (C)
C. Adenine (A) is always paired with Cytosine (C)
D. Cytosine (C) is always paired with Guanine (G)
Ans: C
Tips: AT and GC is always compliment

Copyright © 2000-2020 TIPS Inc. Unauthorized reproduction of this manual is prohibited. This manual is being
used during review sessions conducted by PharmacyPrep.
15-1
Cell and Molecular Biology

5. Glycoproteins are conjugated proteins. These proteins are linked to:


A. DNA
B. RNA
C. mRNA
D. Carbohydrates
E. Lipids
Ans: D
Tips: glycoproteins are linked to carbohydrates

6. An enzyme that cleaves deoxyribonucleic acid (DNA) at a specific site is called:


A. A Trypsin
B. Restrictive ribonuclease
C. Restrictive endonuclease
D. Plasmid
E. Reverse transcriptase
Ans: C
Tips: Restrictive endonuclease cleaves the DNA at a specific site.

7. What is incorrect?
A. DNA replication; DNA  DNA
B. Transcription: DNA RNA
C. Translation: RNA Proteins
D. Reverse transcription DNAmRNA
Ans: D

8. Anticodons are present in?


A. DNA
B. mRNA
C. tRNA
D. rRNA
E. All of the above
Ans: C

9. What is true about codon?


A. Present in tRNA
B. Consists of 3 nucleotide base that express one amino acid
C. Segment of DNA express gene
D. Antisense oligonucleotide
Ans: B

10. Which of the following is the highest percent of RNA type?


A. mRNA
B. tRNA
C. cRNA
Copyright © 2000-2020 TIPS Inc. Unauthorized reproduction of this manual is prohibited. This manual is being
used during review sessions conducted by PharmacyPrep.
15-2
Cell and Molecular Biology

D. rRNA
E. dRNA
Ans: D
Tips: rRNA80%, tRNA15%, mRNA5%

11. Which of the following is a recombinant DNA?


A. rDNA
B. R-DNA
C. cDNA
D. mDNA
E. tDNA
Ans: A
Tips: rDNA is recombinant DNA

12. All are related polymer chain reaction (PCR), EXCEPT:


A. DNA magnification
B. Endonuclease
C. DNA polymerase
D. Taq polymerase
E. Thermos aquaticus
Ans: B

13. Hemophilia type A results from the deficiency of clotting factor?


A. Factor 5
B. Factor10
C. Factor 8
D. Factor 2
E. Factor 9
Ans: C

14. In the manufacture of drugs via DNA technology, the DNA material is microscopically
inserted into the:
A. Cytoplasm
B. Nucleus
C. Endoplasmic reticulum
D. Cell wall
E. Cell membrane
Ans: B

15. Which of these supportive agents may have greater risk than benefit in patients with
cancer when the goal is to cure due to the potential increased risk of tumor
progression?
A. Antiemetics
B. Colony Stimulating Factor (CSF)
Copyright © 2000-2020 TIPS Inc. Unauthorized reproduction of this manual is prohibited. This manual is being
used during review sessions conducted by PharmacyPrep.
15-3
Cell and Molecular Biology

C. Corticosteroids
D. Erythropoietin-Stimulating Agent
E. Oprelvekin (IL-11)
Ans: D

16. Bases found in DNA is/are


I. Uracil
II. Cytosine
III. Thymine
A. I only
B. III only
C. I and II only
D. II and III only
E. All are correct
Ans: D

17. What is not a correct complimentary pair?


A. G-C
B. T-A
C. A-G
D. A-U
E. C-G
Ans: C

18. What is codon?


A. Specific set of three bases are codons and these are expressed at transcription
B. Specific set of DNA and RNA
C. Specific set of DNA
D. Specific set of DNA bases
E. Specific set of RNA bases
Ans: A

19. What is cDNA?


A. DNA template produced by mRNA, is complementary DNA (cDNA)
B. Specific set of DNA and RNA
C. Specific set of DNA
D. Specific set of DNA bases
E. Specific set of RNA bases
Ans: A

20. Functions of restrictive endonuclease include?


A. Cut DNA into piece
B. Cut RNA into pieces
C. Cut DNA and RNA into pieces
Copyright © 2000-2020 TIPS Inc. Unauthorized reproduction of this manual is prohibited. This manual is being
used during review sessions conducted by PharmacyPrep.
15-4
Cell and Molecular Biology

D. Join DNA pieces


E. Join RNA pieces
Ans: A
Tips: The enzymes of bacteria restrict the replication of viral genome but cutting the DNA
into pieces.

21. Antisense technology targets?


A. Codons that stop proteins synthesis are used to stop formation of defected proteins.
B. Antisense technology targets mRNA
C. Antisense technology target DNA synthesis
D. Antisense technology target translation in protein synthesis
E. A and B
Ans: B

22. Which of the following is NOT related to plasmid?


A. Bacteria
B. Virus
C. DNA
D. Eukaryotic cell
E. Prokaryotic cell
Ans: B
Tips: eukaryotic cells have linear DNA some eucaryotics cells like yeast have plasmid.
Circular DNA is plasmid and only present in prokaryote cells.

23. Which of the following is NOT a complimentary base pair?


A. G-C and A-T
B. C-G and A-T
C. G-C and T-A
D. D.T-G and G-C
E. A-U and G-C
Ans: D
Tips: All of the above are complementary pairs except T-G and G-C

24. What is NOT used in inserting DNA sample into a plasmid?


A. DNA
B. Plasmid
C. DNA ligase
D. Topoisomerase II
E. All of the above
Ans: D

25. Single or unpaired chromosome is referred as?


A. Hapten
B. Haploid
Copyright © 2000-2020 TIPS Inc. Unauthorized reproduction of this manual is prohibited. This manual is being
used during review sessions conducted by PharmacyPrep.
15-5
Cell and Molecular Biology

C. Diploid
D. Immunogens
E. Antigen
Ans: B
Tips: Single chromosomes is referred as haploid and present in prokaryotes. Paired
chromosome is diploid and present in eukaryotes.

26. Low molecular weight compounds that act as immunogens after chemically complexing
to a larger molecule or cell surface.
A. Hapten
B. Haploid
C. Diploid
D. Immunogens
E. Antigen
Ans: A

27. What base is NOT found in deoxyribose nucleic acid (DNA)?


A. Thymine
B. Adenine
C. Guanine
D. Cytosine
E. Uracil
Ans: E
Tips: Uracil is not found in DNA. Uracil is present in RNA.

28. Information transfer from DNA to M-RNA is referred to as?


A. Transcription
B. Translation
C. DNA-gyrase
D. protein synthesis
E. DNA-Recombination
Ans: A

29. Complimentary DNA (cDNA) is produced from?


A. mRNA
B. tRNA
C. Cell
D. Gene
E. rRNA
Ans: A

30. Which of the following cell organ in eukaryotic cells cholesterol synthesis takes place?
A. Mitochondria
B. Ribosome
Copyright © 2000-2020 TIPS Inc. Unauthorized reproduction of this manual is prohibited. This manual is being
used during review sessions conducted by PharmacyPrep.
15-6
Cell and Molecular Biology

C. Centrosome
D. Endoplasmic reticulum
E. Cytoplasm
Ans: D

31. Antisense RNA targets specific viral or microbial nucleic acid sequences that interfere
with normal replication and expression. The antisense technology targets?
A. Tcell
B. B Cell
C. DNA
D. mRNA
E. cDNA
Ans: D

32. Nucleotide is?


A. Phosphate and sugar
B. Sugar- base - phosphate
C. Sugar and base
D. Base- c 1 -sugar-c 5 -phosphate
E. Base- c 5 -sugar-c 1 -phosphate
Ans: D

33. Conversion of nucleoside to nucleotide is done by?


A. Double bond
B. Sugar
C. Purine base
D. Ribose
E. Phosphate
Ans: E

34. Gene therapy can be used to treat?


A. Cancers
B. Sickle cell anemia
C. Hemophilia
D. All of the above
Ans: D
Tips: Sickle cell anemia affects hemoglobin of RBC and it is genetic carrier gene disease.

35. A cDNA library contains clones representing which of the following?


A. mRNA
B. Repeated DNA
C. Introns
D. tRNA
E. rRNA
Copyright © 2000-2020 TIPS Inc. Unauthorized reproduction of this manual is prohibited. This manual is being
used during review sessions conducted by PharmacyPrep.
15-7
Cell and Molecular Biology

Ans: A

36. During the evolution of prokaryotes to eukaryotes, what changed?


A. Nucleus
B. Cell membrane
C. DNA
D. Nuclear membrane
Ans: A

37. Ribonuclease catalyzes?


A. RNA to nucleoside
B. RNA to DNA
C. RNA to nucleotide
D. RNA to mRNA
E. DNA to nucleotide
Ans: C
Tips: Ribonuclease is a catalyzing enzyme of nucleotide formation from ribonucleic acid.

Copyright © 2000-2020 TIPS Inc. Unauthorized reproduction of this manual is prohibited. This manual is being
used during review sessions conducted by PharmacyPrep.
15-8
Pharmacogenetics

PHARMACY PREP
PHARMACOGENETICS

1. What is the definition of Pharmacogenetics?


A. Pharmacogenetics is the integration of pharmacology and genetics.
B. The study of pharmacogenetics allows designing and developing drugs that are
customized to each person’s genetic mark up.
C. The pharmacogenetics also utilized to study cytochrome enzymes that are
responsible for drug interactions.
D. The pharmacogenetics identifies the genetic variation that causes drug cause
differences in drug response.
E. All of the above
Ans: E

2. Codeine is metabolized by CYP2D6 to active product as morphine. For a person with


slow metabolizing gene of CYP2D6, what is correct?
A. Need less dose of codeine for analgesic effect.
B. Need high dose of codeine for analgesic effect.
C. Need normal dose of codeine for analgesic effect.
D. Codeine should be avoided in slow metabolizer of CYP2D6.
Ans: B
Tips: Codeine won’t efficiently metabolize to morphine, so codeine dosage may have to
be increased.

3. What is SNP?
A. Single Nucleotide Polymorph
B. Single Nucleotide Pharmacogenetic
C. Single Genetic Markup
D. Single Genetic Variation
E. Single Nucleotide Product
Ans: A
Tips: The SNP is a single nucleotide polymorphism (SNP). It occurs when one base pair
of nucleotide replaces another. These are single base differences that exist between
individual. This is the most common genetic variation in DNA.

4. Enzymes that catalyze coupling of two molecules are classified as


A. Hydrolases
B. Ligases
C. Oxidoreductases
D. Transferases
E. Isomerases
Ans: B

Copyright © 2000-2020 TIPS Inc. Unauthorized reproduction of this manual is prohibited. This manual is being
used during review sessions conducted by PharmacyPrep.
16-1
Pharmacogenetics

5. If you know person genetics mark up and target a medicine, it is defined as?
A. Pharmaceutics
B. Biotechnology
C. Nanotechnology
D. Pharmacogenetics
E. Molecular biology
Ans: D

6. Trastuzumab is used for?


A. A HER 2 RNA protein inhibitors used for breast cancer
B. Contraceptives
C. Rheumatoid arthritis
D. Chemotherapy
E. Crohn's disease
Ans. A

7. All of the following are biological derived drugs, EXCEPT:


A. Infliximab
B. Anakinra
C. Trastuzumab
D. Insulin
E. Methotrexate
Ans: E

8. Rituximab is used for the treatment of?


A. Cell follicular lymphoma
B. Contraceptives
C. Rheumatoid arthritis
D. Chemotherapy
E. Crohn's disease
Ans: A

Copyright © 2000-2020 TIPS Inc. Unauthorized reproduction of this manual is prohibited. This manual is being
used during review sessions conducted by PharmacyPrep.
16-2
Immunology
PHARMACY PREP
IMMUNOLOGY and Immunizations
1. What is not a physiological barrier for innate immunity?
A. Tear
B. Salivary amylase
C. Inflammation
D. Skin
E. Mucus
Ans: B
Tips: Innate immunity acting as physical and chemical barrier to infectious agents by using
physical measures such as skin and chemical barrier like clotting factor in blood. However
second line physical or chemical barriers blood brain barrier which protect nervous system from
microorganism is NOT innate immunity.

2. Which is not considered as a physiological barrier to body surfaces?


A. Skin
B. Mucus membrane
C. Tears
D. Gastric secretions
E. Salivary amylase
Ans: E
Tips: Salivary amylase breaks down starch to carbohydrates. Gastric secretions provide hostile
environment to many microorganisms.

Strategic thinking:
1. Read the question.
2. In Immunolgy Chapter, questions will obviously be about protection provided
naturally by the body. Still, you can apply looking for clues and keywords. Clue is
Physiological and the keyword is barrier. From all the given options, only option E
doesn’t share protective function as is the rest.
3. Thus, the correct answer is E.
(Refer to Misbah’s EE Flashcards to help you memorize concepts from this chapter as this
requires largely memorization.)

3. The FALSE statement about vaccines is/are:


I. It is given only after the person suffer exposition to the virus
II. Passive vaccines promote immunization for a short period of time
III. Active vaccines promote long-term immunization
A. I only
B. III only
C. I and II only
D. II and III only

Copyright © 2000-2020 TIPS Inc. Unauthorized reproduction of this manual is prohibited. This manual is being used during
review sessions conducted by PharmacyPrep.
17-1
Immunology
E. I, II and III
Ans: A
Tips: Vaccines can be given as prophylaxis before the person suffers exposition to the
microorganisms.

4. Differences between passive and active immunization:


I. Passive immunity enhances while active immunity stimulates the patient’s immune
response.
II. In passive immunity, antibody injections are given and in active immunity, antigen
injections are given.
III. Passive immunity is effective for some months while active immunity lasts longer.
A. I only
B. III only
C. I and II only
D. II and III only
E. I, II and III
Ans: E
Tips: Passive vaccines: Normally IV and IM injections of antibodies to enhance patient’s immune
competence; protection depends on serum half-life of the injected antibody that may be
effective for days or months. Active vaccinations: Normally IV, IM, SC and oral administration of
antigen to stimulate the immune response. Immunity is long lasting.

5. Correct statement regarding Hapten?


A. Low molecular weight compounds that act as immunogens after chemically complexing
to a larger molecule or cell surface.
B. A small chemical grouping which reacts with preformed antibodies
C. Hapten cannot induce an antibody response
D. All of the above
Ans. D

6. A woman who is a hemophilia-carrier and has a child from a man with hemophilia, who of the
following children is NOT her child?
A. A girl with hemophilia
B. A boy with hemophilia
C. A girl having no gene of hemophilia
D. A girl having the gene of hemophilia
E. A boy having no hemophilia
Ans: C
Tips:If the mother and father have hemophilia their child may not have the disease hemophilia
but the child will have the gene of the disease.

7. All of the following can be differentiated from a mast cell, EXCEPT:


A. Eosinophil
B. Basophile

Copyright © 2000-2020 TIPS Inc. Unauthorized reproduction of this manual is prohibited. This manual is being used during
review sessions conducted by PharmacyPrep.
17-2
Immunology
C. Neutrophil
D. Platelet
E. Erythrocytes
Ans: B
Tips: Basophile is a pro-inflammatory cell that initiates acute inflammation and cannot be
differentiated from mast cells.

8. Basophiles, neutrophils and eosinophils originate from?


A. Myeloblasts
B. Lymphoblasts
C. Monoblasts
D. Proerythroblasts
E. Megakaryoblasts
Ans: A
Tips: Basophiles, neutrophils and eosinophils are leukocytes-White Blood Cells-WBC originated
from myeloblasts.

9. Which of the following is/are NOT considered autoimmune disorder/s?


A. A Grave’s disease
B. Systemic Lupus Erythomatosus
C. Osteoporosis
D. Hashimoto’s disease
E. Type 1 Diabetes
Ans: C
Tips: Osteoporosis is not an autoimmune disorder. It is a disease characterized by the loss of
bone mass.

10. Increase in number of neutrophil indicates?


A. Bacterial infection
B. Viral infection
C. Fungal infection
D. Mycobacteria
E. Acquired immunodeficiency
Ans: A

11. Which of the following is not related to lymphocytes?


A. Lymph nodes
B. Thymus gland
C. Thoracic duct
D. Spleen
E. Thyroid gland
Ans: E
Tips: Thyroid gland is not part of lymphocytes. Lymphocytes include: Tonsils and adenoid gland,
thymus, lympho nodes, spleen, bone marrow and lymphatic vessels.

Copyright © 2000-2020 TIPS Inc. Unauthorized reproduction of this manual is prohibited. This manual is being used during
review sessions conducted by PharmacyPrep.
17-3
Immunology
12. Attenuated viruses used for vaccination:
I. Poliomyelitis virus
II. MMR-Mumps, measles and rubella virus
III. Influenza
A. I only
B. III only
C. I and II only
D. II and III only
E. I, II and III
Ans: C
Tips: An attenuated vaccine is a vaccine created by reducing the virulence of pathogen
(harmless), but still keeping it viable (live).

13. The antigen moiety on an antigen-presenting cell recognized by the alpha, beta T cell receptor
is?
A. Process peptide and MHC
B. Major histocompatibility complex
C. Antigen receptor
D. Antibody receptor
E. Process peptide
Ans: A

14. All of the following biological are used for active immunization, EXCEPT:
A. Bacterial vaccine
B. Bacterial antigen
C. Multiple antigen preparations
D. Toxoids
E. Toxins
Ans: E
Tips: Toxins are used to refer specifically to a protein produced by some higher plants, certain
animals, and pathogenic bacteria, which is highly toxic for other living organisms therefore
cannot be used for vaccination.

15. Which of the following is useful to STIMULATE antibody production?


A. An adjuvant
B. A hapten
C. Antiserum
D. Purified antigen
E. Crude antigen
Ans: D
Tips: Antigen stimulate production of antibodies (Ig). The adjuvant is pharmacological or
immunological agent that modifies the effect of other agents. Adjuvents are added to a vaccine
to boost the immune response. Adjuvent are also used in the production of antibodies from

Copyright © 2000-2020 TIPS Inc. Unauthorized reproduction of this manual is prohibited. This manual is being used during
review sessions conducted by PharmacyPrep.
17-4
Immunology
immunized animals. The most commonly used adjuvents include aluminum hydroxide and
paraffin oil.
Hapten is small molecule that stimulates production molecule only when conjugated to larger
molecule. The hapten reacts specifically with antibodies generated against it to produce an
immune or allergic response.
Antiserum: Is a human serum containing polycolonal antibodies is used to spread passive
immunity. Example survivor of Ebola virus serum is used to treat Ebola virus exposed patient.

16. Correct statements regarding Klein Felter’s syndrome include:


I. Male with enlarge breast, small testis and infertility
II. The male has 3 chromosomes
III. Chromosomes forming XXY, presence of one extra female chromosome.
A. I only
B. III only
C. I and II only
D. II and III only
E. I, II and III
Ans: E
Tips: Klein filter syndrome is a condition where the male have enlarged breast, small testis and
infertility. The male chromosome is constituted of 3 chromosomes forming XXY, presence of one
extra female chromosome.

17. Correct statements regarding infantile genetic agranulocytosis include:


I. Type I hypersensitivity
II. Caused by a decrease in bone marrow activity
III. Treatment can be done by transfer of WBCs or large doses of antibodies.
A. I only
B. III only
C. I and II only
D. II and III only
E. I, II and III
Ans: D
Tips: It is an autosomal recessive disorder characterized by the early onset of recurrent, severe
pyogenic infections, especially of the skin and lung, total absence of neutrophil in the blood or
presence in reduced numbers, absolute monocytosis and eosinophilia, markedly decreased
numbers of mature neutrophilic precursors in the bone marrow.

18. Which of the following is the organ responsible for the immune system?
A. Thymus
B. Lymphoid organ
C. Megakaryocyte
D. Lymphocyte
E. Spleen
Ans: B

Copyright © 2000-2020 TIPS Inc. Unauthorized reproduction of this manual is prohibited. This manual is being used during
review sessions conducted by PharmacyPrep.
17-5
Immunology
Tips: Lymphoid organ is the organ responsible for the immune system concerned with growth,
development and deployment of lymphocytes.

19. The “key” operative of immune system is known as:


A. Megakaryocytic
B. Monocyte
C. Lymphocyte
D. Proerythrocyte
E. Myelocyte
Ans: C
Tips: Lymphocytes are WBC-White Blood Cells, the key operative of immune system.

20. Which of the following is/are types of lymphocytes?


I. B cells
II. T cells
III. Natural killer cell
A. I only
B. III only
C. I and II only
D. II and III only
E. I, II and III
Ans: E
Tips: Lymphocytes are WBC-White Blood Cells found in the blood and in many other part of body
and may include B and T cells, and natural killer cells.

21. The stem cell of immune system is developed in:


A. Bone marrow
B. Lymphoid organ
C. Liver
D. Spleen
E. Lymph nodes
Ans: A
Tips: Steam cells are blood cells progenitor, or mother cell, having the capacity for both
replication and differentiation originated from bone marrow.

22. Which of the following is a pro-inflammatory cell responsible for the initiation of an acute
inflammation?
A. Eosinophils
B. Platelets
C. Macrophages
D. Basophiles
E. Neutrophils
Ans: D

Copyright © 2000-2020 TIPS Inc. Unauthorized reproduction of this manual is prohibited. This manual is being used during
review sessions conducted by PharmacyPrep.
17-6
Immunology
Tips: Basophiles are white blood cells and pro-inflammatory cell responsible for initiation of an
acute inflammation. It also stimulated the release of heparin and histamine

23. B cells are produced in bone marrow and each B cells generate daughter B cells which consists
unique receptors. Each antigen presenting cell selectively binds to selective receptor of B cell
and then generate millions of copies of B cells with the specific receptors. This is according to?
A. Natural selection theory
B. Clonal selection
C. The danger theory
D. Apoptosis
E. Phagocytosis
Ans: B

24. Which of the following cell types of the innate immunity does not perform phagocytosis?
A. Macrophages
B. Neutrophils
C. Basophils
D. Eosinophils
E. All of the above
Ans: C

25. What immune cells circulations throughout the blood helps in humoral or adaptive immunity?
A. Neutrophils
B. Basophils
C. Eosinophils
D. Antigens
E. Antibodies
Ans: E

26. Pneumococcal vaccine is recommended in all, EXCEPT:


A. Frequent otitis media infection
B. To prevent meningitis
C. To prevent pneumonia infections in COPD
D. To prevent streptococcal infections
E. To prevent cellulitis
Ans: E
Tips: Pneumococcal vaccine is indicated because it can reduce common pneumococcus
pneumonia infections of community acquired pneumonia, otitis media, bacterial meningitis, and
prevent strep bacteria.

27. What is true about active immunity?


A. It is mediated by passive vaccination.
B. Antibodies transferred from another body.
C. It is transferred from mother to fetus.

Copyright © 2000-2020 TIPS Inc. Unauthorized reproduction of this manual is prohibited. This manual is being used during
review sessions conducted by PharmacyPrep.
17-7
Immunology
D. The person is infected by vaccine and makes antibody in the body.
Ans: D

28. A 29-year-old female came to the pharmacy complaining of rashes all over her body for 3 days
now. She was apparently all right 3 days ago. It was only after taking her medicine for urinary
tract infections that the rashes appeared on her face and neck which then spread to her body.
What drug most likely caused this reaction?
A. Nitrofurantoin
B. Ciprofloxacin
C. Sulfamethoxazole and trimethoprim
D. Fosfomycin
Ans: C

29. Hyperphosphatemia is best treated by:


A. Magnesium hydroxide
B. Calcium carbonate
C. Aluminium hydroxide
D. Sodium Bicarbonate
E. Sodium phosphate
Ans: C
Tips: Aluminum hydroxide causes hypophosphatemia as side effect therefore may be used in the
treatment of hyperphosphatemia.

30. Correct statements regarding IPECAC may include:


I. It has an emetic effect when large doses are administrated.
II. It has an expectorant effect when small doses (1 to 2mls) is administrated.
III. Used in decontamination procedures during toxic treatments
A. I only
B. III only
C. I and II only
D. II and III only
E. I, II and III
Ans: E
Tips: IPECAC is a gastric decontaminant agent used in the treatment of overdoses and
poisonings of many different agents. It acts by inducing vomiting and is best used within 60
minutes of agent ingestion. It also has an expectorant effect when used in very low doses.

31. The organophosphates commonly found in insecticides are thought to act by which of the
following mechanisms?
A. Combining with acetylcholine
B. Potentiating the action of acetylcholinesterase
C. Forming a very stable complex with acetylcholinesterase
D. Reacting at the cholinergic receptor
E. Preventing the release of acetylcholine from the nerve ending

Copyright © 2000-2020 TIPS Inc. Unauthorized reproduction of this manual is prohibited. This manual is being used during
review sessions conducted by PharmacyPrep.
17-8
Immunology
Ans: C
Tips: Organophosphates are substances commonly found in insecticides in very toxic to humans
due to formation of a very stable complex with acetylcholinesterase

32. Which employ a "respiratory burst" to produce bactericidal chemicals such as hydrogen
peroxide (H2O2) and hypochlorite?
A. Neutrophils
B. Basophils
C. Cytotoxic T cells
D. Natural killer cells
E. Suppressor T cells
Ans: A
Tips: H2O2 is a free radical produced by neutrophils

33. Poison ivy is what type of hypersensitivity reaction?


A. Type I
B. Type II
C. Type III
D. Type IV
E. Type V
Ans: D

34. Monocytes are white blood cells that can swallow and digest microscopic organisms and
particles in a process known as?
A. Phagocytosis
B. Allergic reaction
C. Natural killer cells
D. Gene therapy
E. Monoclonal antibodies
Ans: A

35. What is normal average life span of a patient after a Human Immunodeficiency (HIV) infection?
A. 10 years
B. 2 to 3 years
C. 1 year
D. Long life
E. 5 years
Ans: B

36. A 70-year-old elderly patient is treated with antibiotics for 7 days. However, he still has
symptoms of the infection such as fever. The patient’s neutrophil count is still increased over
70%. What kind of infection does the patient have?
A. Parasitic
B. Viral

Copyright © 2000-2020 TIPS Inc. Unauthorized reproduction of this manual is prohibited. This manual is being used during
review sessions conducted by PharmacyPrep.
17-9
Immunology
C. Fungal
D. Inflammation
E. Bacterial
Ans: E
Normal Indicator
Neutrophils 55% to 75% Bacterial infection
Lymphocytes 20% to 40% Viral infection
Monocytes 0% to 7% Tuberculosis
Eosinophils 0% to 5% Parasites infection
Basophils 0% to 1% Inflammation, Allergies, Asthma

37. If doctor suspect parasite infection, which cells are increased?


A. Neutrophils
B. Eosinophils
C. Basophils
D. Platelets
E. Lymphocytes
Ans: B

38. A 50-year-old COPD patient is diagnosed with pneumonia. What type of cells are increased?
A. Neutrophils
B. Eosinophils COPD or emphysema is Neutrophilic
C. Basophils Asthma is eosinophilic and basophilic.
D. Platelets
E. Lymphocytes
Ans: A

39. All of the following can transmit Human Immunodeficiency (HIV), EXCEPT?
A. Sexual contact
B. Maternal transmission
C. Drug abuse and sharing needle
D. Cut and wounds contact
E. Orofecal
Ans: E

40. A Human Immunodeficiency (HIV) patient currently has CD 4 T cell count is 350. What treatment
does the doctor have target for prophylaxis?
A. M. Tuberculosis
B. Cytomegalovirus
C. Pneumocystis pneumonia jerovicii or carinii
D. Non-Hodgkin’s lymphoma
E. Kaposi’s sarcoma
Ans: A

Copyright © 2000-2020 TIPS Inc. Unauthorized reproduction of this manual is prohibited. This manual is being used during
review sessions conducted by PharmacyPrep.
17-10
Immunology
41. Amoxicillin suspension was prescribed to treat otitis media in a child. The next day, the child
was hospitalized due to severe urticaria/eczema. Doctor diagnosed it as amoxicillin allergy.
What type of hypersensitive reaction id this?
A. Type 4
B. Type 3
C. Type 2
D. Type 1
Ans: D
Tips:
Type 1: Symptoms urticaria/eczema and anaphylactic reaction (swelling, shortness of breath,
hypotension)
Type 3: Serum sickness, arthritis, vasculitis, nephritis.
Type 4: Delayed inflammation reactions. Contact hypersensitivity or dermatitis.

42. What is true about immunoglobulin structure?


A. Consists of protein and amino acids sequence
B. Consists of long chain amino acids and disulfide bond linkage
C. Consists of heavy chain and light chain protein linked with disulfide Bonds Bridge
D. Consists of specific amino acids selective sites of infections.
E. Humoral antibody structures
Ans: C

43. An elderly person with COPD has contraindications to flu vaccine. Person lives with spouse,
adult son, and daughter and an infant <5-month old age. What is correct?
A. Administer flu vaccine to elderly person
B. Administer flu vaccine to spouse only
C. Administer flu vaccine to spouse, son and daughter
D. Do not administer flu vaccine to any of them
Ans: C

44. A customer wants to know if he can just bring Twinrix (Hep A & Hep B) vaccine in his travel. He
already took two doses but he likes to take the third dose during his vacation in Bahamas. What
is appropriate?
A. Tell him that all three doses should be taken in Canada.
B. Tell him to talk to his doctor.
C. Tell him to carry in ice pack and after arriving to destination put in fridge. Can third dose at
your destination.
D. Tell him that he may not need third dose.
Ans: C
Tips: Vaccines are stored in cold chain (fridge temp 2 to 8 C).

45. Person with history of egg allergy wants to receive flu vaccine. What is appropriate?
A. In Egg allergy, flu vaccine is contraindicated thus skin allergy test has to be done first before
flu vaccine administration.

Copyright © 2000-2020 TIPS Inc. Unauthorized reproduction of this manual is prohibited. This manual is being used during
review sessions conducted by PharmacyPrep.
17-11
Immunology
B. Egg allergy patient should take influenza intranasal Mist as an alternative.
C. Should be vaccinated at a medical setting and supervised by a healthcare provider.
D. Flu vaccine can be given in egg allergy patient at any pharmacy done by flu-vaccine trained
pharmacist.
E. Give combination of bi-valent and tri-valent.
Ans: D

46. Which of the following classes of drugs is most likely associated with Steven’s Johnson
Syndrome?
A. Penicillins
B. Non-Steroidal Anti-inflammatory Drugs (NSAIDs)
C. Antiepileptics
D. Antidepressant
E. Antipsychotics
Ans: C
Tips: Antiepiletpics phenobarbital, carbamazepine, phenytoin, valproic acid, lamotrigine are
associated with SJS.

47. A patient was admitted to the emergency due to hypertension crisis. Doctor initiated
hydralazine IV. Patient reacted with rash and fever. What is the appropriate action?
A. It is systemic lupus.
B. Hydralazine infusion should be stopped immediately.
C. Ask nurse how patient is doing.
D. It’s an allergic reaction.
Ans: B
Tips: Hydralazine is associated with lupus like syndrome reactions and rash, fever is initial
symptoms of lupus.
48. Which of the following vaccine can be use in 2nd or 3rd trimester of pregnancy?
A. Chicken pox
B. Flu vaccine
C. Yellow fever
D. Tuberculosis
Ans: B

49. All of the following vaccines are avoided in immunocompromised patients, EXCEPT:
A. Polio oral vaccine
B. Flu intranasal Mist
C. Typhoid oral
D. Dukoral
Ans: D
Tips: Dukoral is an oral vaccine they provide protection against travelers’ diarrhea and cholera
vibrio is an inactivated (killed) vaccine.

50. A regular customer at your pharmacy asks you when is the best time to take a flu shot?

Copyright © 2000-2020 TIPS Inc. Unauthorized reproduction of this manual is prohibited. This manual is being used during
review sessions conducted by PharmacyPrep.
17-12
Immunology
A. October
B. November
C. December
D. Oct to Nov
E. April
Ans: D
Tips: Flu season is from October to April. The immunization season is Oct to Mid-Nov is flu
immunization time.

51. What is not present in lymph fluid?


A. Urea
B. Lymphocytes
C. Platelets
D. Creatinine
E. Proteins
Ans: C
Tips: lymph fluid contains, oxygen, proteins, glucose and white blood cells.
Lymph is clear colorless, liquid with composition of similar to blood plasma.

52. Lymphatic vessels, which is not true?


A. Contains valves that prevent back flow
B. Connects into small veins
C. Regulated by heart rate
D. Consist of blood cells and proteins
Ans: C

53. Blood group mismatch transfusion can cause?


A. Type I hypersensitive reaction
B. Type 2 hypersensitive reaction
C. Type 3 hypersensitive reaction
D. Type 4 hypersensitive reaction
Ans: B

54. Removal of this organ will be the most harmful for a one-year-old child.
A. Spleen
B. Lymph node
C. Thymus
D. Appendix
E. Palatine tonsil
Ans: C
Tips: Immune cells Tcells mature in thymus and this primary defense for children.

55. Which is used to assess Human Immunodeficiency Virus (HIV) disease progression?
A. Plasma viral load

Copyright © 2000-2020 TIPS Inc. Unauthorized reproduction of this manual is prohibited. This manual is being used during
review sessions conducted by PharmacyPrep.
17-13
Immunology
B. Tuberculin skin test
C. Chest x-ray
D. Gram test
E. EMRI
Ans: A

56. In the treatment regimen of AIDS patient, which is most correct?


A. Take 2 medications that give synergetic action
B. It’s not true to take 2 medications with each other’s
C. Give 2 medications with additive effect of different groups
D. Treatment with each drug group alone
Ans: C

57. Passive immunity?


A. Giving antibodies directly
B. Taking from mother immunity through breast milk
C. Long duration effect
D. Used as prophylaxis
Ans: A
Tips: Passive immunity Example. Hepatitis B immune globulins (HBIG). Prophylaxis and therapy.
Varicella zoster immune globulins (VZIG).
Rh o (D) immune globulin (RhoGAM) prophylaxis for Rh +ve fetus by Rh –ve mother receives RhoGAM).
Prophylaxis given during pregnancy and after labor (delivery).

58. Which of the following does NOT play a role in antigen presentation?
A. Major histocompatibility class 1 molecule
B. Major histocompatibility class II molecules
C. Immunoglobulin IgM
D. None of the above
Ans: C

59. Which of the following soluble molecule helps in detecting disease?


A. Cell associated differentiation antigens (CD) molecules
B. Major histocompatibility class I molecules
C. B-cell clones
D. Cytotoxic T cells
Ans: A
Tips: CD antigen molecules like CD4 count helps in diagnosis of acquired immunodeficiency.

60. Where does lymphocyte maturation occur?


A. Bone marrow
B. Thymus
C. Spleen
D. Thyroid gland
Ans: B

Copyright © 2000-2020 TIPS Inc. Unauthorized reproduction of this manual is prohibited. This manual is being used during
review sessions conducted by PharmacyPrep.
17-14
Biotechnology

PHARMACY PREP
BIOTECHNOLOGY

1. Protein-biotechnology products are put in all, EXCEPT:


A. Capsule
B. Microsphere
C. Nanoparticle
D. Liposome
Ans: D

2. Chimeric antibody chain is?


A. Humanized chain linked to mouse chain.
B. Humanized chain linked to rat chain
C. Humanized chain binding with different antigen
D. One quarter the antibody is from one species origin whereas the other 3 quarters from
other species
Ans: D

3. Herceptin (Trastuzumab), is a humanized monoclonal antibody?


A. Treatment of HER2-overexpressing metastatic breast cancer
B. Tumor necrosis factor alpha inhibitor
C. Glycoprotein inhibitor
D. Interleukin
E. Interferon's
Ans: A

4. Erythropoietins are?
A. Colony stimulating factors
B. Circulating iron storage
C. Interleukins
D. Immunosuppressant
E. Biotechnological products
Ans: A
Tips: Erythropoietin's are CSF. Products epoetin alpha and beta are used for the treatment of
chemotherapy induced anemia.

5. Hematocrit:
A. Is a circulating iron storage.
B. Measures the total iron binding capacity
C. Measures the total free iron
D. The proportion of the blood that consist of packed RBCs
E. It is a transferrin.
Ans: D

Copyright © 2000-2020 TIPS Inc. Unauthorized reproduction of this manual is prohibited. This manual is being used during
review sessions conducted by PharmacyPrep.
18-1
Biotechnology

6. Adalimumab origin is?


A. Mouse
B. Human
C. Humanized
D. Human and mouse
E. Chimeric
Ans: B
Strategic thinking:
1. Read the question.
2. As in other chapters, biotechnology products discussed in Biotechnology chapter adapt
proper nomenclature as prescribed by the responsible organization. So, in light of this
question, take the word and note of the letter that precedes the suffix –mab. Here, it’s –u-.
Remember from your lecture notes that this signifies the human origin of the monoclonal
antibody. Avoid confusing it with Humanized as this suggests different source.
3. Thus, the correct answer is B.
(Refer to Misbah’s EE lecture notes from the Biotechnology chapter of the these infixes)

7. Erythropoietins are produced from?


A. Kidney
B. Liver
C. Bone marrow
D. Muscles
E. Thymus gland
Ans: A

8. Filgrastim is a granulocyte Colony Stimulating Factor (g-CSF) indicated in the treatment of?
A. Renal, cardiac and hepatic acute graft rejection
B. Neutropenia associated with cancer chemotherapy
C. Thrombocytopenia associated with cancer chemotherapy
D. Anemia associated with chronic renal diseases
E. Chemotherapy induced anemia
Ans: B

9. Filgrastim is?
A. Colony stimulating factor
B. Erythropoietin
C. Interferon
D. Human growth hormones
E. Monoclonal antibodies
Ans: A

10. Oprelvekin is?


A. Interleukin 11
B. Colony stimulating factor

Copyright © 2000-2020 TIPS Inc. Unauthorized reproduction of this manual is prohibited. This manual is being used during
review sessions conducted by PharmacyPrep.
18-2
Biotechnology

C. Interleukin 3
D. Interleukin 2
E. Epoetin
Ans: A

11. Oprelvekin (IL-11) is indicated for?


A. Chemotherapy induced thrombocytopenia
B. Bone marrow suppression
C. Acute graft rejection
D. Neutropenia
E. Growth factor
Ans: A
Tips: Interleukin 11 (IL-11) source cells are mesenchymal cells. It targets hematopoetic stem cells
and induces megarkaryocytes proliferation.

12. Muromonab (OKT 3 ) is monoclonal antibody is used for treatment of?


A. Renal, cardiac and hepatic graft rejection
B. Neutropenia associated with cancer chemotherapy
C. Thrombocytopenia associated with cancer chemotherapy
D. Anemia associated with chronic renal diseases
E. None of the above
Ans: A

13. CD 4 cells mainly represent?


A. Helper T cells
B. Suppressor T cells
C. Cytotoxic T cells
D. Immunoglobulins
E. None of the above
Ans: A

14. Which of the following is a Tumor Necrosis Factor (TNF) inhibitor used to treat rheumatoid
arthritis?
A. Rituximab
B. Abciximab
C. Etanercept
D. Trastuzumab
E. Methotrexate
Ans: C

15. Genetic disease hemophilia A is due to deficiency of clotting factor?


A. Factor 11
B. Factor 10

Copyright © 2000-2020 TIPS Inc. Unauthorized reproduction of this manual is prohibited. This manual is being used during
review sessions conducted by PharmacyPrep.
18-3
Biotechnology

C. Factor 12
D. Factor 8
E. Factor 9
Ans: D

16. Interferons are good examples of:


I. Cytokines
II. Biotechnological products
III. Colony stimulating factors (CSF)
A. I only
B. III only
C. I and II only
D. II and III only
E. All of the above
Ans: C

17. HAMA is?


A. Human antigen and Mouse antibody
B. Human antimouse antibody
C. Murine antibody
D. Humanized antibody
E. Human antibody
Ans: B

18. Trastuzumab is produced by?


A. Cell culture
B. Monoclonal antibody method
C. Chimeric monoclonal antibody
D. murine monoclonal antibody
E. Humanized monoclonal antibody
Ans: E
Tips: Trastuzumab is humanized MAB, which is produced from transgenic mice.

19. A 40-year-old man comes to your pharmacy. He is concerned that he may likely get shingles. His
mother and father suffered from it. They took shingle vaccine and it helped them. He wonders
and likes to take vaccine but he is not sure if he had chicken pox before. What is the
appropriate response?
A. All people over age 50 should take shingle vaccine
B. Dispense him shingle vaccine and tell him to take doctor for injection
C. Tell him that he is too young to get shingle vaccine
D. Tell him to test if he has antibodies. If test is positive, he can take the vaccine when he’s
over 50 years old.
Ans: D

Copyright © 2000-2020 TIPS Inc. Unauthorized reproduction of this manual is prohibited. This manual is being used during
review sessions conducted by PharmacyPrep.
18-4
Biotechnology

20. Which of the following statement is false regarding viral vectors used in the production of
biotechnology drugs?
A. Viruses can be used by nutrients
B. viruses can be generated by infected production cell lines
C. The most frequency of source of introduction of virus is growth media
D. Viruses can be inactivated physical and chemical treatment of product
E. There is trend toward using better defined growth media in which serum levels are
significantly reduced.
Ans: C
Tips: viral vector are carriers commonly used to deliver genetic material into cells. Viral vectors
are often given certain genes that help identify which cells took up the viral genes as markers.
The cell can then be isolated easily, as those that have not taken up the viral vectors genes do
not have antibiotic resistance, and so cannot grow in culture with relevant antibiotic present.

21. True statements regarding the use of monoclonal antibodies include which of the following?
I. Flu-like symptoms commonly occur at the start of therapy
II. T-cells are blocked from initiating the rejection process
III. Orthoclone is a human immunoglobulin product
A. I only
B. III only
C. I and II only
D. II and III only
E. I, II and III
Ans: C
Tips: Monoclonal antibodies can deliver toxins specifically to cancer cells and destroy them.
They are also used with radioisotopes to diagnose and visualize cancer cells and may cause flu-
like symptoms in the beginning of the treatment.

Copyright © 2000-2020 TIPS Inc. Unauthorized reproduction of this manual is prohibited. This manual is being used during
review sessions conducted by PharmacyPrep.
18-5
Biotechnology

Copyright © 2000-2020 TIPS Inc. Unauthorized reproduction of this manual is prohibited. This manual is being used during
review sessions conducted by PharmacyPrep.
18-6
Toxicology

PHARMACY PREP
TOXICOLOGY

1. Antidote of Mercury may include:


A. Dimercaprol
B. EDTA
C. Deferoxamine
D. Succimer
Ans: A
Tips: Antidote of elemental mercury inhalation is Dimercaprol and inorganic salt. For
mercury with gastrointestinal absorption, the antidote may be Dimercaprol and
Penicillamine.

2. Which of the following is a benzodiazepine antagonist?


A. Naloxone
B. Physostigmine
C. Flumazenil
D. Naltrexone
E. Penicillamine
Ans: C
Tips: Flumazenil is a benzodiazepine antagonist therefore used as antidote in
benzodiazepine overdoses.

3. What would be the best treatment for cyanide poisoning?


A. Anti-venom
B. Sodium thiosulfite
C. Acetylcysteine
D. Oxygen therapy
E. Sodium carbonate
Ans: B
Tips: Treatment for cyanide poisoning may be done by sodium thiosulfite, amyl and
sodium nitrile as well.

4. Carbon monoxide can be highly toxic because it easily binds to:


I. Hemoglobin
II. Myoglobin
III. Cytochrome oxidase
A. I only
B. III only
C. I and II only
D. All are correct
Ans: D
Tips: Carbon monoxide can be highly toxic due to its ability to rapidly bind to

Copyright © 2000-2020 TIPS Inc. Unauthorized reproduction of this manual is prohibited. This manual is being
used during review sessions conducted by PharmacyPrep. 19-1
Toxicology

hemoglobin, myoglobin and cytochrome oxidase. Essential components for the vital
functions in human body.

5. Acetaminophen toxicity’s most well-known complication is:


A. Cardiovascular failure
B. Pulmonary edema
C. CNS lethargy
D. Liver necrosis
Ans: D
Tips: The main and more serious implication concerning acetaminophen toxicity is liver
necrosis.

6. Salicylate toxicity treatment includes which of the following?


I. Induce emesis with syrup of IPECAC until 30 minutes of overdose
II. Decontamination with CHARCOAL every 6 hours
III. Alkalinisation of urine with sodium bicarbonate
A. I only
B. III only
C. I and II only
D. II and III only
E. All are correct
Ans: E
Tips: Salicylate toxicity treatment can first be done by control of the vital functions
following administration of sodium bicarbonate to alkalinize the urine and correct the
acidosis, induce emesis with syrup of IPECAC until 30 minutes of overdose and
decontamination CHARCOAL every 6 hours may be considered. In case of failure of these
methods, hemodialysis should be considered.

7. Hyperphosphatemia is best treated by:


A. Magnesium hydroxide
B. Calcium carbonate
C. Aluminium hydroxide
D. Sodium Bicarbonate
E. Sodium phosphate
Ans: C
Tips: Aluminum hydroxide causes hypophosphatemia as side effect therefore may be
used in the treatment of hyperphosphatemia.

8. Correct statements regarding IPECAC may include:


I. It has an emetic effect when large doses are administrated.
II. It has an expectorant effect when a small dose (1 to 2mls) is administrated.
III. Used in decontamination procedures during toxic treatments
A. I only
B. III only

Copyright © 2000-2020 TIPS Inc. Unauthorized reproduction of this manual is prohibited. This manual is being
used during review sessions conducted by PharmacyPrep. 19-2
Toxicology

C. I and II only
D. II and III only
E. . All are correct
Ans: E
Tips: IPECAC is a gastric decontaminant agent used in the treatment of overdoses and poisonings
of many different agents. It acts by inducing vomiting and is best used within 60 minutes of
agent ingestion. It also has an expectorant effect when used in very low doses.

9. The organophosphates commonly found in insecticides are thought to act by which of


the following mechanisms?
A. Combining with acetylcholine.
B. Potentiating the action of acetylcholinesterase.
C. Forming a very stable complex with acetylcholinesterase.
D. Reacting at the cholinergic receptor.
E. Preventing the release of acetylcholine from the nerve ending.
Ans: C
Tips: Organophosphates are substances commonly found in insecticides in very toxic to
humans due to formation of a very stable complex with acetylcholinesterase

10. All of the following can be recognized as atropine poisoning, EXCEPT:


A. Dry skin
B. Mydriasis
C. Flushed appearance
D. Diarrhea
E. Delirium
Ans: D
Tips: Constipation is a common side effect of atropine poisoning, therefore we should
not consider diarrhea as atropine symptoms of overdose.

11. Antidote for atropine poisoning may include:


A. Physostigmine
B. Aminophylline
C. Pralidoxine
D. Flumazenil
E. Dimercaprol
Ans: A
Tips: Physostigmine is considered as the best antidote for atropine poisoning and for
antihistaminic overdose as well.

12. Poison Ivy can be treated by which of the following agents?


I. Topical antipruritic
II. Antihistamine oral
III. Systemic antibiotics
A. I only

Copyright © 2000-2020 TIPS Inc. Unauthorized reproduction of this manual is prohibited. This manual is being
used during review sessions conducted by PharmacyPrep. 19-3
Toxicology

B. III only
C. I and II only
D. II and III only
E. All are correct
Ans: C
Tips: Poison Ivy can be treated by topical antipruritic, oral antihistaminic and systemic
corticosteroids in case of severe poisoning. Antibiotic are NOT used.

13. A patient is stabilized on heparin as anticoagulant. After one hour of administration of


the drug, he experienced bleeding from his gums, while he was brushing his teeth.
What is the antidote of heparin?
A. Ascorbic acid
B. Vitamin K
C. Protamine sulfate
D. Warfarin
E. Quinolone
Ans: C
Tips: Protamine sulfate is the recommended antidote for heparin.

14. N-acetylcysteine may be the antidote of choice for which of the following agents?
A. Aspirin
B. Warfarin
C. Digoxin
D. Lead
E. Acetaminophen
Ans: E
Tips: N-acetyl-cysteine is the specific antidote for acetaminophen overdose or poisoning.

15. A customer at your pharmacy suspected Fentanyl overdose. What is appropriate?


A. Give charcoal.
B. Give ipecac.
C. Give poison control center number to contact.
D. Refer to doctor.
E. Call 911 and administer Naloxone as soon as possible.
Ans: E

16. Naloxone is the best antidote for which of the following overdose agents?
A. Opioid overdose.
B. Treatment of alcohol overdose symptoms.
C. Benzodiazepines overdose.
D. Organophosphate overdose.
E. Digitalis overdose.
Ans: A
Tips: Naloxone is a pure opioid antagonist used in opioids overdose.

Copyright © 2000-2020 TIPS Inc. Unauthorized reproduction of this manual is prohibited. This manual is being
used during review sessions conducted by PharmacyPrep. 19-4
Toxicology

17. Management and characteristics of Digoxin toxicity may include:


I. May cause heart failure, cardiac dysrrhythmias, nausea, anorexia, vomiting,
and confusion.
II. Treated by decontamination and supportive therapy.
III. Treated with a digoxin specific antidote known as FAB antibodies.
A. I only
B. III only
C. I and II only
D. II and III only
E. All are correct
Ans: E
Tips: Digoxin toxicity may cause heart failure, cardiac dysrhythmias, nausea, anorexia,
vomiting, and confusion. The first step in the treatment of overdoses should include
decontamination and supportive therapy. Digoxin has a specific antidote known as FAB
antibodies (Digibind).

18. Death due to cyanide poisoning is due to:


A. Cyanide-RBCs complex formation
B. Cyanide-hemoglobin complex formation
C. Cyanide inhibiting of cytochrome oxidase
D. Cyanide increasing hemoglobin levels
E. Coronary vessel oclusion
Ans: B
Tips: Death due to cyanide poisoning is primarily due to cyanide-hemoglobin complex
formation.

19. The toxicity of Methyl alcohol is due to formation of:


A. Ketones
B. Formic acid
C. Free alcohol radicals
D. Ethylene
E. Aldehyde
Ans: B
Tips: The toxicity of methyl alcohol is mainly due to formation of formaldehyde then
oxidized to formic acid.

20. Acetaminophen toxicity is due to:


A. Oxidation stress
B. Active metabolite
C. Free radical chain
D. Reactive metabolite
E. Reduction of metabolite
Ans: D
Tips: Acetaminophen toxicity is mainly due to its reactive metabolite.

Copyright © 2000-2020 TIPS Inc. Unauthorized reproduction of this manual is prohibited. This manual is being
used during review sessions conducted by PharmacyPrep. 19-5
Toxicology

21. Salicylate toxicity excessive respiration is due to:


A. Excess production of CO 2
B. Pulmonary irritation
C. Central stimulation in the brain
D. Hypoventilation
E. Decrease production of CO 2
Ans: A
Tips: Salicylate toxicity excessive respiration is due to excess production of CO 2 .

22. Grayish mouth and loose of teeth are toxic symptoms of:
I. Fe salts poisoning
II. Cu poisoning
III. Lead poisoning
A. I only
B. III only
C. I and II only
D. II and III only
E. All are correct
Ans: B
Comments: Lead poisoning is mainly characterized by grayish mouth and loose of teeth.
The best antidote for lead poisoning includes dimercaprol, edentate, penicillamine and
succimer.

23. What is incorrect about Naloxone?


A. Antidote of Morphine
B. Antidote of Fentanyl
C. Opioid user patient can buy naloxone kit and can be kept at home for self-
administration in-case of overdose.
D. Naloxone inj. Only administered in poison control centers.
Ans: D
Tips: Vomiting induction normally done by administration of IPECA is contra indicated if
poisoning is due to bleaching, gasoline, light petroleum and/or any other corrosive
substance.

24. Universal antidote is a mixture of:


I. Activated charcoal
II. Magnesium oxide
III. Tannic acid
A. I only
B. III only
C. . I and II only
D. II and III only
E. All are correct
Ans: E

Copyright © 2000-2020 TIPS Inc. Unauthorized reproduction of this manual is prohibited. This manual is being
used during review sessions conducted by PharmacyPrep. 19-6
Toxicology

Tips: Universal antidote is a mixture of activated charcoal, magnesium oxide and tannic
acid.

25. Amphetamines overdose associated seizure is best treated with the administration of:
A. Salicylates
B. Benzodiazepes
C. Barbiturates
D. Naloxone
E. Acetaminphen
Ans: B
Tips: The main concern in amphetamines overdose is the control of the seizures that can
be resolved by using either benzodiazepine or barbiturate agents.
Strategic thinking:
1. Read the question.
2. Toxicology chapter is largely about knowing the medications that work
against each other. As such, one is given or used as the antidote in cases of
poisoning of the other. So, in light of this question, Amphetamines, in
general are stimulants. You have to look for a medication that causes the
opposite. In addition, safety profile of your final choice should be
considered.
3. Thus, the correct answer is B.

26. True statements regarding the treatment of hyperkalemia:


I. If change in ECG is detected give Ca to counteract the excess of K on the heart.
II. Bicarbonate and insulin administration can shift K from extra to intracellular.
III. Enema of kayexalate (exchange resin) or dialysis helps to remove excess of K
from the body.
A. I only
B. III only
C. I and II only
D. II and III only
E. All are correct
Ans: E
Tips: Hyperkalemia should be immediately treated if change in ECG is detected; give Ca
to counteract the excess of K on the heart and other treatments may be considered such
as bicarbonate and insulin administration that can shift K from extra to intracellular.
Alternative treatment of hyperkalemia includes enema of kayexalate (exchange resin) or
dialysis that helps to remove excess of K from the body.

27. Cardiovascular drug that causes cyanide intoxication from overdosing include/s:
A. Nitrates
B. Nitroglycerin
C. Nitroprusside
D. Isosorbide

Copyright © 2000-2020 TIPS Inc. Unauthorized reproduction of this manual is prohibited. This manual is being
used during review sessions conducted by PharmacyPrep. 19-7
Toxicology

E. Isosorbide dinitrate
Ans: C
Tips: Nitroprusside is a direct vasodilator agent widely used in cardiovascular
complications. Overdosage may lead to cyanide intoxication because its end metabolism
product is cyanide that has great affinity with hemoglobin.

28. Lead poisoning can be treated by:


I. BAL-Dimercaprol
II. EDTA-Edentate
III. Penicillamine
A. I only
B. III only
C. I and II only
D. II and III only
E. All are correct
Ans: E
Tips: The best antidote for lead poisoning includes dimercaprol, edentate, penicillamine
and succimer.

29. Specific antidote for iron preparations overdose include:


A. Dimercaprol
B. Deferoxamine
C. Penicillamine
D. Naloxone
E. Sucimmer
Ans: B
Tips: Deferoxamine is the only specific antidote used as antidote for iron preparations
overdose or poisoning.

30. Atropine, an anticholinergic drug is used as antidote in the poisoning treatment of:
A. Organophosphates
B. Heavy metal
C. Salicylates
D. Tricyclic antidepressants
E. Iron
Ans: A
Tips: Atropine is considered to be the best antidote in the treatment of
organophosphates poisoning.

31. Treatment of tricyclic antidepressants overdose is done by all the following procedures,
EXCEPT:
A. Control the seizures and cardiotoxicity.
B. Administration of benzodiazepine or phenytoin to control seizures.
C. Activated charcoal can be used for decontamination

Copyright © 2000-2020 TIPS Inc. Unauthorized reproduction of this manual is prohibited. This manual is being
used during review sessions conducted by PharmacyPrep. 19-8
Toxicology

D. MAO antidepressants can helps to overcome the patient’s seizure.


E. Control acidosis with administration of sodium bicarbonate.
Ans: D
Tips: MAO antidepressants administration in tricyclic antidepressant overdoses would
intensify the overdose of the patient and would increase the chances of the patients in
develop a syndrome called “serotoninergic syndrome”.

32. Specific antidote used for poisoning caused by snake and black spider bits
A. Deferoxamine
B. Ancrod
C. Antivenin or antivenom
D. Psysostigmine
E. Naloxone
Ans: C
Tips: Antivenin is the antidote of choice in poisoning due to bits of snake and black
spider.

33. Best treatment for carbon monoxide intoxication:


A. Alkalinization of urine
B. Oxygen therapy
C. Supportive therapy
D. Acidification of urine
E. Gastric lavage
Ans: B
Tips: Oxygen therapy is the most appropriate method to treat carbon monoxide
intoxication to prevent it from binding to hemoglobin.

34. Which of the following is the isoniazide antidote?


A. Vitamin A
B. Vitamin B6
C. Vitamin B12
D. Folic acid
E. Vitamin E
Ans: B
Tips: Vitamin B6 also known as pyridoxine is considered the best antidote in isoniazide
overdoses.

35. Which of the following is considered the warfarin antidote?


A. Vitamin K
B. Vitamin D
C. Vitamin A
D. Vitamin B 12
E. Vitamin E
Ans: A

Copyright © 2000-2020 TIPS Inc. Unauthorized reproduction of this manual is prohibited. This manual is being
used during review sessions conducted by PharmacyPrep. 19-9
Toxicology

Tips: Vitamin K is considered the specific antidote for warfarin overdose.

36. The major first step in the initial management of ANY intoxication and poisoning is the
supportive care treatment that involves:
A. Detoxification
B. Decontamination
C. Hydration
D. Evaluation and support of vital function (Airway, Breath and Circulation-ABC)
E. Toxicology laboratory tests
Ans: D
Tips: Evaluation and support of vital function such as airway, breath and circulation
should be considered as the major first step in the initial management of any
intoxication and poisoning.

37. General management in treating patients with depressed mental status include:
I. Treat hypoglycemia with 50mls of dextrose 50%.
II. Administration of Thiamine 100 mg IV push.
III. Administration of Naloxone
A. I only
B. III only
C. I and II only
D. II and III only
E. All are correct
Ans: E
Tips: Patient with depressed mental status and unknown drug overdose should be
generally treated by control of hypoglycemia with 50mls of dextrose 50%, administration
of Thiamine 100mg IV push and administration of Naloxone.

38. Decontamination procedures may include:


A. Gastric lavage
B. Emesis
C. Administration of activated charcoal
D. Administration of adsorbent agents
E. All are correct
Ans: E
Tips: Decontamination procedures involve removal of the ingesting agent with gastric
lavage, emesis by IPECA administration, and adsorbent agents administration-charcoal.

39. Intravenous calcium administration is used in which of the following situations?


A. Verapamil overdoses
B. Cocaine overdose
C. Verapamil overdose and hyperkalemia
D. Hyperkalemia
E. Nifedipine overdose

Copyright © 2000-2020 TIPS Inc. Unauthorized reproduction of this manual is prohibited. This manual is being
used during review sessions conducted by PharmacyPrep. 19-10
Toxicology

Ans: C
Tips: Verapamil overdose associated with hyperkalemia may be treated by intravenous
calcium administration

40. What metabolic reaction saturation can lead to acetaminophen toxicity?


A. Glucuronidation
B. Sulfonation
C. Oxidation
D. Glutathione conjugation
E. Reduction
Ans: D

41. Ethanol metabolizes to acetic acid by the enzyme called?


A. Alcohol dehydrogenase
B. Alcohol reductase
C. Glucorunidation
D. Alcohol decarboxylation
Ans: A

42. Which of the following is the antidote of ASA?


A. N-acetyl cysteine
B. Flumazenil
C. Pentoxifilin
D. Naloxone
E. None of the above
Ans: E
Tips: There is no antidote for ASA, as overdose has to be treated through alkaline
diuresis using NaHCO 3 .

43. Heparin antidote is?


A. Vitamin K
B. Heparin
C. Protamine sulphate
D. Filgrastim
E. Vitamin E
Ans: C

44. What is true about charcoal?


I. Increase surface area increases adsorption.
II. Decrease impurities on surface of charcoal increases adsorption.
III. Increase temperature on surface of charcoal increases adsorption.
A. I only
B. III only
C. I and II only

Copyright © 2000-2020 TIPS Inc. Unauthorized reproduction of this manual is prohibited. This manual is being
used during review sessions conducted by PharmacyPrep. 19-11
Toxicology

D. II and III only


E. All are correct
Ans: E

45. A mother approaches your pharmacy. Her 3-year-old child swallowed a lot of
acetaminophen. Which are the initial symptoms of this poisoning?
A. Diarrhea
B. Headache
C. Fatigue
D. Fever
E. Vomiting
Ans: E
Tips: Acetaminophen large doses can cause toxicity. Initiall, for 2 to 4 hs, no symptoms.
Then toxic symptoms can occur in 4 stages:
Stage 1: Vomiting can occur.
Stage 2: After 24 hr nausea, vomiting can get severe.
Stage 3: Can cause toxicity, symptoms bleeding, jaundice.
Stage 4: Can recover or if not recovered, it can cause liver failure

46. Which of the following is used as urine acidifier in alkaline toxicities?


A. Ammonium chloride
B. Ascorbic acid
C. Sod. Bicarbonate
D. all of the above
E. none of the above
Ans: B

47. A grandmother administered 2 tbsp tid of Amoxicillin 300 mg/5 ml instead of 2 tsp tid to
9 mo old child. What to do?
A. Refer to emergency.
B. Refer to doctor.
C. Wait and watch.
D. Give antidote
E. Give oral rehydration solution.
Ans: B
Strategic thinking:
1. Read the question. Identify and organize information as you go.
2. In answering toxicity question, AGE factor is an important consideration in
the proper management along with the presenting symptoms. In this
particular case, the patient is YOUNG (<1 y/o). Unless mentioned in the
case, do not assume EMERGENCY need of overdosing. However as this is a
child, the best course of action is to refer it to the doctor to contain the
possible side effects as early as possible.
3. Thus, the correct answer is B.

Copyright © 2000-2020 TIPS Inc. Unauthorized reproduction of this manual is prohibited. This manual is being
used during review sessions conducted by PharmacyPrep. 19-12
Toxicology

48. What is appropriate action after any dispensing error?


A. Complain to college of pharmacy
B. Document and discuss with pharmacy team member about error and prevention
strategies.
C. Make incident report to ISMP.
D. Tell pt. that we will refund prescription fee
Ans: B

49. Which of the following medication dispensing error is the most concern?
A. Acetaminophen
B. Nystatin
C. Insulin
D. Oral rehydration solution
E. Codeine
Ans: C
Tips: insulin is an high alert dispensing error can be very dangerous, so these drugs are
referred as high alert drugs and require extra precaution in dispensing.

50. To minimize error for the high alert drugs. What is appropriate precaution?
A. Counsel patient use cautiously
B. Label “high alert double check” or independent double check.
C. Auxiliary label to patient
D. Dispense in special container
Ans: B
Tips: examples of high alert medications are insulin, heparin, benzodiazepine,
anticoagulants, chemotherapy, anesthetics, oxytosin, nitroprusside, narcotics.

51. EDTA is used to treat toxicity of?


A. Iron
B. Lead
C. Mercury
D. Arsenic
Ans: B
Tips: Any kind of divalent ion is treated by EDTA.
TOXICOLOGY

52. Acetyl salicylic acid (ASA) use in children with fever symptoms is dangerous due to?
A. Metabolic acidosis
B. Respiratory alkalosis
C. Reye syndrome
D. GI bleeding
E. Salicylism
Ans: C

Copyright © 2000-2020 TIPS Inc. Unauthorized reproduction of this manual is prohibited. This manual is being
used during review sessions conducted by PharmacyPrep. 19-13
Toxicology

Tips: Reye’s syndrome is rare but serious condition that causes swelling liver and brain.
Reye’s syndrome most often affects children and teenager recovering from viral infection
such as flu, chickenpox.

53. N-Acetyl cysteine (NAC) is an antidote of acetaminophen. What is mechanism of


reducing the acetaminophen toxicity?
A. Binding with toxic intermediate N-acetyl-p-benzo-quinone imine.
B. Activates glutathione thus increase formation of non-toxic conjugates of cysteine
and mercapturic acid.
C. Acetaminophen phase 2 metabolic reaction of non-toxic glucuronide conjugation.
D. Acetaminophen phase 2 metabolic reaction of non-toxic sulphate conjugation.
Ans: B
NAC Cysteine  Glutathione

54. Unconscious patient was brought to emergency with history of unknown drug overdose.
Which of the following action should physician perform?
A. Administer 50 ml of 50% dextrose; thiamine 100 mg iv; Naloxone iv.
B. Protect patient airway and ensure the vital signs of stable
C. Order the following lab test, CBC, toxicology screen
D. All of the above
Ans: D

55. Ethyl alcohol administered to patient who has ingested ethylene glycol or methanol
because?
A. Helps sedate patient.
B. Increase metabolism of ethylene glycol and methanol.
C. Blocks the formation of toxic metabolite eg and methanol
D. Increase renal clearance.
E. Not an antidote for ethylene glycol and methanol.
Ans: C

56. Contraindication to administration of syrup of ipecac?


A. Unconscious patient
B. Pt. with generalized tonic clonic-seizure
C. Pt. who ingested caustic substance
D. All of the above
Ans: D

57. What is NOT a clinical presentation of organophosphate insecticide poisoning?


A. Diarrhea
B. Diaphoresis
C. Urination
D. Miosis
E. Constipation

Copyright © 2000-2020 TIPS Inc. Unauthorized reproduction of this manual is prohibited. This manual is being
used during review sessions conducted by PharmacyPrep. 19-14
Toxicology

Ans: E
Tips: cholinesterase inh. toxic symptoms are (DUMBLESS): Diarrhea, sweating, miosis,
urination, Emesis (vomiting), lacrimation, salivation and shortness of breath.

58. A patient has been admitted in the ER with severe respiratory depression and
unconscious. Patient was on therapeutic dose of hydromorphone. Which of the
following reverses the respiratory depression in patient?
A. SC naloxone
B. IV naloxone
C. IM naloxone
D. flumazenil
E. low dose of codeine
Ans: B
Tips: Naloxone is available as IV, IM and SC. The onset of IV is 2 min; SC & IM is 2-5 min

59. Poison ivy is treated by?


A. Calamine lotion
B. Aluminum acetate
C. Hydrocortisone
D. Antihistamine
E. All of the above
Ans: E
Tips: Poison ivy is weed or wild plants cause allergy upon contact. Can be treated with
calamine lotion, aluminum acetate (Burrow solution), hydrocortisone and antihistamine
po. However antibiotics are NOT used.

60. Which of the following is used as urine acidifier in alkaline toxicity?


A. Ascorbic acid
B. Sodium bicarbonate
C. Ammonium chloride
D. All of the above
Ans: A

61. Acetaminophen toxicity initial symptoms in children?


A. Diarrhea
B. Nausea
C. Vomiting
D. Fever
E. Comma
Ans: C

62. What is the antidote of acetyl salicylic acid?


A. N-acetylcysteine
B. Flumazenil

Copyright © 2000-2020 TIPS Inc. Unauthorized reproduction of this manual is prohibited. This manual is being
used during review sessions conducted by PharmacyPrep. 19-15
Toxicology

C. Naloxone
D. Naltrexone
E. None of the above
Ans: E
Tips: Sodium bicarbonate is the antidote for ASA overdose by enhancing elimination of
ASA in the urine.

63. Unwanted side effect which requires higher dose to achieve the same side effect?
A. Addiction
B. Tolerance
C. Withdrawal symptoms
D. Dependency
Ans: B
Tips: Addiction: Habit forming
Tolerance : to endure or to get same affect
Withdrawal symptoms: discontinued symptoms
Dependency: state of needing drug to function normally

64. Digoxin is a cardiac digitalis widely used in cardiovascular diseases in which toxicity is
increased by all, EXCEPT:
A. Erythromycin
B. Quinidine
C. Cholestyramine
D. Verapamil
E. Amiodarone
Ans: C
Tips: Cholestyramine does not cause digitalis toxicity however if used concomitantly with
digoxin cause significant decrease in absorption.

65. For Organophosphate insecticide toxicity, what is the antidote?


A. Pralidoxime
B. Atropine
C. Succinylcholine
D. Physostigmine
E. Pilocarpine
Ans: B

66. Which of the following agents may be used as barbiturates antidote?


A. Flumazenil
B. Naloxone
C. Naltrexone
D. Aminophylline
Ans: D
Tips: barbiturates antidote is aminophylline

Copyright © 2000-2020 TIPS Inc. Unauthorized reproduction of this manual is prohibited. This manual is being
used during review sessions conducted by PharmacyPrep. 19-16
Toxicology

67. Benzodiazepines cause sedation and over dose of benzodizepine is toxic. Therefore,
these drugs are classified as controlled drugs. What is the treatment of overdose of
benzodiazepines?
A. Flumazenil
B. Naloxone
C. Naltrexone
D. Atropine
Ans: A

68. Unwanted side effects which would require higher dose to achieve same desired effect
is termed as?
A. Addiction
B. Tolerance
C. Withdrawal symptoms
D. Dependency
Ans: B

69. Antidote for barbiturate toxicity?


A. Flumazenil
B. Aminophylline
C. Naltrexone
D. Dimercaprol
Ans: B

70. Which of the following opioid analgesic is most involved in abuse in nursing ward of
hospitals?
A. Fentanyl
B. Morphine
C. Codeine
D. Meperidine
Ans: B
Tips: WWW.cmpa-acpm.ca . Safe use of opioids analgesics in the hospital setting.
Morphine was most frequently involved in the events, followed by hydromorphone and
fentanyl.

71. Which of the following factors is common to ALL forms of drug dependence and abuse?
A. Tolerance
B. Tachyphylaxis
C. Physical dependence
D. Psychological dependence
Ans: D

72. Which of the following are characteristics of opioid overdose?


A. Pinpoint pupil

Copyright © 2000-2020 TIPS Inc. Unauthorized reproduction of this manual is prohibited. This manual is being
used during review sessions conducted by PharmacyPrep. 19-17
Toxicology

B. Hypotension
C. Decrease respiration
D. Coma
E. All of the above
Ans: E

73. The antagonist of choice in the treatment of opioid overdosage is?


A. Naloxone
B. Nalorphine
C. Pentazocine
D. Meperidine
Ans: A

74. Which of the following is NOT an effect of excessive dose of thyroxine?


A. Diarrhea
B. Tachycardia
C. Sensitivity to heat
D. Hypertension
Ans: D

75. Excessive doses of levothyroxine may cause?


A. Sedation
B. Osteoporosis
C. Bradycardia
D. Cold intolerance
E. Weight loss
Ans: E

76. A man was overdosed with Acetaminophen 5 hours ago. At what time should n-
acetylcysteine be administered?
A. It is too late for acetyl cysteine treatment.
B. Can be administered any time after overdose.
C. Can be administered now.
D. Treat by sodium bicarbonate first.
E. There is no antidote for Acetaminophen.
Ans: C

77. Salicylate toxicity treatment includes which of the following?


I. Decontamination with syrup of IPECAC until 30 minutes of overdose
II. CHARCOAL every 6 hours
III. Alkalinization of urine with sodium bicarbonate
A. I only
B. III only
C. I and II only

Copyright © 2000-2020 TIPS Inc. Unauthorized reproduction of this manual is prohibited. This manual is being
used during review sessions conducted by PharmacyPrep. 19-18
Toxicology

D. II and III only


E. I, II and III
Ans: E
Tips: Salicylate toxicity treatment can first be done by control of the vital functions
following administration of sodium bicarbonate to alkalinize the urine and correct the
acidosis, decontamination with syrup of IPECAC until 30 minutes of overdose and
CHARCOAL every 6 hours may be considered. In case of failure of these methods,
hemodialysis should be considered.

Copyright © 2000-2020 TIPS Inc. Unauthorized reproduction of this manual is prohibited. This manual is being
used during review sessions conducted by PharmacyPrep. 19-19
Toxicology

Copyright © 2000-2020 TIPS Inc. Unauthorized reproduction of this manual is prohibited. This manual is being
used during review sessions conducted by PharmacyPrep. 19-20

You might also like